SSC CAPFs-Delhi Sub-Inspectors Examination-2012 Paper-II Question Paper With Answer Key

SSC CAPFs-Delhi Sub-Inspectors Examination-2012 Paper-II
SSC CAPFs-Delhi Sub-Inspectors Examination-2012 Paper-II Question Paper With Answer Key

SSC CAPFs/Delhi Sub-Inspectors Examination-2012 Paper-II Solved Papers

Paper-II

English Language and Comprehension

   Directions-(Q. 1-20) Some parts of the sentences have errors and some have none. Find out which part of a sentence has an error and blacken the oval corresponding to the appropriate letter (A), (B), (C). If a sentence is free from error, blacken the oval corresponding to (D) in the Answer Sheet.

1. Neither the USA nor the UK (A)/ are the largest democracy (B)/ in the twentieth century. (C)/ No error (D)

Answer: (B)

2. One should not (A)/ tell lies (B)/ to his friends. (C)/ No error (D)

Answer: (C)

3. Twelve inches (A)/ makes (B)/ a foot. (C)/ No error (D)

Answer: (B)

4. The jury (A)/ was divided (B)/ in their opinion about the prisoner. (C)/ No error (D)

Answer: (B)

5. I prefer (A)/ dancing (B)/ to singing. (C)/ No error (D)

Answer: (D)

6. Scarcely he (A)/ started the speech (B)/ audience began to leave. (C)/ No error (D)

Answer: (A)

7. While he was on holiday last week (A)/ he wrote a long poetry (B)/ about the cuckoo. (C)/ No error (D)

Answer: (B)

8. The teacher (A)/ said the class to open (B)/ their books at page 40. (C)/ No error (D)

Answer: (B)

9. Professor Gupta teaches (A)/ both Botany as well as Chemistry (B)/ every semester. (C)/ No error (D)

Answer: (B)

10. Bombay’s slums (A)/ are not any better than (B)/ Calcutta. (C)/ No error (D)

Answer: (B)

11. No sooner (A)/ had we set out (B)/ when a thunder storm occurred. (C)/ No error (D)

Answer: (B)

12. Some of you (A)/ speak French (B)/ Isn’t it ? (C)/ No error (D)

Answer: (C)

13. The girl whom you were speaking to (A)/ while I was on the phone (B)/ is my cousin. (C)/ No error (D)

Answer: (A)

14. Coins were (A)/ made of an alloy (B)/ half a gold and half a copper. (C)/ No error (D)

Answer: (C)

15. The price of everything (A)/ is raising (B)/ fast. (C)/ No error (D)

Answer: (B)

16. The principal asked (A)/ Mani and I (B)/ where our teacher was. (C)/ No error (D)

Answer: (D)

17. Speculation is still rife (A)/ as to (B)/ whom will captain the Indian team. (C)/ No error (D)

Answer: (C)

18. We congratulated (A)/ Miss Pushpa (B)/ for her success. (C)/ No error (D)

Answer: (C)

19. Newspaper(A)/ carried a report (B)/ of the plane crash. (C)/ No error (D)

Answer: (C)

20. He had scarcely set foot (A)/ on the road than (B)/ he was knocked down by a car. (C)/ No error (D)

Answer: (B)

   Directions-(Q. 21-25) Sentences are given with blanks to be filled in with an appropriate word(s). Four alternatives are suggested for each question. Choose the correct alternative out of the four and indicate it by blackening the appropriate oval in the Answer Sheet.

21. She did not approve …….. my going abroad for further studies.

(A)  over

(B)  with

(C)  of

(D)  about

Answer: (C)

22. Racial violence erupted throughout the United States………….Martin Luther King was assassinated in April 1968.

(A)  for

(B)  because

(C)  when

(D)  while

Answer: (C)

23. She …….. because she had been waiting for you for an hour.

(A)  is angry

(B)  will be angry

(C)  shall be angry

(D)  was angry

Answer: (A)

24. The leader, with all his men,…….. imprisoned.

(A)  were

(B)  are

(C)  will

(D)  was

Answer: (D)

25. I can imagine no place ………….. interesting than a railway station.

(A)  better

(B)  more

(C)  equally

(D)  as

Answer: (B)

Directions-(Q. 26-28) Out of the four alternatives, choose the one which best expresses the meaning of the given word and mark it in the Answer Sheet.

26. She baffled me with her argument.

(A)  surprised

(B)  perplexed

(C)  convinced

(D)  defeated

Answer: (B)

27. Indigenous

(A)  homely

(B)  indignant

(C)  native

(D)  wholesome

Answer: (C)

28. At loggerheads

(A)  heading

(B)  gorgeous

(C)  logical

(D)  quarrelling

Answer: (D)

   Directions-(Q. 29-31) Choose the word opposite in meaning to the given word and mark it in the Answer Sheet.

29. Wholesome

(A)  Unhealthy

(B)  Incomplete

(C)  Discrete

(D)  Discursive

Answer: (B)

30. Flagrant

(A)  Mediocre

(B)  Meek

(C)  Mild

(D)  Modest

Answer: (*)

31. Ostensible

(A)  insensible

(B)  actual

(C)  apparent

(D)  unostentatious

Answer: (*)

   Directions-(Q. 32-34) There are four different words out of which one is correctly spelt. Find the correctly spelt word and indicate it by blackening the appropriate oval in the Answer Sheet.

32.

(A)  Embarass

(B)  Embarres

(C)  Embarrass

(D)  Embaras

Answer: (C)

33. 

(A)  Courageaous

(B)  Coragageous

(C)  Corrageous

(D)  Courageous

Answer: (D)

34.

(A)  Changeable

(B)  Changable

(C)  Changablle

(D)  Changeble

Answer: (A)

   Directions-(Q. 35-44) Four alternatives are given for the Idiom/Phrase underlined in the sentence. Choose the alternative which best expresses the meaning of the Idiom/Phrase and mark it in the Answer Sheet.

35. It is high time we buried the hatchet and worked for the progress of our country.

(A)  start fighting, kill the enemy

(B)  stop being lazy, work hard

(C)  start burying the hatchet

(D)  stop fighting, be friendly

Answer: (D)

36. I’m sure the new office order serves them right.

(A)  changes their mind

(B)  gives good service to them

(C)  get that they deserve

(D)  realize their mistakes

Answer: (B)

37. He is always in debt as he makes ducks and drakes off his money.

(A)  often squanders money

(B)  grows ducks and hens

(C)  gives loans to other people

(D)  often buys non-vegetarian dishes with

Answer: (A)

38. My teacher is a man of his word.

(A)  one who fulfils his promise

(B)  one who pays lip service

(C)  one who makes no promise

(D)  a man of letters

Answer: (A)

39. Trying to find life on the Mars is a wild goose chase.

(A)  competition where one is prepared to face to consequences

(B)  a futile exercise

(C)  useful enterprise

(D)  mad race for something

Answer: (B)

40. Beware of the wolf in sheep’s clothing.

(A)  terror

(B)  satirist

(C)  sly

(D)  hypocrite

Answer: (D)

41. When he took charge as the General Manager of the company, the situation was at sixes and sevens.

(A)  the money was meagre

(B)  the company was in debt

(C)  only a few workers were there

(D)  in utter confusion

Answer: (D)

42. All the participants were first asked to break the ice soon after the key note address was over.

(A)  to bring out good news

(B)  to accomplish a task

(C)  familiarize with each other

(D)  to get the truth

Answer: (C)

43. The manager gave his secretary her marching orders when he found her work unsatisfactory.

(A)  dismissal

(B)  transfer

(C)  training

(D)  punishment

Answer: (A)

44. After getting selected for the job, he has been putting on airs these days.

(A)  travelling only by air

(B)  putting on weight

(C)  becoming very boastful

(D)  wearing costly clothes

Answer: (C)

   Directions-(Q 45-64) The Ist and the last sentence/parts of the sentence are numbered I and 6. The rest of the sentences/sentence is split into four parts and named P, Q, R and S. These four parts are not given in their proper order. Read the sentences and find out which of the four combinations is correct. Then find the correct answer and indicate in the Answer Sheet.

45. (1) Let us admit

(P) in the number of cars, lorries etc.

(Q) a daily occurrence in the cities

(R) that traffic jam has become

(S) because of the enormous increase

(6) which clog the roads to much.

(A)  RQSP

(B)  SPRQ

(C)  QRPS

(D)  PRSQ

Answer: (A)

46. (1) A recent study has shown :

(P) come out of various colonies of major cities

(Q) the essential arteries of city life.

(R) that about 250 tonnes of plastic waste

(S) which disrupt the sewer system,

(6) Hence ‘Ban on Plastics’.

(A)  SRPQ

(B)  RPSQ

(C)  PSRQ

(D)  PRSQ

Answer: (B)

47. (1) Advertising is a form of

(P) mass communication which is presented in

(Q) and further persuading people to

(R) giving information and arousing interest

(S) form of sign and symbols

(6) buy the product.

(A)  PSRQ

(B)  RSPQ

(C)  QPRS

(D)  SQPR

Answer: (A)

48. (1) Every country popularizes

(P) which are handed down from

(Q) old generation to the new.

(R) some stories known as epic

(S) that facilitates transmission of virtues

(6) values and cultural norms.

(A)  RPQS

(B)  SQPR

(C)  QPRS

(D)  PQRS

Answer: (A)

49. (1) On the last Saturday of March every year,

(P) as people from across the world

(Q) showing their commitment towards

(R) the lights around the globe will go off

(S) take part in the ‘Earth Hour’.

(6) the cause of protecting the earth’s environment

(A)  PRSQ

(B)  PQRS

(C)  RSPQ

(D)  RPSQ

Answer: (D)

50. (1) A thief was trying to break into a house.

(P) The thief tried to run away.

(Q) People chased him and caught hold of him.

(R) The neighbor rushed to the house.

(S) The dog started barking.

(6) He was handed over to the police

(A)  PRSQ

(B)  RPSQ

(C)  SRPQ

(D)  RPQS

Answer: (C)

51. (1) There is a dire need to set up

(P) social homes for the blind

(Q) so as to enable them to enter

(R) where they can learn useful skills

(S) the main streams of society

(6) which will make them self reliant.

(A)  PSRQ

(B)  RSPQ

(C)  PRQS

(D)  SPRQ

Answer: (C)

52. (1) A famous magician

(P) to the college of Arts

(Q) failed to entertain

(R) who was invited

(S) the crowd of students when he showed

(6) his old tricks to them.

(A)  PSQR

(B)  SQPR

(C)  QRSP

(D)  RPQS

Answer: (D)

53. (1) Laugh together without reason for healthy life

(P) by the followers of laughter therapy.

(Q) Those who propound the therapy insist

(R) is a concept propagated

(S) that laughing has various physical and psychological effect,

(6) which are good for our health.

(A)  PRSQ

(B)  PQRS

(C)  PSRQ

(D)  RPQS

Answer: (D)

54. (1) The charity birds hospital is situated in Chandni Chowk in Delhi.

(P) Injured birds are admitted there,

(Q) Which is the only one of its kind in the country.

(R) It was started in 1929.

(S) and after recovery they fly away.

(6) The treatment is free of cost.

(A)  SRPQ

(B)  SRQP

(C)  QRPS

(D)  RSPQ

Answer: (C)

55. (1) Naseeruddin Shah is a legendary actor

(P) later contributed to the Indian film industry

(Q) through his meaningful cinema and has

(R) as a theatre artist and

(S) who started his career

(6) also worked in Hollywood films.

(A)  RSPQ

(B)  SRPQ

(C)  PSQR

(D)  QSRP

Answer: (B)

56. (1) Noise pollution is mainly caused by loudspeakers

(P) which has serious impact on Central Nervous System

(Q) They are used indiscriminately

(R) to cause deafness.

(S) their noise is jarring and harmful

(6) Noise pollution is as harmful as pollution of air or water.

(A)  QSPR

(B)  PRSQ

(C)  SPRQ

(D)  QPRS

Answer: (B)

57. (1) Sahil was very disappointed

(P) for ten  years with sincerity and

(Q) any increment in salary

(R) because he worked for his company

(S) diligence but was never given

(6) or appreciation for his work.

(A)  SQPR

(B)  RPSQ

(C)  QRSP

(D)  PSQR

Answer: (B)

58. (1) Democracy is a perfect system

(P) provided it is lubricated

(Q) goal oriented individuals, who

(R) and put in the hands of

(S) work with dedication

(6) for the maximum good of the largest number of people.

(A)  PRQS

(B)  RPSQ

(C)  QSPR

(D)  SQRP

Answer: (A)

59. (1) When a person

(P) put in sincere and diligent

(Q) strives for perfection

(R) efforts along with authentic

(S) in his work, he must

(6) and novel ideas of his own.

(A)  SPQR

(B)  RQPS

(C)  QSPR

(D)  PSQR

Answer: (C)

60. (1) Her mother often

(P) tuition fee after the

(Q) failed to pay for

(R) sudden death of Madhu’s father

(S) the books and school.

(6) who died many years ago.

(A)  PSQR

(B)  QSPR

(C)  SPRQ

(D)  RSQP

Answer: (B)

61. (1) The press should help to create

(P) who govern the fate of nation

(Q) the masses and those

(R) a healthy understanding between

(S) and reflect reactions and opinions

(6) to the contemporary issues concerning them.

(A)  PQRS

(B)  SPRQ

(C)  RQPS

(D)  QRPS

Answer: (C)

62. (1) Character is that

(P) If we want to build a great nation,

(Q) to look upto the men of vision and sacrifice

(R) on which the destiny of a nation is built.

(S) we must try to train young men and women

(6) and emulate them.

(A)  RPSQ

(B)  QPRS

(C)  SRPQ

(D)  PRSQ

Answer: (A)

63. (1) India has the capacity

(P) provided it utilized its

(Q) to emerge as a super power

(R) material resources and manpower

(S) in the next century,

(6) effectively and efficiently.

(A)  RSPQ

(B)  QSPR

(C)  SPRQ

(D)  PQRS

Answer: (B)

64. (1) We first became aware

(P) who sat at one table

(Q) when one of the ship’s officers came up to the chief engineer.

(R) that something unusual was happening

(S) and spoke to him in a low voice

(6) which made us feel that something was wrong.

(A)  PRQS

(B)  SRPQ

(C)  RSPQ

(D)  RQPS

Answer: (D)

   Directions-(Q. 65-76) Out of the four alternatives choose the one which can be substituted for the given words/sentence.

65. A supporter for the cause of women :

(A)  Feminist

(B)  Womanish

(C)  Hero

(D)  Colleague

Answer: (A)

66. Conveyance of property as security for debt until money is repaid

(A)  subterfuge

(B)  refuge

(C)  mortgage

(D)  reridity

Answer: (C)

67. ‘One who has knowledge of everything’.

(A)  satisfactory

(B)  fool proof

(C)  trust worthy

(D)  omniscient

Answer: (D)

68. An illness of accident that causes death.

(A)  fatal

(B)  eternal

(C)  mortar

(D)  fatalist

Answer: (A)

69. Science of heavenly bodies:

(A)  Astronomy

(B)  Archaeology

(C)  Anatomy

(D)  Astrology

Answer: (A)

70. A person who is indifferent to pleasure and pain.

(A)  apostate

(B)  sceptic

(C)  stoic

(D)  patriot

Answer: (C)

71. ‘A remedy for all diseases or cure-all’.

(A)  panacea

(B)  antidote

(C)  quick-fix

(D)  nullifier

Answer: (A)

72. Relating to the car:

(A)  Nasal

(B)  Oral

(C)  Renal

(D)  Aural

Answer: (D)

73. One who eats every kind of food.

(A)  Gourment

(B)  Omnivore

(C)  Herbivore

(D)  Cannibal

Answer: (B)

74. The book containing life story of a man.

(A)  Fable

(B)  Travelogue

(C)  Biography

(D)  Autobiography

Answer: (C)

75. A specialist who studies about snakes.

(A)  Snake charmer

(B)  Ophiodist

(C)  Ophthalmologist

(D)  Ophiologist

Answer: (A)

76. The act of killing one self by cutting open one’s stomach with a sword, to avoid losing honour-

(A)  sati

(B)  assassination

(C)  self-immolation

(D)  hara-kiri

Answer: (D)

   Direction-(Q. 77-96) A sentence has been given in Active Voice/Passive Voice. Out of the four alternatives suggested, select the one which best expresses the same sentence in Passive/Active Voice and mark you answer in the Answer-Sheet.

77. He offered me all the money at his command.

(A)  I was offered all the money at his command.

(B)  I am being offered all the money at his command.

(C)  I had been offered all the money at his command.

(D)  I have been offered all the money at his command.

Answer: (A)

78. We must thank God for all the good things.

(A)  Let us thank God for all the good things.

(B)  God must be thanked for all the good things.

(C)  God could be thanked for all good things.

(D)  God must be thanked by us.

Answer: (B)

79. The comic scenes in the play were over done by the actors.

(A)  The actors over did the comic scenes in the play.

(B)  The actors had over done the comic scenes in the play.

(C)  The actors are over doing the comic scenes in the play

(D)  The actors over do the comic scenes in the play.

Answer: (A)

80. Little strokes fell great oaks.

(A)  Great oaks fell with little strokes

(B)  Great oaks were felled by little strokes

(C)  Great oaks can be felled with little strokes

(D)  Great oaks are felled by little strokes

Answer: (B)

81. Each member of the academy was given a prize.

(A)  Every member of the academy got a prize

(B)  They give a prize to each member of the academy

(C)  They are giving a prize to each member of the academy

(D)  They gave a prize to each member of the academy

Answer: (D)

82. Rabindranath Tagore wrote the ‘Gitanjali’.

(A)  The ‘Gitanjali’ was written by Rabindranath Tagore.

(B)  The ‘Gitanjali’ is written by Rabindranath Tagore.

(C)  The ‘Gitanjali’ is being written by Rabindranath Tagore.

(D)  The ‘Gitanjali’ has been written by Rabindranath Tagore.

Answer: (A)

83. When will you return my books?

(A)  When will my books be returned?

(B)  When my books will be returned?

(C)  When are my books returned by you?

(D)  When will my books return?

Answer: (A)

84. The Government is building new bridges.

(A)  New bridges were built by the government.

(B)  New bridges are building by the government.

(C)  New bridges have been built by the government.

(D)  New bridges are being built by the government.

Answer: (D)

85. The authorities are distributing food packets to the flood victims.

(A)  Food packets are distributed to the flood victims by the authorities.

(B)  Food packets is being distributed to the flood victims by the authorities.

(C)  Food packets were being distributed to the flood victims by the authorities.

(D)  Food packets are being distributed to the flood victims by the authorities.

Answer: (D)

86. They returned my money with interest.

(A)  My money was return with interest.

(B)  My money was returned with interest.

(C)  My money is being returned with interest.

(D)  My money had been returned by them.

Answer: (B)

87. He was given the details of his uncle’s will be the lawyer.

(A)  The lawyer gives him the details of his uncle’s will.

(B)  The lawyer has to give him the details of his uncle’s will.

(C)  The lawyer will be giving him the details of his uncle’s will.

(D)  The lawyer gave him the details of his uncle’s will.

Answer: (D)

88. All vulnerable places are kept under constant vigil by the Policeman.

(A)  The Policeman keep all vulnerable places under constant vigil.

(B)  The Policeman are keeping all vulnerable places under constant vigil.

(C)  The Policeman will be keeping all vulnerable places under constant vigil.

(D)  The Policeman will keep all vulnerable places under constant vigil.

Answer: (A)

89. Big Cars must be driven very carefully.

(A)  Drivers drive big cars very carefully

(B)  Drives have to drive big cars carefully

(C)  Drivers are driving big cars carefully

(D)  Drivers must drive big cars very carefully

Answer: (D)

90. You may choose the book.

(A)  The book may be choosed by you.

(B)  The book may be choosen by you.

(C)  The book may choose by you.

(D)  The book is choosen by you.

Answer: (B)

91. The board selects the members.

(A)  The members shall be selected by the board.

(B)  The members are selected by the board.

(C)  The members are being selected by the board.

(D)  The members were selected by the board.

Answer: (B)

92. Many children have been kidnapped by the militants.

(A)  The militants have kidnapped many children

(B)  The militants are kidnapping many children

(C)  The militants kidnapped many children

(D)  The militants had kidnapped many children

Answer: (A)

93. Follow all the instruction carefully.

(A)  All the instruction could be followed carefully

(B)  Let all the instruction be followed carefully

(C)  Instruction may be followed carefully

(D)  Let instruction be followed carefully

Answer: (B)

94. The thieves victimize the innocent commoners.

(A)  The innocent commoners had been victimized by the thieves

(B)  The innocent commoners will be victimized by the thieves

(C)  The innocent commoners are victimized by the thieves

(D)  The innocent commoners were victimized by the thieves

Answer: (C)

95. The Car is being washed by the driver

(A)  The driver has been washing the Car

(B)  The driver is washing the Car

(C)  The driver has washed the Car

(D)  The driver was washing the Car

Answer: (B)

96. Das has posted the letter.

(A)  The letter has been posted by Das.

(B)  The letter has posted by Das.

(C)  The letter had been posted by Das.

(D)  The letter have been posted by Das.

Answer: (A)

   Directions-(Q. 97-118) A part of the sentence is underlined. Below are given alternatives to the underlined part at (A), (B), (C) which many improve the sentence. Choose the correct alternative. In case no improvement is needed, your answer is (D). Mark your answer in the Answer Sheet.

97. We are looking forward to seeing you at next year’s conference.

(A)  to see

(B)  seeing

(C)  that we will see

(D)  no improvement

Answer: (B)

98. To assist and English course can be an interesting experience.

(A)  To attend

(B)  Attending

(C)  Assisting

(D)  no improvement

Answer: (B)

99. The number of orders went up because we increased our prices by 15%.

(A)  when

(B)  although

(C)  if

(D)  no improvement

Answer: (B)

100. How many copies did we sent with the order ?

(A)  sent we

(B)  did we send

(C)  have we send

(D)  no improvement

Answer: (B)

101. If the components would have been delivered earlier, we might have been able to start the work on time.

(A)  were

(B)  may have been

(C)  had been

(D)  no improvement

Answer: (A)

102. The warehouse entrance is opposite to the main car park.

(A)  opposite

(B)  opposite of

(C)  opposite from

(D)  no improvement

Answer: (D)

103. The paper started on time, didn’t it ?

(A)  hadn’t it ?

(B)  did it ?

(C)  isn’t it ?

(D)  no improvement

Answer: (D)

104. The level of discount is depending on the size of the order that is placed.

(A)  depends on

(B)  depends with

(C)  is depending upon

(D)  no improvement

Answer: (C)

105. Have you finish writing the report of the meeting?

(A)  Have you finished

(B)  Do you finish

(C)  Will you have finished

(D)  no improvement

Answer: (A)

106. Kaul has been head of this department since ten years.

(A)  has been head of this department for

(B)  is head of this department for

(C)  is head of this department since

(D)  no improvement

Answer: (A)

107. He went out of his way to oblige his superiors.

(A)  on his way

(B)  in his way

(C)  beyond his way

(D)  no improvement

Answer: (C)

108. He told us that he had been applying for a new job.

(A)  had applied

(B)  applies

(C)  is applying

(D)  no improvement

Answer: (A)

109. Their product is marketed more imaginatively this reason.

(A)  is been marketed

(B)  is being marketed

(C)  is marketing

(D)  no improvement

Answer: (B)

110. The new price lists are being printing and will be available in a few days.

(A)  will be printed

(B)  are printed

(C)  are being printed

(D)  no improvement

Answer: (C)

111. The documents arrived on Thursday, didn’t they ?

(A)  weren’t they ?

(B)  haven’t they ?

(C)  arrived they ?

(D)  no improvement

Answer: (B)

112. If I don’t succeed this year, I will give the examination again next 

(A)  will take

(B)  will repeat

(C)  shall give

(D)  no improvement

Answer: (B)

113. I have just sent our brochure, which should arrive next week.

(A)  I just have sent

(B)  Just I have sent

(C)  I have just send

(D)  no improvement

Answer: (D)

114. If you placed ice in warm water, it soon melts.

(A)  will place

(B)  would place

(C)  place

(D)  no improvement

Answer: (C)

115. I can’t be put with this insult.

(A)  put upon

(B)  put up

(C)  put by

(D)  no improvement

Answer: (B)

116. Iyer is the man, whose office you’ll be sharing this month.

(A)  man, of whom the

(B)  man with whom the

(C)  man whose

(D)  no improvement

Answer: (D)

117. When the post will arrive, I’ll bring it to your office.

(A)  arrives

(B)  shall arrive

(C)  is going to arrive

(D)  no improvement

Answer: (A)

118. I can’t find my glasses, you saw them anywhere?

(A)  have you seen

(B)  saw you

(C)  are  you seeing

(D)  no improvement

Answer: (A)

   Directions-(Q. 119-145) A sentence has being given in Direct/Indirect form. Out of the four alternatives suggested, select the one which best expresses the same sentence in Direct/Indirect form and mark  your answer in the Answer Sheet.

119. Rohit asked me, “Have you read this novel?”

(A)  Rohit asked me if I had read that novel.

(B)  Rohit asked me if I could read this novel.

(C)  Rohit asked me if I would have read that novel.

(D)  Rohit asked me if I was reading that novel.

Answer: (A)

120. “Who was the first man to fly in space ?”

(A)  The examiner asked who the first man to fly in space was.

(B)  The examiner asked about the first man to fly in space.

(C)  The examiner questioned about the first man to fly in space.

(D)  The examiner asked who was the first man to fly in space.

Answer: (A)

121. The teacher said, “Neha why are you late again today?”

(A)  The teacher asked Neha why was she late again that day.

(B)  The teacher asked Neha why she was late again today.

(C)  The teacher asked Neha why she was

(D)  The teacher asked Neha why she was late again that day.

Answer: (A)

122. I was told to leave the room.

(A)  He told me that I should leave the room.

(B)  He told me to leave the room.

(C)  He told me that I should be leaving the room.

(D)  He said to me to leave the room.

Answer: (B)

123. The teacher said, “Boys, what do you understand by this phrase?”

(A)  The teacher asked the boys what they understood by that phrase.

(B)  The teacher asked the boys that they understood by this phrase.

(C)  The teacher asked the boys that they understand by this phrase.

(D)  The teacher asked by boys what they understand by that phrase.

Answer: (A)

124. The engineers located the fault in the machine.

(A)  The fault in the machine was to be located by the engineers.

(B)  In the machine, the fault was located by the engineers.

(C)  The fault in the machine would be located by the engineers

(D)  The fault in the machine was located by the engineers.

Answer: (D)

125. The passenger said to the clerk, ‘When is the next train to Hyderabad?”

(A)  The passenger asked the clerk when the next train to Hyderabad was.

(B)  The passenger enquired the clerk that when the next train was to Hyderabad.

(C)  The passenger enquired the clerk when was the next train to Hyderabad.

(D)  The passenger enquired the clerk when was the next train to Hyderabad.

Answer: (A)

126. The patient asked the doctor what he should normally eat.

(A)  The patient exclaimed, “What should I normally eat, Doctor?”

(B)  The patient said, “What should I normally eat Doctor?”

(C)  The patient told, “What should I normally eat ?”

(D)  The patient said, “What should I normally eat doctor?”

Answer: (D)

127. His angry mother jeered, “Do you suppose you know better than your father ?”

(A)  His angry mother jeered and asked if he was supposed to know better than his father.

(B)  His angry mother jeered and asked whether he knew better than this father.

(C)  His angry mother jeered and asked whether he supposed that he would know better than his father.

(D)  His angry mother jeered and masked whether he supposed that he knew better than his father.

Answer: (D)

128. He said to her, ‘Where is she going’ ?

(A)  He asked her where she was going.

(B)  He wanted to know where she is going.

(C)  He wants to know where she was going.

(D)  He said where she was going.

Answer: (A)

129. “I have to go into the town today”, Mr. Rao said to the Station Master.

(A)  Mr. Rao told the Station Master that he had to go into the town that day.

(B)  Mr. Rao advised the Station Master that he had to go into the town that day.

(C)  Mr. Rao spoke to the Station Master that he had to go into the town that day.

(D)  Mr Rao told the Station Master that he would go into the town that day.

Answer: (A)

130. ‘I’ll take a quick nap here in this soft grass’, said the hare.

(A)  The hare said that he would take a quick nap here in this soft grass.

(B)  The hare said that he would take a quick nap there in that soft grass.

(C)  The hare said he will take a quick nap here in this soft grass.

(D)  The hare said that I will take a quick nap here in this soft grass.

Answer: (B)

131. My mother said to me, “Where are you going at this time of the day ?”

(A)  My mother asked me where I was going at that time of the day.

(B)  My mother told me where I was going at that time of the day

(C)  My mother told me not to go at that time of the day.

(D)  My mother asked me where was I going at that time of the day.

Answer: (A)

132. He said, “Oh, that I had the wings of a bird”.

(A)  He proposed to have the wings of a bird.

(B)  He wished that he might have the wings of a bird.

(C)  He wished that he could have the wings of a bird.

(D)  He wished that he had the wings of a bird.

Answer: (B)

133. She said to Dhiru, “I am planning to leave tomorrow”.

(A)  She told Dhiru that she had planned to leave the next day.

(B)  She told Dhiru that she is planning to leave the next day.

(C)  She advised Dhiru that she was planning to leave next day.

(D)  She told Dhiru that she was planning to leave the next day.

Answer: (D)

134. Krishna said to his friends, ‘Let us go to a movie today’.

(A)  Krishna suggested to his friends that let them go to a movie that day.

(B)  Krishna proposed to his friends if they would go to a movie that day.

(C)  Krishna proposed to his friends what if they would go to a movie that day.

(D)  Krishna suggested to his friends that they should go to a movie that day.

Answer: (D)

135. Mother said to the baby, “Don’t play with fire”.

(A)  Mother warned the baby to play with fire.

(B)  Mother warned the baby not to play with fire.

(C)  Mother forbade the baby not to play with fire.

(D)  Mother requested the baby not to play with fire.

Answer: (C)

136. “Leave the room at once Ravi”, he shouted.

(A)  He ordered Ravi to leave the room at once.

(B)  He forbade Ravi to leave the room at once.

(C)  He suggested that Ravi should leave the room at once.

(D)  He requested Ravi to leave the room at once.

Answer: (A)

137. They said to him, “We shall visit your house tomorrow”.

(A)  They said that they will go to his house the next day.

(B)  They expressed a desire to visit his house the next day.

(C)  They told him that they might visit his house the next day.

(D)  They told him that they would visit his house the next day.

Answer: (D)

138. “Are we never to meet ?” Amit asked him.

(A)  Amit questioned him whether they were ever to be meet.

(B)  Amit wondered whether they are never to meet.

(C)  Amit said whether they were never to meet.

(D)  Amit asked him whether they were never to meet.

Answer: (D)

139. He said, ‘If I had the key with me, I could give you the solution’.

(A)  He said that if he had the key with him, he could given him/her the solution.

(B)  He said if he could have they key with him, he would give him/her the solution.

(C)  He told that if he could have the key with him, he would given him/her the solution.

(D)  He said that if he have had the key with  him, he could have given him/her the solution.

Answer: (D)

140. He said to her, “Rest assured. I shall repay your loan”.

(A)  He assured her that he would repay her loan.

(B)  He told her that he would repay her loan.

(C)  He said that he might repay her loan.

(D)  He told her not to worry about the repayment of her loan.

Answer: (A)

141. “Do you know anything about Robots, Sonu ?”

(A)  Meeta asked Sonu whether he knows anything about Robots.

(B)  Meeta asked Sonu if he know anything about Robots.

(C)  Meeta asked Sonu if he had known anything about Robots.

(D)  Meeta asked Sonu if he knew anything about Robots.

Answer: (D)

142. “Please bring me a glass of water”, she said to me.

(A)  She requested to bring her a glass of water.

(B)  She requested me brought her a glass of water.

(C)  She requested me to bring her a glass of water.

(D)  She requested me that bring her a glass of water.

Answer: (C)

143. The gardener said to the boys, “Do not pluck the flowers”.

(A)  The gardener said to the boys that they should not pluck flowers.

(B)  The gardener forbade the boys to pluck the flowers.

(C)  The gardener told the boys that they should not pluck flowers.

(D)  The gardener scolded the boys for plucking flowers.

Answer: (B)

144. The teacher said, “Surabhi, where are you going to spend your summer break this  year?”

(A)  The teacher asked Surabhi where she is going to spend her summer break.

(B)  The teacher asked Surabhi where was she going to spend her summer break.

(C)  The teacher asked to Surabhi where she was going to spend her summer break.

(D)  The teacher asked Surabhi where she was going to spend her summer break.

Answer: (B)

145. The General said, “Bravo ! Well done, my soldiers”.

(A)  The General told the soldiers that they had done well.

(B)  The General applauded his soldiers saying that they would do well.

(C)  The General applauded his soldiers for their excellent job.

(D)  The General applauded his soldiers saying that they had done well.

Answer: (D)

   Directions-(Q. 146-170) You have following three passages with 10 questions each in passage I and II and 5 questions in passage-III. Read the passages carefully and chose the best answer to fill in the blanks out of the four alternatives given and mark it in the answer sheet.

Passage-I (146-155)

   Men had to save himself …(146)… the attacks of animals and …(147)… cold of the winter season. He …(148)… how to light fire …(149)… probably the greatest invention in …(150)… world. Necessities grow with the …(151)… in times. Had there been …(152)… disease, the necessity to invent …(153)… would have never arisen. It …(154)… necessary to save man from the vagaries of …(155)…, house was invented, clothes were invented.

146.

(A)  on

(B)  for

(C)  from

(D)  by

Answer: (C)

147.

(A)  acute

(B)  dreadful

(C)  very

(D)  severe

Answer: (D)

148.

(A)  discovered

(B)  find out

(C)  decided

(D)  invented

Answer: (A)

149.

(A)  great

(B)  most

(C)  much

(D)  more

Answer: (B)

150.

(A)  this

(B)  such

(C)  the

(D)  one

Answer: (C)

151.

(A)  movement

(B)  Exchange

(C)  Span

(D)  change

Answer: (D)

152.

(A)  no

(B)  other

(C)  some

(D)  any

Answer: (A)

153.

(A)  medicine

(B)  Technology

(C)  Surgery

(D)  Science

Answer: (A)

154.

(A)  was

(B)  might be

(C)  could be

(D)  became

Answer: (B)

155.

(A)  fashion

(B)  idea

(C)  Time

(D)  nature

Answer: (D)

Passage-II (156-165)

   Pleasure and pain are two sides of the coin in one’s life. The great Roman orator Cicero, in …(156)… celebrated treatise on friendship remarks …(157)… truth that it increases happiness and …(158)… misery by the doubling …(159)… our joy and the …(160)… of our grief. When we do well, it is …(161)… to have friends who are …(162)… of our success that they …(163)… as much pleasure from it as we do …(164)… . For the …(165)… man, the attainment of wealth’s of little value.

156.

(A)  his

(B)  her

(C)  which

(D)  whose

Answer: (A)

157.

(A)  for

(B)  on

(C)  under

(D)  with

Answer: (D)

158.

(A)  creates

(B)  improves

(C)  diminishes

(D)  develops

Answer: (C)

159.

(A)  with

(B)  of

(C)  off

(D)  in

Answer: (B)

160.

(A)  division

(B)  retouch

(C)  rebuff

(D)  removal

Answer: (D)

161.

(A)  delightful

(B)  open

(C)  needful

(D)  good

Answer: (D)

162.

(A)  scornful

(B)  worried

(C)  proud

(D)  nice

Answer: (C)

163.

(A)  understand

(B)  receive

(C)  retrieve

(D)  derive

Answer: (D)

164.

(A)  it

(B)  please

(C)  them

(D)  ourselves

Answer: (D)

165.

(A)  friendly

(B)  nice

(C)  friendless

(D)  tall

Answer: (B)

Passage-III (166-170)

   Communication is the most …(166)… aspect of life. Like human beings, insects, birds and animals are also said to have some …(167)… of communication. It has been …(168)… that insects …(169)…to chemical communication of messages. Some female insects use their scent as communication to attract the males. The ants are said …(170)… trails which are used by other ants for transporting food.

166.

(A)  exclusive

(B)  essential

(C)  innovative

(D)  ordinary

Answer: (B)

167.

(A)  plan

(B)  proof

(C)  attempts

(D)  system

Answer: (D)

168.

(A)  invented

(B)  hoped

(C)  proved

(D)  suggested

Answer: (C)

169.

(A)  resort

(B)  attempt

(C)  pleased

(D)  want

Answer: (A)

170.

(A)  to leave

(B)  to be leaving

(C)  to have left

(D)  that they leave

Answer: (A)

   Directions-(Q. 171-200) You have following three brief passages with 10 questions in each passage. Read the passage carefully and choose the best answer to each question of the four alternatives and mark it in the Answer Sheet.

Passage-I (171-180)

   In concert with the HUDCO and State Govts., the Ministry has launched a National programme for setting up Nirman or Nirmithi Kendras (Building Centres) for training of artisans in low cost construction skills and production of building materials and components through innovative techniques. Set with a target of 100 Building Centres, so for 86 Centres have been identified out of which 38 centres had already been set  up by the end of January, 1989.

   Till the end of the 6th Plan, 235 towns benefitted under the Scheme and a sum of Rs 63.57 crores was provided as Central assistance. The Scheme has been continued during the 7th Plan with an outlay of Rs 88.00 crores part of which is to be utilized for funding the on-going schemes and the remaining amount is to be utilized for schemes to be taken up in 102 additional towns in various States/UTs. 107 towns have been brought under the purview of the scheme and Rs 63.49 crores have been made available as Central assistance to the States/UTs till 29.3.1989, including Rs 15.84 crores released during 1988-89.

   An expenditure of approximately Rs 28 crores is likely to be incurred during the current year on construction of residential and non-residential accommodation in general pool. More than 5800 quarters are under construction in general pool out of which approximately 3000 quarters will be completed during the current year.

171. The target of the National programme was to build-

(A)  100 centres

(B)  86 centres

(C)  38 centres

(D)  not less than 265 centres

Answer: (A)

172. Innovative techniques means-

(A)  stylish techniques

(B)  new techniques

(C)  unique ways

(D)  wonderful technique

Answer: (B)

173. In the context of the passage, current year should stand for-

(A)  88-89

(B)  87-88

(C)  86-87

(D)  89-90

Answer: (A)

174. The programme was meant for-

(A)  farming

(B)  health care

(C)  education

(D)  housing

Answer: (D)

175. The passage appears to be a part of a-

(A)  report

(B)  press release

(C)  diary entry

(D)  proposal

Answer: (A)

176. In the context of the given passage, concert means-

(A)  company

(B)  collaboration

(C)  communication

(D)  connection

Answer: (B)

177. The word approximately is-

(A)  an adverb

(B)  a noun

(C)  determiner

(D)  an adjective

Answer: (D)

178. The National programme for setting up Nirman Kendras is taken care of by-

(A)  the central Governments

(B)  the state Governments

(C)  private parties

(D)  both (A) and (B)

Answer: (D)

179. In the context of the passage, UTs stands for-

(A)  Unit trust

(B)  Union traders

(C)  Union territories

(D)  United travels

Answer: (A)

180. Nirmithi or Nirman Kendras are meant for training-

(A)  skilled workers

(B)  farmers

(C)  artists

(D)  None of the above

Answer: (C)

Passage-II (181-190)

   In looking at Air India in the 21st Century, I believe that the airline has an intrinsic advantage over its competitors. It being the national carrier of one of the largest markets in the world and the fact that it holds merely 25 per cent of the present market share in and out of India provides it with a unique opportunity of increasing it to atleast 50 per cents by the turn of the century. I believe that Air India has an inherent advantage in achieving this goal as it represents a nation which is traditionally known for its hospitality and culture which give it a cutting edge in the service industry.

   All of us who are familiar with the technological excellence of the west, are also distinctly conscious of the lack of a service culture to support the same. Natural courtesy and friendliness must however, be polished with professionalism and I believe that this attribute will contribute to the excellence of Air India in the future and provide it with the competitive edge it requires. Air lines which are deemed ‘great’ in our part of the world today base their product advantage on this attribute. I see a revival of a style which was created for aviation industry by Air India.

181. Which part of speech is the word distinctly?

(A)  verb

(B)  adverb

(C)  adjective

(D)  noun

Answer: (B)

182. India has an edge over the west because of-

(A)  a service culture

(B)  technological excellence

(C)  professionalism

(D)  All the above

Answer: (A)

183. Choose the most appropriate title for the passage-

(A)  Air India : a competitive airlines

(B)  Air India : an embodiment of Hospitality

(C)  Air India : an Airlines with a History

(D)  Air India : an economy airlines

Answer: (A)

184. In the context of the passage, professionalism implies-

(A)  good knowledge of technology

(B)  refined behaviour

(C)  courtesy and friendliness

(D)  Both (A) and (B)

Answer: (D)

185. The passage seems to have been written-

(A)  in the early nineties

(B)  a couple of years ago

(C)  just before the close of the 20th century

(D)  at the beginning of the 21st century

Answer: (D)

186. The writer of the passage sounds ………. About the future of Air India.

(A)  optimistic

(B)  uncertain

(C)  pessimistic

(D)  skeptical

Answer: (A)

187. The intrinsic advantage that Air India has over the other airline is-

(A)  It has a large market share

(B)  It has an excellent service culture

(C)  It is highly professional

(D)  It is the national air lines

Answer: (A)

188. The phrase this goal refers to-

(A)  a cutting edge in the service industry

(B)  excel in hospitality

(C)  increasing the market share to 50 per cent

(D)  All the above

Answer: (D)

189. The last sentence is a/an of/for Air India-

(A)  advertisement

(B)  analytic study

(C)  eulogy

(D)  critique

Answer: (B)

190. In the context of the passage, competitive edge means-

(A)  spirit of competition

(B)  privileged

(C)  has a great potential

(D)  the end of competition

Answer: (A)

Passage-III (191-200)

   The Director/Deputy Secretary in charge of Administration in the Ministry continued to function as Liaison Officer for the work relating to employment of ex-servicemen. Additional Liaison Officers were also nominated in the Attached/Sub-ordinate Offices and Public Sector Undertakings to watch the implementation of the Government rules, regulations and orders on the subject. Statistics relating to appointment of ex-servicemen in the Ministry, its Attached/Subordinate Offices and Public Sector Undertakings are given in Appendix-V.

   During the period of 87-88, the workload of the Department registered a growth of more than 15%. In the year 88-89, the workload is expected to increase by more than 35%.

   A Special Cell in the Ministry watches the implementation of Government orders regarding reservation in services for Scheduled Castes and Scheduled Tribes. The Cell, in addition to carrying out periodic inspections of the rosters maintained to ensure compliance of the Government orders, also looks into complaints, if any, received from the employees belonging to Scheduled Castes and Scheduled Tribes.

   Statistics relating to the representation of Scheduled Castes and Scheduled Tribes in the Ministry and its other offices are contained in Appendices VI to IX.

191. The passage could be an extract from-

(A)  a Govt. circular

(B)  a rule book

(C)  an inter office memo

(D)  None of the above

Answer: (A)

192. Tick the option which is false. The work of the special cell is-

(A)  looking into complaints

(B)  to oversee the promotion roster

(C)  implementation of government orders

(D)  periodic inspection

Answer: (C)

193. Appendix means an attachment containing-

(A)  corrections

(B)  addition

(C)  omissions

(D)  epilogue

Answer: (B)

194. Liason officer is-

(A)  one who exchanges information between different organisations

(B)  one who takes care of every one’s personal problems

(C)  incharge of the personal deptt.

(D)  None of the above

Answer: (D)

195. Which part of speech is the word Implementation ?

(A)  Verb

(B)  noun

(C)  adjective

(D)  adverb

Answer: (B)

196. Appendices VI to IX contain the-

(A)  Government orders

(B)  complaints by the public

(C)  report of periodic inspection

(D)  statistics regarding the representation of SC’s and ST’s

Answer: (D)

197. In the context of the passage cell means-

(A)  a place where monks and nuns live

(B)  a small group within a larger organisation

(C)  the smallest part of a plant

(D)  a small room where prisoners are locked

Answer: (B)

198. Periodic means something-

(A)  regularly

(B)  often

(C)  seldom

(D)  happening occasionally

Answer: (D)

199. The interests of the SC’s and ST’s are looked after by-

(A)  a special cell

(B)  the deputy secretary

(C)  the liason officer

(D)  the director

Answer: (A)

200. Compliance of government orders means-

(A)  execution of orders

(B)  complaints against government orders

(C)  take care of order

(D)  None of the above

Answer: (C)

SSC CAPFs-Delhi Sub-Inspectors Examination-2012 Paper-I Question Paper With Answer Key

SSC CAPFs-Delhi Sub-Inspectors Examination-2012 Paper-I
SSC CAPFs-Delhi Sub-Inspectors Examination-2012 Paper-I Question Paper With Answer Key

SSC CAPFs/Delhi Sub-Inspectors Examination-2012 Paper-I Solved Papers

Part-A

General Intelligence & Reasoning

   Directions-(Q. 1 to 9) Select the related letter/word/number from the given alternatives-

1. SKIT : TIKS : : ? : FLAP

(A)  PLFA

(B)  PFAL

(C)  PLAF

(D)  PALF

Answer: (D)

2. 6 : 72 :: 8 : ?

(A)  94

(B)  96

(C)  74

(D)  92

Answer: (B)

3. 6 : 36 : : 9 : ?

(A)  81

(B)  98

(C)  42

(D)  56

Answer: (A)

4. 384 : ? : : 216 : 63

(A)  128

(B)  124

(C)  112

(D)  192

Answer: (C)

5. Asthma : Lungs : : Conjunctivitis : ?

(A)  Bones

(B)  Teeth

(C)  Eyes

(D)  Blood

Answer: (C)

6. Communicable disease : Malaria : : Non-communicable disease : ?

(A)  Tuberculosis

(B)  Hepatitis

(C)  AIDS

(D)  Cancer

Answer: (D)

7. Thermometer : Temperature : : Glucometer : ?

(A)  Body sugar

(B)  Body resistance

(C)  Blood

(D)  Blood sugar

Answer: (D)

8. Dismay : Joy : : Tend : ?

(A)  Regret

(B)  Ignore

(C)  Negligible

(D)  Spoil

Answer: (B)

9. DCBA : WXYZ : : ? : ZYXW

(A)  ABCD

(B)  ABDC

(C)  CBAD

(D)  BCDA

Answer: (A)

Directions-(Q. 10 to 14) Find the odd word/number/letters from the given alternatives-

10.

(A)  4249

(B)  7586

(C)  4623

(D)  9781

Answer: (C)

11.

(A)  ee gg ii kk

(B)  pp rr tt uu

(C)  bb dd ff hh

(D)  kk mm oo qq

Answer: (B)

12.

(A)  X

(B)  P

(C)  D

(D)  I

Answer: (D)

13.

(A)  Pen

(B)  Letter-head

(C)  Paper

(D)  Envelope

Answer: (A)

14.

(A)  Reed

(B)  Pit

(C)  Tab

(D)  Hold

Answer: (A)

Directions-(Q. 15 to 23) A series are given, with one term missing. Choose the correct alternative from the given one that will complete the series-

15. CDF, DEG, EFH ….

(A)  FHG

(B)  EFI

(C)  FGH

(D)  FGI

Answer: (D)

16. ZA, VC, RE, NG, ……., FK

(A)  OH

(B)  KI

(C)  IJ

(D)  JI

Answer: (D)

17. 4, 16, 36, …., 100

(A)  56

(B)  64

(C)  48

(D)  52

Answer: (B)

18. 2, 3, 10, 15, 26, …..

(A)  30

(B)  52

(C)  35

(D)  24

Answer: (C)

19. 12, 23, 34, 45, ….., 67, 78, 89

(A)  76

(B)  69

(C)  56

(D)  54

Answer: (C)

20. EAB, GCD, MIJ, ….

(A)  ROP

(B)  QOP

(C)  OKL

(D)  FOP

Answer: (C)

21. ACE, BDF, GIK, ….

(A)  JHM

(B)  HJI

(C)  HJL

(D)  MJH

Answer: (C)

22. WBP, SGM, OLJ, …

(A)  NPH

(B)  KQG

(C)  LPG

(D)  MQG

Answer: (B)

23. DEF, HIJ, MNO, …

(A)  RTV

(B)  SRQ

(C)  RST

(D)  STU

Answer: (D)

24. If 20 + 15 = 24 and 64 + 13 = 42, then 11 + 28 = ?

(A)  33

(B)  36

(C)  31

(D)  34

Answer: (B)

25. In the diagram alongside, AD = 2AB = What is the area of the shaded portion ?

(A)  2d2

(B)  d/2

(C)  d2/2

(D)  d2/4

Answer: (D)

26. Find out the set among the four sets which is like the given set.

Given set : (9, 27, 108)

(A)  (15, 45, 90)

(B)  (8, 32, 96)

(C)  (5, 15, 75)

(D)  (11, 33, 132)

Answer: (D)

27. Number of letters skipped in between adjacent letters in the series is two. Which one of the following alternatives observes this rule?

(A)  UROLIF

(B)  WTQNKJ

(C)  SPMLI

(D)  TSPNKH

Answer: (A)

28. Choose the synonym for the given word from the given options.

‘Exposition’

(A)  Examination

(B)  Expression

(C)  Elucidation

(D)  Revelation

Answer: (D)

29. Some equations are solved on the basis of a certain principle. On the same basis, find the correct answer from among the alternatives given.

27(18)3; 216(180)6; 729(?)9

(A)  810

(B)  700

(C)  81

(D)  648

Answer: (C)

30. Karan facing towards South moved straight 2 km and from there, turned to his right 90° and travelled 2 km. Then, he took a 45° turn to his left and travelled 1 km. Where would he be now with respect to the starting point?

(A)  North West region

(B)  South West region

(C)  South region

(D)  South East region

Answer: (B)

31. Reena walks 4 km East and then cycles 10 km North, turns to the left and covers 4 km and once again turns to the North and moves 2 km. How far is she from the starting point?

(A)  10 km

(B)  14 km

(C)  8 km

(D)  12 km

Answer: (D)

32. Laxmi started walking Southwards and after covering a distance of 30 m, she turned left. She walked 25 meters, and turned to the left and again walked 30 meters. How far is she from the starting point and in which direction?

(A)  20 m West

(B)  25 m West

(C)  25 m East

(D)  30 m East

Answer: (C)

33. In a certain code, ‘CAPITAL’ is written as ‘CPATILA’. How is ‘PERSONS’ written in that code?

(A)  PSONRES

(B)  PONSRES

(C)  PESONRS

(D)  PREOSSN

Answer: (D)

34. If PAINT is coded as 74128 and EXCEL is coded as 93596, then how would you encode ACCEPT?

(A)  455978

(B)  735961

(C)  554978

(D)  547978

Answer: (A)

35. In the following problem :-

= Stand for ÷

+ stands for –

× stands for =

− stands for >

> stands for +

< stands for ×

÷ stands for <

When these new symbols are substituted, only one will be correct. Identify the correct equation-

(A)  24 = 4 > 5 × 17 + 13

(B)  24 = 4 + 2 × 17 + 13

(C)  24 = 4 > 5 × 17 > 13

(D)  24 + 4 = 5 × 17 + 13

Answer: (B)

Directions-(Q. 36) Two statements are followed by two assumptions I and II. You have to consider the statements to be true, even if they seem to be at variance from commonly known facts. You have to decide which of the given conclusions, if any, definitely follows from the given statements.

36. Statements:

(1) Parental role in the education of their children is very significant.

(2) All parents must realize this.

Conclusions:

(I) Parentless children cannot develop to their maximum potential.

(II) Parents can help their children in their education.

(A)  Neither conclusion I nor II follows

(B)  Both conclusions I and II follow

(C)  Only conclusion I follows

(D)  Only conclusion II follows

Answer: (D)

37. Find out the number which does not belong to the group of ABCD-

Answer: (D)

Directions-(Q. 38 to 40) Select the missing number from the given responses-

38.

5            7          4          39

6             9          5          59

7             11        ?          83

(A)  6

(B)  7

(C)  4

(D)  8

Answer: (A)

39.

8             16        10

2             8          5

4             ?          9

8             5          9

(A)  2

(B)  5

(C)  4

(D)  3

Answer: (B)

40.

8             (14)      15

13           (18)      22

25           (?)        41

(A)  33

(B)  42

(C)  14

(D)  36

Answer: (D)

Directions0(Q. 41) Which answer figure will complete the question figure?

41. 

Answer: (A)

42. Which of the answer figures is exactly the mirror image of the given figures, when the mirror is held as shown?

Answer: (A)

43. A word is represented by only one set of numbers is given in any one of the alternatives. The sets of numbers given in the alternatives are represented by two classes of alphabets as in the 2 matrices given below. The columns and rows of matrix I are numbered from 0 to 4 and that of matrix II from 5 to 9. A letter from these matrices can be represented first by its row and next column number. e.g., ‘F’ can be represented by 14, 21 etc. ‘S’ can be represented by 58, 96 etc. similarly, identify the word TRIP.

(A)  78, 76, 21, 76

(B)  59, 57, 41, 56

(C)  85, 88, 33, 89

(D)  66, 69, 40, 69

Answer: (B)

44. Select the answer figure in which the question figure is hidden-

Answer: (A)

45. A triangular sheet of paper has been folded and punched as shown in the figure. You have to figure out from amongst the four response figures, how it will appear when opened?

Answer: (C)

46. If a person has intelligence, character and position, which figure represents all of these?

Answer: (A)

47. How many squares are there in a given figure?

(A)  12

(B)  13

(C)  10

(D)  11

Answer: (A)

48. Which figure represents the relationship among Hockey players, Rubber balls, Poets?

(A)  1

(B)  4

(C)  2

(D)  5

Answer: (D)

49. In the question, one part of the problem figure is subtracted. Select the option that shows the correct shape after subtraction-

Answer: (A)

50. Identify the answer figure from which the pieces given in question figure have been cut-

Answer: (B)

Part-B

General Knowledge & General Awareness

51. There is no selling cost under-

(A)  Oligopoly

(B)  Duopoly

(C)  Perfect competition

(D)  Monopolistic competition

Answer: (C)

52. Which of the following is not a function of the Exim Bank of India?

(A)  Financing of joint venture in foreign countries

(B)  Loans to Indian parties for contributing to share capital of joint ventures abroad

(C)  Financing of export and import of goods and services

(D)  Inspection of exported goods for quality assurance

Answer: (D)

53. Which of the following systems in independent India goes against the very basis of democracy?

(A)  Party system

(B)  Parliamentary system

(C)  Caste system

(D)  Economic system

Answer: (C)

54. According to Karl Marx, the change in economic system results in inevitable changes in-

(A)  the entire systems

(B)  religious system only

(C)  political system only

(D)  social system only

Answer: (D)

55. Which one of the following statements is correct?

(A)  The value of commodity depends upon its price.

(B)  The value of commodity is entirely dependent upon the substitutes.

(C)  A commodity will have value if it is wanted by somebody

(D)  A commodity will have value only if it is scarce relative to demand.

Answer: (C)

56. Which of the following formulae could be used for calculating the per capital income of a country?

(A) 

(B) 

(C) 

(D) 

Answer: (D)

57. For controlling inflation, the Central Bank should-

(A)  purchase Government securities in the open market

(B)  lower the reserve ratio of the banks

(C)  sell Government securities in the open market

(D)  lower the bank rate

Answer: (C)

58. Match the medieval travellers with their countries:

List-I

(a) Marco Polo

(b) Ibnbattuta

(c) Antonio Monserrate

(d) Mahmud Wali Balkhi

List-II

(1) Spain

(2) Balkh

(3) Italy

(4) Morocco

(A)  (a)-3; (b)-4; (c)-1; (d)-2

(B)  (a)-1; (b)-3; (c)-2; (d)-4

(C)  (a)-4; (b)-3; (c)-1; (d)-2

(D)  (a)-3; (b)-1; (c)-4; (d)-2

Answer: (A)

59. Rana Kumbha constructed the Tower of Victory at Chittor to mark his victory over-

(A)  The Khan of Nagaur

(B)  Rao Jodha of Marwar

(C)  Ahmad Shah of Gujarat

(D)  Mahmud Khalji of Malwa

Answer: (D)

60. Match the following

List-I

(a) Sarojini Naidu

(b) M.A. Jinnah

(c) Sir Tej Bahadur Sapru

(d) V.D. Savarkar

List-II

(1) Muslim League

(2) Indian National Congress

(3) Hindu Mahasabha

(4) Liberal Party

(A)  (a)-2; (b)-4; (c)-1; (d)-3

(B)  (a)-4; (b)-1; (c)-3; (d)-2

(C)  (a)-2; (b)-1; (c)-4; (d)-3

(D)  (a)-2; (b)-1; (c)-3; (d)-4

Answer: (C)

61. The Social Forestry Scheme was introduced during-

(A)  Second Five Year Plan

(B)  Sixth Five Year Plan

(C)  Fourth Five Year Plan

(D)  Eighth Five Year Plan

Answer: (D)

62. The model code of conduct for political parties and candidates to be followed during the election is-

(A)  enjoined by the Supreme Court

(B)  agreed through a voluntary agreement among the recognized political parties

(C)  laid down in the Constitution of India

(D)  specified in The Representation of the People Act. 1951

Answer: (D)

63. A citizen can directly move the Supreme Court for any violation of Fundamental Rights under-

(A)  Article 33

(B)  Article 34

(C)  Article 31

(D)  Article 32

Answer: (D)

64. Which one of the following sitting Vice-Presidents of India contested for the post of President and lost the election?

(A)  S. Radhakrishnan

(B)  V. V. Giri

(C)  Bhairon Singh Shekhawat

(D)  Both (B) and (C)

Answer: (C)

65. The French challenges to British in India came to an end with-

(A)  Battle of Plassey

(B)  Battle of Buxar

(C)  Battle of Wandiwash

(D)  Battle of Srirangapattnam

Answer: (C)

66. Identify the medical trio of ancient India from the following names-

(A)  Charaka, Susruta and Bharata

(B)  Charaka, Susruta and Patanjali

(C)  Charaka Susruta and Vagbhata

(D)  Charaka, Vatsyayana and Vagbhata

Answer: (C)

67. Vegetation is effective in absorbing-

(A)  High frequency sound

(B)  Pollutant metals

(C)  Pollutant gases

(D)  Polluted water

Answer: (C)

68. When a person cries, there is a watery discharge from the nose due to activation of-

(A)  Thyroid gland

(B)  Endocrine gland

(C)  Salivary gland

(D)  Lachrymal gland

Answer: (B)

69. The fundamental role of root hairs in plants is-

(A)  to absorb water and mineral salts from the soil

(B)  to bind soil particles to the root for firm fixation of the plant

(C)  to protect the  young root from damage by coarse soil particles

(D)  to protect the root from soil microbes

Answer: (A)

70. Catch crops are-

(A)  crops to be cut and fed green to the cattle

(B)  substitute crops planted after the regular crop has failed

(C)  crops planted to attract certain insect pests to be destroyed

(D)  crops planted to attract certain useful insects to be used for biological control of pests

Answer: (B)

71. Which one of the following acts as a mediator between the user’s program and the hardware-

(A)  Operating System

(B)  Browser

(C)  Compiler

(D)  Editor

Answer: (A)

72. The project similar to T.V.A. (Tennessee Valley Authority) of U.S.A. in India is-

(A)  Ramganga Multipurpose Project

(B)  Idukki Project

(C)  Damodar Valley Project

(D)  Mahanadi Delta Project

Answer: (C)

73. Petrology is the study of-

(A)  earth

(B)  minerals

(C)  rocks

(D)  soils

Answer: (C)

74. Match the Following:

List-I                                          List-II

(a) Rosewood                             1. Mangrove forests

(b) Bushes                                  2. Alpine Forests

(c) Birch                                     3. Deciduous forests

(d) Sundari tree                         4. Dry forests

(A)  (a)-1; (b)-2; (c)-3; (d)-4

(B)  (a)-2; (b)-3; (c)-4; (d)-1

(C)  (a)-4; (b)-3; (c)-2; (d)-1

(D)  (a)-3; (b)-4; (c)-2; (d)-1

Answer: (D)

75. During the period of South-West monsoon, Tamil Nadu remains dry because-

(A)  it lies in the rain-shadow area

(B)  the temperature is too  high to let the winds cool down

(C)  the winds do not reach this area

(D)  there are no mountains in this area

Answer: (C)

76. The red colour of ripe tomatoes is due to the presence of-

(A)  Chlorophyll

(B)  Carotenoids

(C)  Hormones

(D)  Vitamins

Answer: (B)

77. Anticoagulants are not present in-

(A)  Leech

(B)  Wasp

(C)  Mosquito

(D)  Bed bug

Answer: (B)

78. To eliminate the glare of headlights in motor cars-

(A)  thin films are used

(B)  filters are used

(C)  polaroids are used

(D)  glass prisms are used

Answer: (C)

79. The substances which have infinite electrical resistance are called-

(A)  conductors

(B)  resistors

(C)  insulators

(D)  condensers

Answer: (C)

80. The headquarters of International Atomic Energy Agency is located at-

(A)  Vienna

(B)  Sydney

(C)  Geneva

(D)  Washington

Answer: (A)

81. If you use a font that is not supported by a browser, then the text-

(A)  will be displayed using ‘ARIAL’ font only

(B)  will be displayed with a particular background

(C)  will be displayed in the default font

(D)  will not be displayed

Answer: (C)

82. The pigment that protects plants from UV damage is-

(A)  Phycocyanin

(B)  Carotenoids

(C)  Chlorophyll

(D)  Xanthophyll

Answer: (D)

83. United Nations Conference on Environment and Development is called-

(A)  Air Summit

(B)  Resource Summit

(C)  Earth Summit

(D)  Water Summit

Answer: (C)

84. A bio-energy source obtained by fermentation to supplement fossil fuel petrol is-

(A)  Diesel

(B)  Methane

(C)  Kerosene

(D)  Ethanol

Answer: (D)

85. The substance that causes the worst air pollution is-

(A)  Carbon dioxide

(B)  Carbon monoxide

(C)  Smoke

(D)  Sulphur dioxide

Answer: (C)

86. A liquid is aid to boil when its-

(A)  vapor pressure is greater than the surrounding pressure

(B)  vapour pressure is less than the surrounding pressure

(C)  vapour pressure equal the surrounding pressure

(D)  vapour pressure vanishes to zero

Answer: (C)

87. Which is not correct regarding covalent compounds?

(A)  Compounds are usually liquids and gases

(B)  Boiling points and melting points are low

(C)  The reaction is slow

(D)  The reaction is fast

Answer: (D)

88. Choose the correct statement-

(A)  Mixtures are homogeneous

(B)  In a mixture the components are present in a fixed ratio

(C)  The components of a mixture cannot be separated

(D)  Chemical properties of a mixture are the same as those of its components

Answer: (D)

89. PVC is obtained from-

(A)  Styrene

(B)  Acetylene

(C)  Propene

(D)  Polyvinyl Chloride

Answer: (D)

90. What is the name of the first research station established by India in the Antarctica?

(A)  Agnihotri

(B)  Aryabhatta

(C)  Dakshin Gangotri

(D)  Maitri

Answer: (C)

91. Who among the following was awarded Padma Vibhushan in 2011 for his exceptional and distinguished service in the field of trade and industry?

(A)  S. Gopalakrishnan

(B)  Mecca Rafeeque Ahmed

(C)  Azim Premji

(D)  Ratan  Tata

Answer: (C)

92. The five permanent members of the U.N. Security Council are-

(A)  China, Germany, Russia, U.K. and U.S.A.

(B)  China, Germany, U.S.A. U.K. and Canada

(C)  China, France, Russia, U.K. and U.S.A.

(D)  China, Canada, France, U.S.A. and Germany

Answer: (C)

93. Which of the following is not correctly paired?

(A)  Harbhajan Singh – Kabaddi

(B)  Saina Nehwal – Badminton

(C)  Jwala Gutta – Tennis

(D)  Virat Kohli – Cricket

Answer: (A)

94. Which of the following States has the largest gap in the male and female literacy as per 2011 Census (provisional data)?

(A)  Rajasthan

(B)  Kerala

(C)  Uttar Pradesh

(D)  Madhya Pradesh

Answer: (A)

95. Stainless steel is an alloy of-

(A)  iron, carbon and zinc

(B)  iron, zinc and manganese

(C)  iron, chromium and nickel

(D)  iron, chromium and carbon

Answer: (D)

96. Enriched uranium used in a nuclear reactor is-

(A)  uranium alloyed with aluminium

(B)  uranium with a high percentage of a particular isotope

(C)  uranium free of all impurities

(D)  235U

Answer: (D)

97. In a refrigerator, cooling is produced by-

(A)  the sudden expansion of a compressed gas

(B)  the evaporation of a volatile liquid

(C)  the ice which deposite in the freezer

(D)  None of these

Answer: (B)

98. The first woman to conquer Mount Everest twice is-

(A)  Santosh Yadav

(B)  Suma Shirur

(C)  Surja Lata Devi

(D)  Jyoti Randhawa

Answer: (A)

99. Which one of the following iron and steel works in India is not under public sector?

(A)  Bhilai

(B)  Durgapur

(C)  Bokaro

(D)  Jamshedpur

Answer: (D)

100. Which Indian hockey player has a road named after him in Germany?

(A)  Roop Singh

(B)  Dhanraj Pillai

(C)  Dhyan Chand

(D)  Zafar Iqbal

Answer: (C)

Part-C

Quantitative Aptitude

101. In a fort, there was sufficient food for 200 soldiers for 31 days. After 27 days, 120 soldiers left the fort. For how many extra days will the rest of the food last for the remaining soldiers?

(A)  4 days

(B)  12 days

(C)  10 days

(D)  6 days

Answer: (D)

102. Equal amounts of water were poured into two empty jars of different capacities, which made one jar 1/4 full and the other jar 1/3 full. If the water in the jar with lesser capacity is then poured into the jar with greater capacity, then the part of the larger jar filled with water is-

(A)  1/4

(B)  1/3

(C)  1/2

(D)  7/12

Answer: (C)

103. A bookseller makes 8% profit after selling the book at 10% discount. The ratio of the cost price to the marked price is-

(A)  5 : 6

(B)  6 : 5

(C)  4 : 5

(D)  5 : 4

Answer: (A)

104. A certain sum of money is distributed to A and B in the ratio 2:5. If A received Rs 100, then the money received by B is-

(A)  Rs 250

(B)  Rs 300

(C)  Rs 200

(D)  Rs 150

Answer: (A)

105. Last year my age was a perfect square number. Next year it will be a cubic number. What is my present age?

(A)  26 years

(B)  24 years

(C)  25 years

(D)  27 years

Answer: (A)

106. What is the value of 

(A)  92.51

(B)  0.9251

(C)  0.9261

(D)  92.61

Answer: (C)

107. The greatest number that can divide 140, 176, 264 leaving remainders of 4, 6 and 9 respectively is-

(A)  17

(B)  2

(C)  85

(D)  34

Answer: (A)

108. There are 4 terms in an A.P. such that the sum of two means is 110 and product of their extremes is 2125. The 3rd term is-

(A)  55

(B)  45

(C)  65

(D)  75

Answer: (C)

109. The value of  is-

(A)  106

(B)  136

(C)  116

(D)  126

Answer: (D)

110. If 10 men or 18 boys can do a work in 15 days, then the number of days required by 15 men and 33 boys to do twice the work is-

(A)  9

(B)  36

(C) 

(D)  8

Answer: (A)

111. A man walks a certain distance and rides back taking a total time of 37 minutes. He could walk both ways in 55 minutes. How long would he take to ride both ways?

(A)  19 minutes

(B)  20 minutes

(C)  9.5 minutes

(D)  18 minutes

Answer: (A)

112. Mahesh starts work as a sales representative on an annual salary of Rs 1,60,000. If he receives a 15% pay-rise each year, the number of years he has worked for the company, ,when his annual salary became Rs 2,79,841 is-

(A)  4

(B)  5

(C)  2

(D)  3

Answer: (A)

113. The base of a triangle is 2 cm more than twice its altitude. If the area is 12 sq. cm, its altitude will be-

(A)  4 cm

(B)  3 cm

(C)  6 cm

(D)  5 cm

Answer: (B)

114. A man leaves Rs 12,600 to be divided among 7 sons, 3 daughters and 5 nephews. If each daughter receives there times as much as each nephew and each son seven times as much as each nephew, then each daughter’s share is-

(A)  Rs 600

(B)  Rs 750

(C)  Rs 700

(D)  Rs 650

Answer: (A)

115. The average of three numbers 70, *7 and 5* is 57. If * represents the same digit, then it must be-

(A)  4

(B)  7

(C)  3

(D)  6

Answer: (A)

116. Three years ago, the average of a family of 8 members was 30 years. If one child is also included in the family, the present average age of the family remained the same. Then the present age of the child is-

(A)  6 years

(B)  1 year

(C)  3 years

(D)  4 years

Answer: (A)

117. By selling an article for Rs 21,000, a man gains 5%. To get a profit of 15%, he has to sell it for-

(A)  Rs 23,000

(B)  Rs 25,000

(C)  Rs 19,800

(D)  Rs 20,700

Answer: (A)

118. Rahul bought two cycles for a total sum of Rs 1,500. He sold one cycle at 20% loss and the other cycle at 20% gain. If the selling price of both the cycles is the same, find the cost piece of the two cycles.

(A)  Rs 750 each

(B)  Rs 550, Rs 950

(C)  Rs 500, Rs 1,000

(D)  Rs 600, Rs 900

Answer: (D)

119. The salary of an employee increases every year in the month of July by 10%. If his salary in May 2000 was Rs 15,000, his salary in October 2001 was-

(A)  Rs 18,150

(B)  Rs 19,965

(C)  Rs 16,500

(D)  Rs 18,000

Answer: (A)

120. 72% of the students of a certain class took Biology and 44% took Mathematics. If each student took Biology or Mathematics and 40 took both, the total number of students in the class was-

(A)  250

(B)  320

(C)  200

(D)  230

Answer: (A)

121. If the volume and the surface area of a sphere are numerically equal, then the numerical value of the radius of the sphere is-

(A)  3

(B)  4

(C)  1

(D)  2

Answer: (A)

122. If   then the value of   is-

(A)  5

(B)  10

(C)  15

(D)  3

Answer: (A)

123. Minimum value of  is-

(A)  0

(B)  −1

(C)  −3

(D)  −2

Answer: (C)

124. If a + b = 5, a2 + b2 = 13, the value of a – b (where a > b) is-

(A)  1

(B)  −2

(C)  2

(D)  −1

Answer: (A)

125. If (3x – y) : (x + 5y) = 5 : 7, then the value of (x + y) : (x – y) is-

(A)  2 : 3

(B)  3 : 2

(C)  3 : 1

(D)  1 : 3

Answer: (C)

126. The line passing through the points (−2, 8) and (5, 7)-

(A)  Cuts y-axis only

(B)  Cuts both the axes

(C)  Does not cut any axes

(D)  Cut x-axis only

Answer: (B)

127. In ∆ ABC, ∠B = 60°, ∠C = 40°, If AD bisects ∠BAC and AE ⊥ BC, then ∠EAD is-

(A)  10°

(B)  20°

(C)  40°

(D)  80°

Answer: (A)

128. ABCD is a quadrilateral in which diagonal BD = 64 cm, AL ⊥ BD and CM ⊥ BD, such that AL = 13.2 cm and CM = 16.8 cm. The area of the quadrilateral ABCD in square centimeter is-

(A)  422.4

(B)  690.0

(C)  537.6

(D)  960.0

Answer: (D)

129. The area of a semi-circular field is 308 sq. m; taken taking π = 22/7, the length of the railing to surround it has to be-

(A)  88 m

(B)  80 m

(C)  44 m

(D)  72 m

Answer: (D)

130. Volume of a right circular cone is numerically equal to its slant surface area. Then value of  where h and r are height and radius of the cone respectively, is-

(A)  4 units

(B)  1/4 unit

(C)  9 units

(D)  1/9 unit

Answer: (D)

131. If the numerical value of volume of a right circular cylinder and its curved surface area are equal, then its radius is-

(A)  3 units

(B)  6 units

(C)  2 units

(D)  4 units

Answer: (C)

132. ABCDEF is a regular hexagon of side 2 feet. The area, in square feet, of the rectangle BCEF is-

(A)  8

(B)  4 + 4√3

(C)  4

(D)  4√3

Answer: (D)

133. If in ∆ ABC, ∠A = 90°, BC = a, AC = b and AB = c, then the value of tan B + tan C is-

(A)  a2/ab

(B) 

(C)  b2/ac

(D)  a2/bc

Answer: (D)

134. A ladder is resting against a wall at a height of 10 m. If the ladder is inclined with the ground at an angle of 30°, then the distance of the foot of the ladder from the wall is-

(A)  10/√3 m

(B)  20/√3 m

(C)  10√3 m

(D)  20√3 m

Answer: (C)

135. tan 7° tan 23° tan 60° tan 67° tan 83° is equal to-

(A)  0

(B)  √3

(C)  1/√3

(D)  1

Answer: (B)

136. The value of (sec θ – cos θ) (tan θ + cot θ) is-

(A)  1

(B)  3/2

(C)  2

(D)  0

Answer: (A)

137. In the figure below, if AB || CD and CE ⊥ ED, then the value of x is-

(A)  53

(B)  63

(C)  37

(D)  45

Answer: (A)

138. PA and PB are two tangents drawn from an external point P to a circle with centre O where the points A and B are the points of contact. The quadrilateral OAPB must be-

(A)  A rectangle

(B)  A rhombus

(C)  A square

(D)  Cyclic

Answer: (D)

139. G is the centroid of ∆ If AG = BC, then ∠BGC is-

(A)  90°

(B)  30°

(C)  60°

(D)  120°

Answer: (A)

140. In the following figure, if OA = 10 and AC = 16, then OB must be-

(A)  5

(B)  6

(C)  3

(D)  4

Answer: (B)

141. The graph f y = x + |x| is given by-

(A)  3

(B)  4

(C)  1

(D)  2

Answer: (C)

142. The equation of this graph is-

(A)  y = x

(B)  y = 3x

(C)  y = −x

(D)  y = −3x

Answer: (B)

143. If tan(θ1 + θ2) = √3, and sec(θ1 – θ2) = 2/√3, then the value of sin 2θ1 + tan 3θ2 is equal to-

(assume that 0 < θ1 – θ2 < θ1 + θ2 < 90°)

(A)  0

(B)  3

(C)  1

(D)  2

Answer: (D)

144. The area of the shaded region in the following graph is-

(A)  6 units

(B)  8 units

(C)  2 units

(D)  4 units

Answer: (D)

145. The shaded region represents-

(A)  y ≥ x

(B)  y ≤ −x

(C)  y ≤ x

(D)  y ≥ −x

Answer: (A)

Directions-(Q. 146-150) The pie-chart given below shows the distribution of workforce by occupational category for country X in 1981 and 1995. Study the chart and answer the questions.

146. The increase in the number of Clerical workers in the workforce of country X from 1981 to 1995 (in millions) is-

(A)  0.5

(B)  1.25

(C)  0.75

(D)  1.5

Answer: (D)

147. The percentage decrease in the number of Blue-Collar workers in the workforce of country X from 1981 to 1995 is-

(A)  20

(B) 

(C) 

(D)  35

Answer: (B)

148. In 1981, the number of Service workers in the workforce, in millions, was-

(A)  22.5

(B)  28.0

(C)  15.0

(D)  20.5

Answer: (A)

  1. In 1981, the number of categories which comprised of more than 25 million workers each, is-

(A)  Four

(B)  Five

(C)  Two

(D)  Three

Answer: (D)

150. The ratio of the number of workers in the Professional category in 1981 to the number of such workers in 1995 is-

(A)  9 : 14

(B)  14 : 9

(C)  4 : 9

(D)  5 : 14

Answer: (A)

Part-D

English Comprehension

   Directions-(Q. 151-155) Some parts of the sentence have errors and some have none, Find out which part of a sentence has an error and blacken the rectangle corresponding to the appropriate letter (A, B, C). If there is no error, blacken the rectangle corresponding to letter (D) in the Answer Sheet.

151. The concentration of human and material resources at a few centres (A)/ have resulted in (B)/ large scale migration of rural population to urban areas. (C)/ No Error (D)

Answer: (B)

152. Who wouldn’t be triumphant (A)/ in their success (B)/ at the examination ? (C)/ No Error (D)

Answer: (B)

153. I dared (A)/ Mohan fights (B)/ a duel. (C)/ No Error (D)

Answer: (B)

154. All of you except Sheena (A)/ have done the homework (B)/ properly. (C)/ No Error (D)

Answer: (D)

155. The summit began (A)/ on a much voiced (B)/ optimism note. (C)/ No Error (D)

Answer: (B)

Directions-(Q. 156 to 160) Sentences are given with blanks to be filled in with an appropriate word(s). Four alternatives are suggested for each question. Choose the correct alternative out of the four and indicate it by blackening the appropriate rectangle in the Answer Sheet.

156. Many modern methods of agriculture ……… thousands of years ago.

(A)  sprang

(B)  started

(C)  began

(D)  originated

Answer: (C)

157. He is utterly ……… at the end of the day.

(A)  worn away

(B)  worn on

(C)  worn through

(D)  worn out

Answer: (D)

158. People should refrain ………. Evil deeds.

(A)  from doing

(B)  doing

(C)  to do

(D)  in doing

Answer: (A)

159. A rare audio tape released by the court at the end of the hearing showed many judges asking ………… they should intervene at all.

(A)  in only

(B)  only if

(C)  weather

(D)  whether

Answer: (D)

160. Rita ……….. her home work yet.

(A)  has not done

(B)  will not do

(C)  does not do

(D)  did not do

Answer: (A)

Directions(Q. 161 to 165) Out of the four alternatives, choose the one which BEST EXPRESSES the MEANING of the given bold word and mark it in the Answer Sheet.

161. Clandestinely

(A)  secretly

(B)  openly

(C)  financially

(D)  systematically

Answer: (A)

162. Periphery

(A)  circle

(B)  boundary

(C)  edge

(D)  terminus

Answer: (B)

163. Exonerate

(A)  absolve

(B)  execute

(C)  negate

(D)  charge

Answer: (A)

164. Exquisite

(A)  exact

(B)  wonderful

(C)  quizzical

(D)  exciting

Answer: (B)

165. Comprehensive

(A)  adhesive

(B)  expensive

(C)  alternative

(D)  inclusive

Answer: (D)

Directions-(Q. 166-170) Choose the word OPPOSITE in MEANING to the given bold word and mark it in the Answer Sheet.

166. Precedence

(A)  subordination

(B)  seclusion

(C)  succession

(D)  supersession

Answer: (C)

167. Misdemeanour

(A)  adventure

(B)  cowardice

(C)  humility

(D)  courtesy

Answer: (D)

168. Nibble

(A)  swallow

(B)  munch

(C)  devour

(D)  consume

Answer: (A)

169. Jeopardy

(A)  pleasure

(B)  stability

(C)  safety

(D)  comfort

Answer: (C)

170. Hindrance

(A)  assistance

(B)  authorization

(C)  approval

(D)  approbation

Answer: (A)

Directions-(Q. 171-175) Four alternative are given for the idiom/phrase underlined in the sentence. Choose the alternative which best EXPRESSES the MEANING of the idiom/phrase and mark it in the Answer Sheet.

171. Suman held court with the studio audience.

(A)  tried to please to gain support

(B)  entertained with interesting funny stories

(C)  took to the court of law

(D)  had a formal discussion with

Answer: (B)

172. Ravi wants to blaze a trail in his activities.

(A)  be supported

(B)  be the centre of attraction

(C)  initiate work and be a pioneer

(D)  blow the trumpet

Answer: (C)

173. Their friendship lasted without a cloud to the end of his life.

(A)  without concealment

(B)  without stigma

(C)  without trouble

(D)  without romance

Answer: (B)

174. She took a wrong decision by not marrying this gentleman. She doesn’t know that beauty is only skin deep.

(A)  physical charm is important

(B)  internal beauty is not important

(C)  physical touch is not important

(D)  physical beauty is not important

Answer: (D)

175. The officials played ducks and drakes with the public money.

(A)  spent

(B)  preserved

(C)  collected

(D)  squandered

Answer: (D)

Directions-(Q. 176-180) A part of the sentence is underlined. Below are given alternatives to the underlined part at (A), (B) and (C) which may improve the sentence. Choose the correct alternative. In case no improvement is needed,  your answer is letter (D)

176. He won’t hear you provided you shout.

(A)  in case

(B)  as long as

(C)  unless

(D)  No improvement

Answer: (C)

177. The ship is at the anchor and the sailors are not at ease.

(A)  at an anchor

(B)  at its anchor

(C)  at anchor

(D)  No improvement

Answer: (B)

178. The objected to the plan on the grounds that it was too expensive.

(A)  declined

(B)  disagreed

(C)  refused

(D)  No improvement

Answer: (D)

179. Very little rice was found in the pot after all the guests had eaten.

(A)  remained

(B)  was cooked

(C)  stored

(D)  No improvement

Answer: (A)

180. The policeman captured the first car that approached and ordered the driver to take the injured child to the hospital.

(A)  commandeered

(B)  interrupted

(C)  caught

(D)  No improvement

Answer: (C)

Directions-(Q. 181-185) Out of the four alternatives, choose the one which can be substituted for the given words/sentence.

181. A study of words and the way words develop-

(A)  Phraseology

(B)  Phonography

(C)  Philology

(D)  Phonology

Answer: (A)

182. Something which is not in harmony-

(A)  Cacophonous

(B)  Noisy

(C)  Smooth

(D)  Quiet

Answer: (A)

183. A list of books-

(A)  Catalogue

(B)  Index

(C)  Anthology

(D)  Glossary

Answer: (A)

184. Man with womanly habits and manners-

(A)  Effeminate

(B)  Gay

(C)  Bovine

(D)  Suave

Answer: (A)

185. Something unusual or strange, having peculiar notions-

(A)  Whimsical

(B)  Aggressive

(C)  Dramatic

(D)  Reasonable

Answer: (C)

Directions-(Q. 186-190) There are four different words out of which one is correctly spelt. Find the correctly spelt word and indicate it by blackening the appropriate rectangle in the Answer Sheet.

186. 

(A)  flabbergast

(B)  flabbergust

(C)  flabergast

(D)  flabergust

Answer: (A)

187.

(A)  holocost

(B)  halocaust

(C)  holocaust

(D)  holocast

Answer: (C)

188.

(A)  revarential

(B)  reverentiel

(C)  reverantial

(D)  reverential

Answer: (D)

189.

(A)  metikyulus

(B)  meticulus

(C)  meteculous

(D)  meticulous

Answer: (D)

190.

(A)  suvenir

(B)  souviener

(C)  souvenir

(D)  suvenire

Answer: (C)

Directions-(Q. 191-200) In the following passage, some of the words have been left out. First read the passage over try to understand what it is about. Then fill in the blanks with the help of the alternatives given. Blacken the appropriate rectangle in the Answer Sheet.

   The League of Nations was (191) in 1919. It became ineffective and the UNO began to (192) from October 24th, 1945. The Second World War (193) in 1939s. It caused great (194) to human lives and property. As a (195) of use of atom bombs, many people were (196) and many others became (197). The world leaders feared that another world war would (198) the entire world. It was a question of the (199) of mankind. To ensure peace and (200) the world leaders established a world organization, the UNO.

191. 

(A)  made

(B)  opened

(C)  created

(D)  formed

Answer: (D)

192.

(A)  function

(B)  commence

(C)  start

(D)  operate

Answer: (A)

193.

(A)  broke into

(B)  broke up

(C)  broke in

(D)  broke out

Answer: (A)

194.

(A)  loss

(B)  disturbance

(C)  wastage

(D)  calamity

Answer: (A)

195.

(A)  reason

(B)  matter

(C)  result

(D)  cause

Answer: (C)

196.

(A)  mutilated

(B)  disabled

(C)  destroyed

(D)  killed

Answer: (D)

197.

(A)  unfit

(B)  handicapped

(C)  disabled

(D)  disadvantaged

Answer: (C)

198.

(A)  jeopardize

(B)  destroy

(C)  perish

(D)  demolish

Answer: (A)

199.

(A)  presence

(B)  existence

(C)  continuity

(D)  survival

Answer: (B)

200.

(A)  safety

(B)  order

(C)  tranquility

(D)  security

Answer: (D)

SSC CAPFs-Delhi Sub-Inspectors Examination-2013 Paper-I Question Paper With Answer Key

SSC CAPFs-Delhi Sub-Inspectors Examination-2013 Paper-I
SSC CAPFs-Delhi Sub-Inspectors Examination-2013 Paper-I Question Paper With Answer Key

SSC CAPFs/Delhi Sub-Inspectors Examination-2013 Paper-I

Solved Papers

Part-A

General Intelligence & Reasoning

   Directions-(Q. 1 to 6) Select the related word/letter/number from the given alternatives.

1. ADHM : ZUQN : : GIKM : ?

(A)  NOQS

(B)  TQRN

(C)  TRPN

(D)  TNRP

Answer: (C)

2. 7 : 133 : : 9 : ?

(A)  147

(B)  99

(C)  171

(D)  158

Answer: (C)

3. 36 : 216 : 81 : ?

(A)  729

(B)  629

(C)  319

(D)  826

Answer: (A)

4. Victory : Joy : : ? : Sorrow

(A)  Defeat

(B)  Depression

(C)  Melancholy

(D)  Cry

Answer: (A)

5. Body : Stomach : : Library : ?

(A)  Cash

(B)  Book

(C)  Headmaster

(D)  School

Answer: (B)

6. Hand : JBPE : : PALM : ?

(A)  RBNM

(B)  RBMN

(C)  QBNN

(D)  RBNN

Answer: (D)

Directions-(Q. 7 to 12) Select the one which is different from the other three responses.

7.

(A)  Teaching

(B)  Counselling

(C)  Instruction

(D)  Guidancce

Answer: (A)

8.

(A)  (25, 49)

(B)  (121, 169)

(C)  (7, 169)

(D)  (9, 25)

Answer: (C)

9.

(A)  HEAT

(B)  MEAT

(C)  MEET

(D)  BEAT

Answer: (C)

10.

(A)  8465

(B)  2643

(C)  4867

(D)  6243

Answer: (C)

11.

(A)  FhjL

(B)  PrtV

(C)  KnpR

(D)  CegI

Answer: (C)

12.

(A)  Sitar

(B)  Violin

(C)  Tanpura

(D)  Veena

Answer: (C)

13. Arrange the following words as per order in the dictionary?

(1) Forge            (2) Forget

(3) Forgo            (4) Forgive

(5) Format

(A)  (5), (2), (4), (3), (1)

(B)  (1), (4), (3), (2), (5)

(C)  (3), (4), (5), (2), (1)

(D)  (1), (2), (4), (3), (5)

Answer: (D)

14. Which one of the given responses would be a meaningful order of the following words?

(1) Police

(2) Punishment

(3) Crime

(4) Judge

(5) Judgement

(A)  (1), (2), (3), (4), (5)

(B)  (5), (4), (3), (2), (1)

(C)  (3), (1), (4), (5), (2)

(D)  (3), (1), (2), (4), (5)

Answer: (C)

Directions-(Q. 15 to 18) A series is given with one term missing. Choose the correct alternative from the given ones that will complete the series.

15. KJL, ONP, SRT, ….?…..

(A)  WVX

(B)  VWX

(C)  WXV

(D)  VUW

Answer: (A)

16. 198, 202, 211, 227 …?….

(A)  236

(B)  252

(C)  275

(D)  245

Answer: (B)

17. AN, BO, CP, DQ, ….?…

(A)  EG

(B)  ER

(C)  EH

(D)  EF

Answer: (B)

18. 7, 25, 61, 121 …?…

(A)  210

(B)  211

(C)  212

(D)  209

Answer: (B)

19. A group of alphabets are given with each being assigned a number. These have to be unscrambled into a meaningful word and correct order of letters may be indicated from the given responses.

(A)  (5), (4), (6), (1), (2), (3)

(B)  (6), (5), (4), (2), (3), (1)

(C)  (2), (3), (4), (5), (6), (1)

(D)  (5), (6), (4), (3), (1), (2)

Answer: (D)

20. Which one of the following diagrams best depicts the relationship among Mammals, Cows and Crows ?

Answer: (C)

21. Which of the following figure best represents the relationship amongst the Human being Girl, Boy?

Answer: (B)

22. In a row at a bus stop, ‘A’ is 7th from the left and ‘B’ is 9th from the right. They both interchange their positions. Now A becomes 11th from the left. How many people are there in the row?

(A)  10

(B)  20

(C)  19

(D)  18

Answer: (C)

23. At present, the ratio between the ages of Arun and Deepak is 4 : 3. After 6 years, Arun’s age will be 20 years. What is the age of Deepak at present?

(A)  15 years

(B)  19 years

(C)  24 years

(D)  12 years

Answer: (A)

24. From the given alternatives select the word which cannot be formed using the letters of the given word-

Encouragement

(A)  game

(B)  tear

(C)  neck

(D)  meat

Answer: (C)

25. If DELHI is coded as 73541 and CALCUTTA as 82589662, then how could CALICUT be coded in that code?

(A)  5978213

(B)  8251896

(C)  8543691

(D)  5279431

Answer: (B)

26. If ‘JUNE’ is written as ‘PQRS’ and ‘AUGUST’ is written as ‘WQFQMN’. How can ‘GUEST’ be written in this same coding language?

(A)  5978213

(B)  8251896

(C)  8543691

(D)  5279431

Answer: (B)

27. In the given figure, circles represents students studying three different subjects. How many students study all the three subjects?

(A)  2

(B)  3

(C)  4

(D)  1

Answer: (B)

28. Arun said, “This girl is the wife of the grandson of my mother.” Who is Arun to the girl?

(A)  Grandfather

(B)  Husband

(C)  Father-in-law

(D)  Father

Answer: (C)

Directions-(Q. 29 and 30) Some equations are solved on the basis of a certain system. On the same basis, find out the correct answer for the unsolved equation.

29. If 235 = 38 and 452 = 45, then 345 = ?

(A)  49

(B)  66

(C)  72

(D)  50

Answer: (D)

30. 2 × 3 = 49, 5 × 6 = 2536, 1 × 9 = 181, 4 × 7 = ?

(A)  1628

(B)  1649

(C)  2549

(D)  1219

Answer: (B)

31. If ‘×’ means ‘+’, ‘÷’ means ‘−’, ‘+’ means ‘÷’ and ‘−’ means ‘×’ then what should be the value of the given equation?

14 × 4 ÷ 70 + 10 – 2 = ?

(A)  33

(B)  15

(C)  30

(D)  4

Answer: (D)

32. Select the correct combination of mathematical signs to replace * signs and to balance the given equation.

5 * 5 * 5 * 3 * 10

(A)  × + = ×

(B)  + − × =

(C)  + ÷ = ×

(D)  + ÷ × =

Answer: (A)

Directions-(Q. 33 and 34) Select the missing number from the given responses.

33.

(A)  330

(B)  336

(C)  428

(D)  420

Answer: (B)

34.

12            15        16

03           04        05

04           06        04

40           66        ?

(A)  104

(B)  320

(C)  25

(D)  84

Answer: (D)

35. Seema walks 30 m North. Then she turns right and walks 30 m then she turns right and walks 55 m. Then she turns left and walks 20 m. Then she again turns left and walks 25 m. How many matres away is she from her original position?

(A)  45 m

(B)  50 m

(C)  60 m

(D)  55 m

Answer: (B)

36. Four different positions of dice are as shown below. What number is opposite to face 3?

(A)  4

(B)  3

(C)  2

(D)  6

Answer: (A)

37. After walking 10 m, Shankar turned left and covered a distance of 6 m, then turned right and covered a distance of 20 m. In the end, he was moving towards the south. From which direction did Shankar start his journey?

(A)  West

(B)  North

(C)  South

(D)  East

Answer: (C)

38. Six persons are sitting in a circle. A is facing B, B is to the right of E and left of C. C is to the left of D. F is to the right of A. Now D exchanges his seat with F and E with B. Who will be sitting to the left of D?

(A)  D

(B)  E

(C)  A

(D)  B

Answer: (C)

Directions-(Q. 39 and 40) One/two statements are given followed by two/four conclusions I, II, III and IV. You have to consider the statements to be true even if they seem to be at variance form commonly known facts. You have to decide which of the given conclusions, if any, follow from the given statements.

39. Statement : Songs always have singers to sing them.

Conclusions : I. Singers make a song.

(II) There is no un-sun song.

(A)  Only conclusion II follows

(B)  Both conclusions I and II follow

(C)  Neither conclusion I nor II follows

(D)  Only conclusion I follows

Answer: (C)

40. Statement : Some cats are dogs.

(II) No dog is a Toy.

Conclusions : I. Some dogs are cats.

(II) Some Toys are cats.

(III) Some cats are not Toys.

(IV) All Toys are cats.

(A)  Only Conclusions I and III follow

(B)  Only Conclusions II and III follow

(C)  Only Conclusion I and II follow

(D)  Only Conclusion I follows

Answer: (D)

Directions-(Q. 41 and 42) Which answer figure will complete the question figure?

41.

Answer: (A)

42.

Answer: (C)

Directions-(Q. 43 and 44) Select the answer figure is which the question figure is hidden/embedded.

43.

Answer: (A)

44.

Answer: (D)

Directions-(Q. 45 and 46) A piece of paper is folded and cut as shown below in the question figures. From the given answer figures, indicate how it will appear when opened?

45.

Answer: (B)

46.

Answer: (C)

Directions-(Q. 47 and 48) If a mirror is placed on the line AB, then which of the answer figures is the right image of the given figure?

47.

Answer: (A)

48.

Answer: (B)

49. A word is represented by only one set of numbers as given in any one of the alternatives. The sets of numbers given in the alternatives are represented by two classes of alphabets as in two matrices given below. The columns and rows of Matrix-I are numbered from 0 to 4 and that of Matrix-II are numbered from 5 to 9. A letter from these matrices can be represented first by its row and next by its column, e.g., ‘C’ can be represented by 00, 12, 23, etc. and ‘M’ can be represented by 56, 67, 77, etc. Similarly, you have to identify the set for the given word-GOD.

(A)  10, 11, 65

(B)  95, 79, 12

(C)  30, 65, 40

(D)  00, 10, 75

Answer: (C)

50. A word is represented by only one set of numbers are given in any one of the alternatives. The sets of numbers given in the alternatives are represented by two classes of alphabets as in two matrices given below. The columns and rows of Matrix-I are numbered from 0 to 4 and that of Matrix-II are numbered from 5 to 9. A letter from these matrices can be represented first by its row and next by its column, e.g., ‘B’ can be represented by 01, 10, 22, etc. and ‘F’ can be represented by 55, 76, 86, etc. Similarly, you have to identify the set for the given word-CAGE.

(A)  95, 82, 31, 14

(B)  20, 00, 65, 40

(C)  14, 20, 41, 86

(D)  00, 21, 41, 95

Answer: (B)

Part-B

General Knowledge & General Awareness

51. If a firm is operating at loss in the short-period in perfect competition, it should-

(A)  decrease the production and the price

(B)  increase the production and the price

(C)  continue to operate as long as it covers even the variable costs

(D)  Shut-down and leave the industry

Answer: (A)

52. For which one of the following commissions, there is no provision in the Indian Constitution ?

(A)  Election Commission

(B)  Finance Commission

(C)  Planning Commission

(D)  Union Public Service Commission

Answer: (C)

53. Which one of the following statements about PYROLYSIS, which is a process for solid waste treatment is incorrect?

(A)  It converts the waste into solid, liquid and gas of which the resultant liquid and gas can be used to produce energy

(B)  The process occurs at a temperature above 430°C at atmospheric pressure

(C)  The process occurs  under high pressure at temperature above 430°C

(D)  It is a thermo-chemical decomposition of organic waste

Answer: (C)

54. Which Sultan received a robe of honour from the caliph?

(A)  Ala-ud-din Khilji

(B)  Iltutmish

(C)  Balban

(D)  Qutub-ud-din Aibak

Answer: (B)

55. Sugatha Kumari chosen for the award of the Saraswati Samman 2012 is an eminent poetess in-

(A)  Kannada

(B)  Tamil

(C)  Telugu

(D)  Malayalam

Answer: (D)

56. Which one of the following is not a line of demarcation between two countries?

(A)  International Date Line

(B)  MacMahon Line

(C)  Radcliffe Line

(D)  Durand Line

Answer: (A)

57. Which of the following supports particle nature of photons?

(A)  Diffraction

(B)  Polarization

(C)  Photoelectric effect

(D)  Interference

Answer: (C)

58. Which country won the Sultan Azlan Shah Hockey tournament in Malaysia in March 2013?

(A)  Malaysia

(B)  Newzealand

(C)  India

(D)  Australia

Answer: (D)

59. The heaviest naturally occurring element is-

(A)  Mercury

(B)  Polonium

(C)  Thorium

(D)  Uranium

Answer: (D)

60. Which of the following is not an operating system?

(A)  Multi user-Single tasking

(B)  Single user-Single tasking

(C)  Single user-Multitasking

(D)  Multi user-Multitasking

Answer: (C)

61. Which one of the following language is not specified in the English schedule of the Indian Constitution?

(A)  Urdu

(B)  English

(C)  Sindhi

(D)  Sanskrit

Answer: (B)

62. Haptens are-

(A)  Pseudoantigens

(B)  Incomplete antigens

(C)  Antibodies

(D)  Isoantigens

Answer: (B)

63. Which state in India occupies the same ranking position in respect of area and population?

(A)  Manipur

(B)  Meghalaya

(C)  Nagaland

(D)  Gujarat

Answer: (*)

64. Sulphur dioxide bleaches colouring matter by-

(A)  Reduction

(B)  Dehydration

(C)  Decomposition

(D)  Oxidation

Answer: (D)

65. Which of the following elements is not radioactive?

(A)  Radium

(B)  Plutonium

(C)  Zirconium

(D)  Uranium

Answer: (C)

66. Wilting of plants occurs due to excessive-

(A)  Respiration

(B)  Guttation

(C)  Absorption

(D)  Transpiration

Answer: (D)

67. Gypsum is used for improvement of-

(A)  Alkaline soils

(B)  Saline soils

(C)  Podsols

(D)  Acidic soils

Answer: (D)

68. Which of the following is an indirect Tax?

(A)  Capital Gains Tax

(B)  Excise Duty

(C)  Wealth tax

(D)  Estate Duty

Answer: (B)

69. Indicate the correct arrangement for electromagnetic radiation in order of their increasing wavelength-

(A)  Microwave, infrared, visible, X-rays

(B)  X-rays, visible, infrared, microwave

(C)  Visible, infrared, microwave, X-rays

(D)  X-rays, infrared, visible, microwave

Answer: (B)

70. Where was the Fifth BRICS Summit hosted by South Africa in March 2013?

(A)  Durban

(B)  Johannesburg

(C)  Pretoria

(D)  Cape Town

Answer: (A)

71. Lake formed in a cut off river meander is called-

(A)  Playa Lake

(B)  Meteoric Lake

(C)  Ox-Bow Lake

(D)  Crater Lake

Answer: (C)

72. The rapidly growing mass of phytoplankton covering the surface water of a lake or pond is known as-

(A)  Eutrophication

(B)  Water bloom

(C)  Water pollution

(D)  Water hyacinth

Answer: (A)

73. Which is the leading state in India in ‘Ship Breaking’ industry?

(A)  Tamil Nadu

(B)  Maharashtra

(C)  Gujarat

(D)  West Bengal

Answer: (C)

74. The telephone line is connected through the ……….. on the computer.

(A)  USB

(B)  Modem

(C)  Ethernet

(D)  PS2

Answer: (B)

75. In the nuclear reactors, moderators are used to-

(A)  Accelerate neutrons

(B)  Absorb the neutrons

(C)  Slow down the neutrons

(D)  Generate neutrons

Answer: (C)

76. When Margaret Thatcher was P.M. with which country Britain waged a war to regain control of Falkland Islands?

(A)  Chile

(B)  Argentina

(C)  Brazil

(D)  None of these

Answer: (B)

77. Match the following-

(a) Chalukyas     1. Mlakhed

(b) Hoysalas       2. Vatapi

(c) Rashtrakutas 3. Warangal

(d) Kakatiyas      4. Dwarasamudra

(A)  (a)-2; (b)-4; (c)-1; (d)-3

(B)  (a)-4; (b)-3; (c)-1; (d)-2

(C)  (a)-1; (b)-2; (c)-3; (d)-4

(D)  (a)-3; (b)-2; (c)-4; (d)-1

Answer: (A)

78. Wings of birds are-

(A)  Modified hind limbs

(B)  New structure

(C)  Integumentary outgrowth

(D)  Modified for limbs

Answer: (D)

79. The world’s tallest statue of Mahatma Gandhi is in ……….. .

(A)  Champaran

(B)  Patna

(C)  Lucknow

(D)  Rajkot

Answer: (B)

80. Which one of the following is the principal source of information on Asoka’s campaign against Kalinga?

(A)  Pillar Edict VII

(B)  Mahavamsa

(C)  Divyavadana

(D)  Rock Edit XIII

Answer: (D)

81. Gandhi’s concept of Trusteeship-

(A)  Recognises right of private ownership of property

(B)  Transforms the capitalistic society into an egalitarian one

(C)  Excludes legislative regulation of the ownership and use of wealth

(D)  Does not fix minimum or maximum income

Answer: (B)

82. Colortrum is very good as it is rich in protein and-

(A)  Carbohydrates

(B)  Anti bodies

(C)  Minerals

(D)  Vitamins

Answer: (B)

83. Among the following states, which one has adopted the ‘Neem’ tree as the state tree?

(A)  Maharashtra

(B)  Tamil Nadu

(C)  Kerala

(D)  Andhra Pradesh

Answer: (D)

84. Contous line is the imaginary line joining places of equal-

(A)  Rainfall

(B)  Humidity

(C)  Elevation

(D)  Temperature

Answer: (C)

85. Who is the Chairperson of the National Green Tribunal ?

(A)  Justice J.S. Verma

(B)  Justice Swatantra Kumar

(C)  Justice K.G. Balakrishnan

(D)  Justice Markandey Singh

Answer: (B)

86. Which is the first state in India to enact the Food Security Act?

(A)  Chhattisgarh

(B)  Gujarat

(C)  Tamil Nadu

(D)  Bihar

Answer: (A)

87. Which article of the Indian Constitution provides for equal opportunities for all citizen in Public employment?

(A)  Article-22

(B)  Article-16

(C)  Article-20

(D)  Article-25

Answer: (B)

88. Sandstone is metamorphosed to-

(A)  Shale

(B)  Slate

(C)  Quartzite

(D)  Marble

Answer: (C)

89. ‘Taxes on professions’ can be levied by-

(A)  State government only

(B)  both by state and union government

(C)  by panchayats only

(D)  Union government only

Answer: (A)

90. Which one of the following is opposite to democratic state?

(A)  Despotism

(B)  Republic

(C)  Socialism

(D)  Monarchy

Answer: (A)

91. Which one is not a function of money?

(A)  Transfer of value

(B)  Store of value

(C)  Price stabilisation

(D)  Value measurement

Answer: (C)

92. Instrument used to study the behaviour of a vibrating string is-

(A)  Barometer

(B)  Hydrometer

(C)  Hygrometer

(D)  Sonometer

Answer: (D)

93. Match the following-

(a) Hunter’s Commission                    1. 1948

(b) Wardha Scheme                              2. 1904

(c) University’s Act                               3. 1937

(d) Radhakrishnan Commission       4. 1882

(A)  (a)-3; (b)-2; (c)-4; (d)-1

(B)  (a)-4; (b)-2; (c)-3; (d)-1

(C)  (a)-3; (b)-4; (c)-1; (d)-2

(D)  (a)-4; (b)-3; (c)-2; (d)-1

Answer: (D)

94. The casual organism of Polio is-

(A)  A fungi

(B)  A virus

(C)  A worm

(D)  A bacteria

Answer: (B)

95. Panda belongs to the same family as that-

(A)  Kangaroo

(B)  Porcupine

(C)  Whale

(D)  Bear

Answer: (D)

96. ‘Closed Economy’ means-

(A)  no provision for public sector

(B)  no provision for private sector

(C)  economic policy not well defined

(D)  a country having no imports and exports

Answer: (D)

97. The Gandhara are flourished under-

(A)  The Kushans

(B)  The Satavahanas

(C)  The Guptas

(D)  The Mauryas

Answer: (A)

98. At ‘Break-even point’-

(A)  the industry is in equilibrium in the long-run

(B)  the producers suffers the minimum losses

(C)  the seller earn maximum profit

(D)  the firm is at zero-profit point

Answer: (D)

99. Who said : “Where there is no law, there is not freedom”?

(A)  Bentham

(B)  Lenin

(C)  Marx

(D)  Locke

Answer: (D)

100. Which one of the following statements about the Chief Justice of India (CJI) is not correct?

(A)  He appoints the Chief Justice of all High Courts

(B)  The CJI administers the oath of office to the President

(C)  When both the offices of the President and Vice-President fall vacant simultaneously, the CJI discharges the duties of the President

(D)  The CJI can hold his office till he attains the age of 65 years.

Answer: (A)

Part-C

Quantitative Aptitude

101. The ratio of two numbers is 3 : 4 and their HCF is 5. Then LCM is-

(A)  20

(B)  60

(C)  15

(D)  12

Answer: (B)

102. The value of tan 1° tan 2° tan 3° …… tan 89° is-

(A)  1

(B)  2

(C)  Undefined

(D)  0

Answer: (A)

103. A man sold 250 chairs and had a gain equal to selling price of 50 chairs. His profit per cent is-

(A)  20%

(B)  25%

(C)  50%

(D)  15%

Answer: (B)

104. A can do as much work as B and C together can do. A and B can together do a piece of work in 9 hours 36 minutes and C can do it in 48 hours. The time (in hours) that B needs to do the work alone, is-

(A)  18

(B)  24

(C)  30

(D)  12

Answer: (B)

105. A solid right circular cylinder and a solid hemisphere stands on equal bases and have the same height. The ratio of their whole surface areas is-

(A)  3 : 2

(B)  3 : 4

(C)  4 : 3

(D)  2 : 3

Answer: (C)

106. If area of an equilateral triangle is a and height b, then value of b2/a is-

(A)  3

(B)  1/3

(C)  √3

(D)  1/√3

Answer: (C)

107. The batting average for 30 innings of a cricket player is 40 runs. His highest score exceeds his lowest score by 100 runs. If these two innings are not included, the average of the remaining 28 innings is 38 runs. The lowest score of the player is-

(A)  15

(B)  18

(C)  20

(D)  12

Answer: (B)

108. An article was sold at 16% gain. Had it been sold for Rs 200 more, the gain would have been 20%. Then the cost price of the article is-

(A)  Rs 5000

(B)  Rs 4800

(C)  Rs 4500

(D)  Rs 5200

Answer: (A)

109. From the top of a hill 200 m high, the angle of depression of the top and the bottom of a tower are observed to 30° and 60°. The height of the tower is (in m)-

(A) 

(B) 

(C) 

(D)  200√3

Answer: (C)

110. A copper sphere of diameter 18 cm is drawn into a wire of diameter 4 mm. The length of the wire, in metre is-

(A)  2.43

(B)  243

(C)  2430

(D)  24.3

Answer: (B)

111. Triangle PQR circumscribes a circle with centre O and radius r cm such that ∠PQR = 90°. If PQ = 3 cm, QR = 4 cm, then the value of r is-

(A)  2

(B)  1.5

(C)  2.5

(D)  1

Answer: (C)

112. The value of 

(A)  12/29

(B)  8/19

(C)  48/29

(D)  2/19

Answer: (C)

113. Water flows at the rate of 10 metres per minute from a cylindrical pipe 5 mm in diameter. How long it take to fill up a conical vessel whose diameter at the base is 30 cm and dept 24 cm?

(A)  28 minutes 48 seconds

(B)  51 minutes 12 seconds

(C)  51 minutes 24 seconds

(D)  28 minutes 36 seconds

Answer: (A)

114. Three circles of equal radius ‘a’ cm touch each other. The area of the shaded region is-

(A) 

(B) 

(C) 

(D) 

Answer: (D)

115. If x = a – b, y – b = c, z = c – a, then the numerical value of the algebraic expression x3+y3 + z3 – 3xyz will be-

(A)  a + b + c

(B)  0

(C)  4(a + b + c)

(D)  3abc

Answer: (B)

116. The linear equation such that each point on its graph has an ordinate four times its abscissa is-

(A)  y + 4x = 0

(B)  y = dx

(C)  x = 4y

(D)  x + 4y = 0

Answer: (B)

117. A godown is 15m long and 12 m broad. The sum of the areas of the floor and the ceiling is equal to the sum of areas of the four walls. The volume (in m3) of the godown is-

(A)  900

(B)  1200

(C)  1800

(D)  720

Answer: (B)

118. 3 men and 7 women can do a job in 5 days, while 4 men and 6 women can do it in 4 days. The number of days required for a group of 10 women working together, at the same rate as before, to finish the same job is-

(A)  30

(B)  36

(C)  40

(D)  20

Answer: (D)

119. The value of  is

(A)  4.8484

(B)  4.8694

(C)  4.8884

(D)  4.8234

Answer: (C)

120. A shopkeeper marks the selling price of h is goods in such a way that after giving a discount of 10%, he gains 17%. How much per cent above the cost price is the marked price?

(A)  36%

(B)  27%

(C)  30%

(D)  40%

Answer: (C)

121. One of the factors of the expression 4√3x2 + 5x − 2√3 is-

(A)  4x + √3

(B)  4x + 3

(C)  4x – 3

(D)  4x – √3

Answer: (D)

122. On a journey across Kolkata, a taxi averages 50 km per hour for 50% of the distance, 40 km per hour for 40% of it and 20 km per hour for the remaining. The average speed, in km/hour, for the whole journey is-

(A)  32

(B)  40

(C)  35

(D)  45

Answer: (B)

123. A man ordered 4 Paris of black socks and some pairs of brown socks. The price of a black socks is double that of a brown pair. While preparing the bill the clerk interchanged the number of black and brown pairs by mistake which increased the bill by 50%. The ratio of the number f black and brown pair of socks in the original order was-

(A)  2 : 1

(B)  1 : 4

(C)  1 : 2

(D)  4 : 1

Answer: (B)

124. In the following figure, AB be diameter of a circle whose centre is O. If ∠AOE = 150°, ∠DAO = 51°, then the measure of ∠CBE is-

(A)  115°

(B)  110°

(C)  105°

(D)  120°

Answer: (C)

125. If the square of the sum of two numbers is equal to 4 times of their product, then the ratio of these numbers is-

(A)  2 : 1

(B)  1 : 3

(C)  1 : 1

(D)  1 : 2

Answer: (C)

126. If a2 + b2 = 5ab, then the value of  is-

(A)  32

(B)  16

(C)  23

(D)  −23

Answer: (C)

127. In a big garden 60% of the trees are coconut trees, 25% of the number f coconut trees are mango trees and 20% of the number of mango trees are apple trees. If the number of apple trees are 1500, then the number of trees in the garden in-

(A)  48,000

(B)  50,000

(C)  51,000

(D)  45,000

Answer: (B)

128. A sum of money placed at compound interest doubles itself in 5 years. It will amount to eight times itself in-

(A)  15 years

(B)  12 years

(C)  10 years

(D)  20 years

Answer: (A)

129. Minimum value of 4tan2θ + 9cot2θ is-

(A)  12

(B)  1

(C)  6

(D)  13

Answer: (A)

130. If 50% of (P-Q) = 30% of (P + Q) and Q = x% of P, then the value of x is-

(A)  30

(B)  25

(C)  20

(D)  50

Answer: (B)

131. If sin θ – cos θ = 1/2, the value of sin θ + cos θ is-

(A)  −2

(B)  ±2

(C)  √7/2

(D)  2

Answer: (C)

132. If cosec θ – cot θ = 7/2, the value of cosec θ is

(A)  47/28

(B)  51/28

(C)  53/28

(D)  49/28

Answer: (C)

133. If  and  then the value of x3 + y3 is-

(A)  950

(B)  730

(C)  650

(D)  970

Answer: (D)

134. A single discount equivalent to the following three successive discounts of 30%, 20% and 10% is given by-

(A)  49.6%

(B)  50.60%

(C)  49.40%

(D)  50.40%

Answer: (A)

135. The greatest among the following numbers of 31/3, 21/2, 1, 616 is

(A)  21/2

(B)  1

(C)  61/6

(D)  31/3

Answer: (D)

136. The areas of two similar triangles ABC and DEF are 20 cm2 and 45 cm2 If AB = 5 cm, then DE is equal to-

(A)  6.5 cm

(B)  7.5 cm

(C)  8.5 cm

(D)  5.5 cm

Answer: (B)

137. In a triangle ABC, BC is produced to D so that CD = AC. If ∠BAD = 111° and ∠ACB = 80°, then the measure of ∠ABC is-

(A)  31°

(B)  33°

(C)  35°

(D)  29°

Answer: (D)

138. In ∆ABC, ∠A + ∠B = 145° and ∠C + 2∠B = 180°. State which one of the following relation is true-

(A)  CA = AB

(B)  CA < AB

(C)  BC > AB

(D)  CA > AB

Answer: (D)

139. If the volumes of two right circular cones are in the ratio 4 : 1 and their diameters are in the ratio 5 : 4, then the ratio of their heights is-

(A)  25 : 16

(B)  25 : 64

(C)  64 : 25

(D)  16 : 25

Answer: (C)

140. The proportion of acid and water in 3 samples is 2 : 1, 3 : 2 and 5 : 3. A mixture containing equal quantities of all 3 samples is made. The ratio of water and acid in the mixture is-

(A)  120 : 133

(B)  227 : 133

(C)  227 : 120

(D)  133 : 227

Answer: (B)

141. A train 270 metres long is running at a speed of 36 km per hour; then it will cross a bridge of length 180 metres in-

(A)  40 sec.

(B)  45 sec.

(C)  50 sec.

(D)  35 sec.

Answer: (B)

Directions-(Q. 142 and 145) The following table shows the productions of food-grain (In million tons) in a state for the period 1999-2000 to 2003-04. Read the table and answer the questions.

142. In 2002-03, the percentage increase in the production of barley as compared to the previous year was-

(A)  14.20

(B)  17.85

(C)  18.75

(D)  7.90

Answer: (C)

143. During the period 1999-2000 to 2003-2004 x per cent of the total production is production of wheat. The value of x is about-

(A)  12.6

(B)  37.4

(C)  37.8

(D)  20.2

Answer: (B)

144. In the year 2003-04, the increase in production was maximum over the previous year for-

(A)  Rice

(B)  Barley

(C)  Other cereals

(D)  Wheat

Answer: (C)

145. The difference of average production of rice and the average production of barley over the year is-

(A)  50

(B)  60

(C)  80

(D)  40

Answer: (D)

Directions-(Q. 146-150) Production of three different flavours soft drinks X, Y and Z over a period of six  years has been expressed in the following graph. Study the graph and answer the questions.

146. The approximate decline in the production of flavor Z in 2010 as compared to the production in 2008 is-

(A)  33%

(B)  22.5%

(C)  42%

(D)  25%

Answer: (A)

147. The average annual production was maximum in the given period for the flavor-

(A)  Y only

(B)  Z only

(C)  X and Z

(D)  X only

Answer: (A)

148. What per cent of the total production of flavor X in 2005 and 2006 combined is the total production of flavor Z in 2007 and 2008 combined?

(A)  102.25

(B)  115.57

(C)  133.33

(D)  96.67

Answer: (C)

149. The percentage of rise/fall in production from the previous year is maximum for the flavour Y in this year-

(A)  2007

(B)  2008

(C)  2009

(D)  2006

Answer: (A)

150. The difference (in lakh bottles) between the average production of flavour X in 2005, 2006, 2007 and the average production of flavour Y in 2008, 2009 and 2010 is-

(A)  2.4

(B)  0.5

(C)  1.5

(D)  5

Answer: (D)

Part-D

English Comprehension

   Directions-(Q. 151 to 155) Some parts of the sentences have errors and some are correct. Find our which part of a sentence has an error and blacken the oval corresponding to the appropriate letter (A), (B), (C). If a sentence is free from error, blacken the oval corresponding to (D) in the Answer Sheet.

151. When one hears of the incident (A)/ about the plane crash (B)/ he feels very sorry. (C)/ No error (D)

Answer: (C)

152. I went there (A)/ with a view to survey (B)/ the entire procedure. (C)/ No error (D)

Answer: (B)

153. It had laid (A)/ in the closet (B)/ for a week before we found it. (C)/ No error (D)

Answer: (A)

154. He was present (A)/ in the court (B)/ to give witness. (C)/ No error (D)

Answer: (C)

155. He laughed (A)/ on her (B)/ as she fell off the tree. (C)/ No error (D)

Answer: (B)

Directions-(Q. 156 to 160) Sentences are given with blanks to be filled in with an appropriate word(s). Four alternatives are suggested for each question. Choose the correct alternative out of the four and indicate it by blackening the appropriate oval in the Answer Sheet.

156. There are not solitary, free-living creatures; every form of life is ……….. other forms

(A)  dependent on

(B)  parallel to

(C)  overshadowed by

(D)  segregated from

Answer: (A)

157. I’ll take …….. now as I have anothers appointment somewhere else.

(A)  departure

(B)  my leave

(C)  permission

(D)  leave from work

Answer: (C)

158. A garden knife is …… used for pruning.

(A)  generally

(B)  compulsorily

(C)  systematically

(D)  daily

Answer: (A)

159. Serious threat to our ecology and environment can be ……….. with organic cultivation.

(A)  hastened

(B)  impeded

(C)  aggravated

(D)  combated

Answer: (D)

160. ‘My India’ by Corbett deals ………. the author’s familiarity with and loved of India.

(A)  in

(B)  of

(C)  at

(D)  with

Answer: (D)

Directions-(Q. 161 to 165) Out of the four alternatives, choose the one which best expresses the meaning of the given word and mark it in the Answer Sheet.

161. Abnormal

(A)  Unnatural

(B)  Aggressive

(C)  Unique

(D)  Informal

Answer: (A)

162. Venal

(A)  Corrupt

(B)  Comprehensible

(C)  Legible

(D)  Forgivable

Answer: (A)

163. Conjurer

(A)  Magician

(B)  Jester

(C)  Performer

(D)  Trickster

Answer: (A)

164. Invoice

(A)  Word

(B)  Sound

(C)  Statement

(D)  Lanaguage

Answer: (C)

165. Ameliorate

(A)  Improve

(B)  Degrade

(C)  Motivate

(D)  Agree

Answer: (A)

Directions-(Q. 166 to 170) Choose the word opposite in meaning to the given word and mark it in the Answer Sheet.

166. Debacle

(A)  Success

(B)  Response

(C)  Acceptance

(D)  Agreement

Answer: (A)

167. Abusive

(A)  Laudatory

(B)  Profuse

(C)  Effusive

(D)  Noble

Answer: (A)

168. Amorphous

(A)  Amoral

(B)  Definite

(C)  Perfect

(D)  Irregular

Answer: (B)

169. Unitary

(A)  Single

(B)  Triple

(C)  Multiple

(D)  Double

Answer: (C)

170. Adulteration

(A)  Purification

(B)  Normalization

(C)  Rejuvenation

(D)  Consternation

Answer: (A)

Directions-(Q. 171 to 175) Four alternatives are given for the Idiom/Phrase underlined in the sentence. Choose the alternative which best expresses the meaning of the Idiom/Phrase and mark it in the Answer Sheet.

171. I tried to feel his pulse on the issue, but in vain.

(A)  find his views

(B)  enlighten him

(C)  argue with him

(D)  guide him

Answer: (A)

172. For this act of a indifference he will be taken to task by the authority.

(A)  he will get on official reprimand from the authority

(B)  he will be rewarded by the authority

(C)  he will tender his resignation to the authority

(D)  he will be entrusted with an official job

Answer: (A)

173. You need to have something up your sleeve if the present plan does not work.

(A)  have some honest means

(B)  have some hidden sources of money

(C)  have a secret pocket in the sleeve

(D)  have an alternative plan

Answer: (D)

174. The new manager ruled the roost over every one.

(A)  exercised authority

(B)  rushed through work

(C)  got paid very handsomely

(D)  created good impression

Answer: (A)

175. Despite his initial arrogance he had to eat humble pie.

(A)  he had to yield under pressure

(B)  he maintained composure

(C)  he failed to protest eventually

(D)  he accepted the food offered

Answer: (A)

Directions-(Q. 176 to 180) A part of the sentence is underlined. Below are given alternatives to the underlined part at (A), (B) and (C) which may improve the sentence. Choose the correct alternative. In case no improvement is needed your answer is (D).

176. The gentry of the town was invited.

(A)  A invited

(B)  has been unvited

(C)  were invited

(D)  No improvement

Answer: (C)

177. After the written exam, you will also have an oral exam,

(A)  practical

(B)  viva voce

(C)  vocal

(D)  No improvement

Answer: (B)

178. The regular use of alcohol, only in small quantities, tends to cause mischief in many ways to various organs of the body.

(A)  though in small quantities

(B)  even in a little quantity

(C)  even in small quantities

(D)  No improvement

Answer: (C)

179. If you were the Prime Minister of India what steps would you have taken to end unemployment?

(A)  will you take

(B)  will you be taking

(C)  would be take

(D)  No improvement

Answer: (C)

180. A high school student is not even understanding the basics of Chemistry and Physics.

(A)  does not understand even

(B)  was not even understanding

(C)  has even not understood

(D)  No improvement

Answer: (A)

Directions-(Q. 181 to 185) Out of the four alternatives choose the one which can be substituted for the given words/sentence.

181. A person who attends to the diseases of the eye is an-

(A)  oculist

(B)  optimist

(C)  obsterician

(D)  optician

Answer: (A)

182. The study of worms and insects-

(A)  taxidermy

(B)  entomology

(C)  ornithology

(D)  paleontology

Answer: (B)

183. A person who devotes his/her life for the welfare of others-

(A)  altruist

(B)  hermit

(C)  volunteer

(D)  martyr

Answer: (A)

184. A person who shows off his learning-

(A)  pedant

(B)  educationist

(C)  exhibitor

(D)  researcher

Answer: (A)

185. Written law of a legislative body-

(A)  statute

(B)  stature

(C)  static

(D)  statue

Answer: (A)

Directions-(Q. 186 to 190) Groups of four words are given. In each group, one word is correctly spelt. Find the correctly spelt word and mark your answer in the Answer Sheet.

186.

(A)  rhythym

(B)  rhithim

(C)  rhythim

(D)  rhythm

Answer: (D)

187.

(A)  indeganeous

(B)  indigenous

(C)  indegenous

(D)  indigeneous

Answer: (B)

188.

(A)  saccarine

(B)  sacarine

(C)  sachharine

(D)  saccharine

Answer: (D)

189.

(A)  revolutionize

(B)  revoulutionize

(C)  revvolutinoize

(D)  revollutionize

Answer: (A)

190.

(A)  disentry

(B)  dysentry

(C)  diesentry

(D)  dysentery

Answer: (D)

Directions-(Q. 191 to 195) There are two passages with 5 questions in each passage. Read the passage carefully and choose the best answer to each question out of the four alternatives and mark it by blackening the appropriate oval in the Answer Sheet.

Passage-I

(Q. 191-195)

   It is not luck but labour that makes the luck, says an American author, is ever waiting for something to turn up; labour with keen eyes and strong will turns up something. Luck lies in bed and wishes the postman would bring him news of a legacy, labour turns out at six and with busy pen and ringing hammer lays the foundation of competence. Luck whines, labour watches. Luck relies upon chance, labour upon character. Luck slips downwards to selff0indulgence; labour strides upwards and aspires to independence. The conviction, therefore, is extending the diligence is the mother of food luck. In other words, a man’s success in life will be proportionate to his efforts, to his industry, to his attention to small things.

191. Which one of the following words in the passage indicates that the writer does not ultimately reject the element of luck?

(A)  ‘Luck whines’

(B)  ‘Diligence is the mother of good luck’

(C)  ‘Luck …… wishes the postman would bring him news’

(D)  ‘Luck …… is ever waiting’

Answer: (C)

192. Which pair of words means the same thing?

(A)  Labour and industry

(B)  Industry and legacy

(C)  Diligence and legacy

(D)  Legacy and labour

Answer: (A)

193. Which one of the following statements sums up the meaning of the Luck?

(A)  Luck waits and complains without working while labour achieves success although it complains

(B)  Luck is self indulgent but labour is selfless

(C)  Luck often end in defeat but labour produces luck

(D)  Luck waits without exertion, but labour exerts without waiting

Answer: (D)

194. “……. labour turns out at six and with busy pen and ringing hummer lays the foundation of competence”. What does this statement mean?

(A)  Labour lays the foundation of the building

(B)  The writer and the labourer are the true eyes of the society

(C)  There is no worker who works so hard as the labourer who begins his day at six in the morning

(D)  Hardwork of all kinds makes people different

Answer: (D)

195. Which one of the statement is true about the passage?

(A)  Success depends only on hardluck

(B)  Expectation of good luck always meets with disappointment

(C)  Success is exactly proportionate to hard work

(D)  Luck is necessary for success

Answer: (C)

Passage-II

(Q. 196-200)

Violence has played a great part in the world’s history. It is today playing an equally important part and probably it will continue to so for a considerable time. It is impossible to ignore the importance of violence in the past and present. To do so is to ignore life. Yet violence is undoubtedly bad and brings an unending trail of evil consequences with it. And worse even than violence are the motives of hatred, cruetly, revenge and punishment which very often accompany violence. Indeed, violence is bad, not intrinsically, but because of these motives that go with it. There can be violence without these motives, there can be violence for a good object as well as for an evil object. But it is extremely difficult to separate violence from these motives, and therefore, it is desirable to avoid violence as far as possible. In avoiding it, however one cannot accept a negative attitude of submitting to and far greater evils, Submission to violence or the avoidance of an unjust regime based on violence is the very deceptive of the spirit of non-violence. The non-violent method in order justify itself, must be dynamic and capable of changing such a regime of social order.

196. The word ‘dynamic’ in the concluding line of the passage means-

(A)  active

(B)  energetic

(C)  capable of change and progress

(D)  all of the above

Answer: (D)

197. Which of the following statements is incorrect?

(A)  Only violence can be used against violence

(B)  Violence is not inherently evil

(C)  Violence is a historically accepted fact

(D)  Violence cannot be ignored

Answer: (A)

198. ‘Violence without these motives’ is possible only in-

(A)  practice

(B)  reality

(C)  dream

(D)  theory

Answer: (A)

199. ‘Indeed, violence is bad, not intrinsically but because of these motives that go with it’. This suggests-

(A)  Violence is basically good

(B)  Violence is bad only when it is associated with certain motives

(C)  Violence is bad because the people who exercise it are bad

(D)  Violence is basically bad

Answer: (B)

200. Non-violence, according to the writer, means-

(A)  Violence without the evil motives

(B)  Giving in to the tyranny of the powerful

(C)  Accepting violence as a fact of life

(D)  None of the above

Answer: (A)

SSC CAPFs-Delhi Sub-Inspectors Examination-2014 Paper-II Question Paper With Answer Key

SSC CAPFs-Delhi Sub-Inspectors Examination-2014 Paper-II
SSC CAPFs-Delhi Sub-Inspectors Examination-2014 Paper-II Question Paper With Answer Key

SSC CAPFs/Delhi Sub-Inspectors Examination-2014 Paper-II

English Language and Comprehension

   Directions-(Q. 1-12) Out of the four alternatives, choose the one which can be substituted for the given words/sentence.

1. A woman whose husband is dead.

(A)  divorcee

(B)  fiancee

(C)  widower

(D)  widow

Answer: (D)

2. One who eats no animal flesh.

(A)  cannibal

(B)  pilgrim

(C)  vegetarian

(D)  pedestrian

Answer: (C)

3. Man who has more than one wife at a time.

(A)  celibate

(B)  bigamist

(C)  misogamist

(D)  polygamist

Answer: (D)

4. One who deals in flowers.

(A)  A drouer

(B)  A brazier

(C)  A florist

(D)  A fruiterer

Answer: (C)

5. House or shelter of a gipsy.

(A)  wigwam

(B)  chalet

(C)  caravan

(D)  igloo

Answer: (C)

6. A person who leaves one country to settle in another.

(A)  emigrant

(B)  traveller

(C)  tourist

(D)  globe-trotter

Answer: (A)

7. A partner in a crime.

(A)  friend

(B)  comrade

(C)  accomplice

(D)  companion

Answer: (C)

8. Relating to the countries of the west.

(A)  Oriental

(B)  Hellenistic

(C)  Occidental

(D)  Platonic

Answer: (C)

9. A person who always runs after women.

(A)  Philanthropist

(B)  Don Juan Casanova

(C)  Philanderer

(D)  Philatelist

Answer: (C)

10. A person employed to drive a private or hired car.

(A)  Chauffeur

(B)  Autoist

(C)  Cabbie

(D)  Automobilist

Answer: (A)

11. Sentimental longing for a period in the past.

(A)  Recollection

(B)  Nostalgia

(C)  Reminiscence

(D)  Wistfulness

Answer: (B)

12. A medicine that softens the bowels.

(A)  Antacid

(B)  Laxative

(C)  Remedy

(D)  Herb

Answer: (B)

Directions-(Q. 13-32) A sentence has been given in Active/Passive Voice. Out of the four alternatives suggested, select the one which best expresses the same sentence in Passive/Active Voice and mark you answer in the Answer Sheet.

13. Children make a noise.

(A)  A noise was being made by the children

(B)  Noise is made by children

(C)  A noise was made by the children

(D)  A noise is being made by the children

Answer: (B)

14. She taunted her husband with the memory of his early promise.

(A)  Her husband is taunted by her with the memory of his early promise

(B)  Her husband was taunted with the memory of his early promise

(C)  Her husband was taunted by her with the memory of his early promise

(D)  The husband was taunted by her with the memory of his early promise

Answer: (C)

15. He impresses his boss with his performance.

(A)  His boss is impressed by his performance

(B)  His performance impresses his boss

(C)  His boss is impressed at his performance

(D)  His boss is impressed in his performance

Answer: (A)

16. How as it managed by you?

(A)  How have you managed it?

(B)  Did you manage it?

(C)  How did you manage it?

(D)  Was it managed by you?

Answer: (C)

17. I was astonished at his behavior.

(A)  His behavior is astonishing to me

(B)  His behavior astonished me

(C)  His behavior astonishes me

(D)  His behavior had astonished me

Answer: (B)

18. All his friends laughed at him.

(A)  He was laughed at by all his friends

(B)  He was laughed by all his friends

(C)  He was being laughed by all his friends

(D)  He was being laughed at by all his friends

Answer: (A)

19. Every disaster affected victim has been compensated by the government.

(A)  The government has compensated every disaster affected victim

(B)  The government had compensated every disaster affected victim

(C)  The government has been compensating every disaster affected victim

(D)  The government have been compensating disaster affected victim

Answer: (A)

20. Indiscipline should not be encouraged.

(A)  We are not encouraging indiscipline

(B)  We have not encouraging indiscipline

(C)  We should not encourage indiscipline

(D)  We must not encouraged indiscipline

Answer: (C)

21. This ho use is used very rarely by us.

(A)  We are using this house very rarely

(B)  We use this house very rarely

(C)  We used this house very rarely

(D)  We uses this house very rarely

Answer: (B)

22. The box can contain no more.

(A)  No more this box can contain

(B)  No more could be contained in this box

(C)  No more can be contained in this box

(D)  Nothing can be contained in this box

Answer: (C)

23. They held a meeting and delivered speeches.

(A)  A meeting was held and delivered speeches

(B)  A meeting will be held and speeches delivered

(C)  A meeting is held and speeches delivered

(D)  A meeting was held and speeches delivered

Answer: (D)

24. It is time for food to be eaten.

(A)  It is time to eat our food

(B)  It is time for eating food

(C)  It is time to eat

(D)  Food is to be eaten now

Answer: (D)

25. The teacher may punish you.

(A)  You should be punished by the teacher

(B)  You shall have been punished by the teacher

(C)  You will be punished by the teacher

(D)  You may be punished by the teacher

Answer: (D)

26. None likes him.

(A)  He is liked by none

(B)  He was not liked by anyone

(C)  He has not been liked by anyone

(D)  He is not liked by anyone

Answer: (A)

27. You are requested to keep the book on the table.

(A)  You should keep the book on the table

(B)  You must keep the book on the table

(C)  Please kept the book on the table

(D)  Please keep the book on the table

Answer: (D)

28. He was being chased by the dogs.

(A)  The dogs are chasing him

(B)  The dogs chased him

(C)  The dogs chase him

(D)  The dogs were chasing him

Answer: (D)

29. She wrote a letter.

(A)  A letter is being written by her

(B)  A letter is written by her

(C)  A letter was written by her

(D)  A letter was being written by her

Answer: (C)

30. My mother mustn’t be disturbed.

(A)  You mustn’t disturb my mother

(B)  You shouldn’t disturb my mother

(C)  You must be disturbing my mother

(D)  You have disturbed my mother

Answer: (A)

31. His lordship soon found me to be unfit for the service.

(A)  I had been found by his lordship to be unfit for the service

(B)  I was soon found by his lordship to be unfit for the service

(C)  I was found by his lordship to be unfit for the service

(D)  I was soon found out by his lordship to be  unfit for the service

Answer: (B)

32. They have not been submitting their class work regularly.

(A)  The class work have not been submitted regularly by them

(B)  The class work has not been submitted regularly by them

(C)  The class work are not being submitted regularly

(D)  The class work is not being submitted regularly by them

Answer: (B)

Directions-(Q. 33-54) A part of the sentence is underlined. Below are given alternatives to the underlined part at (A), (B), (C) which may improve the sentence. Choose the correct alternative. In case no improvement is needed your answer in the Answer Sheet.

33. You need to read these kinds of books for the test.

(A)  these kind of book

(B)  this kind of book

(C)  this kind of a book

(D)  No improvement

Answer: (D)

34. Due to power grid collapse essential services like hospitals, the railways and water plants were perturbed.

(A)  was perplexed

(B)  were paralysed

(C)  were abandoned

(D)  No improvement

Answer: (B)

35. Satish told his mother that he had been reading for six hours.

(A)  since six hours

(B)  from six hours

(C)  till six hours

(D)  No improvement

Answer: (D)

36. No sooner had he agreed to join the job than he started to have doubts.

(A)  No sooner did he

(B)  No sooner than he

(C)  No sooner was he

(D)  No improvement

Answer: (D)

37. Laws were enacted to do away with social evils.

(A)  to eradicate

(B)  to stop

(C)  to prevent

(D)  No improvement

Answer: (C)

38. The woman is waiting to see you looks rather angry.

(A)  The women whose waiting to see you looks rather angry.

(B)  The women who is waiting see you looks rather angry.

(C)  The women who is waiting to see you looks rather angry.

(D)  No improvement

Answer: (C)

39. If you are a cricket fan, make sure you are witness the grand opening ceremony today.

(A)  you witnessed

(B)  you witnessing

(C)  you witness

(D)  No improvement

Answer: (C)

40. Gauri was for waiting for Hema and I.

(A)  Gauri was waiting for Hema and me.

(B)  Gauri were waiting for Hema and me.

(C)  Gauri were waiting for Hema and I.

(D)  No improvement

Answer: (A)

41. The doctor made no farther comment to justify his action.

(A)  The doctor made no farther commandments to justify his action.

(B)  The doctor made no farther comments to justify his action.

(C)  The doctor made no further commitments to justify his action.

(D)  No improvement

Answer: (B)

42. When the thief saw the police approaching he showed a clean pair at heels.

(A)  a clean pair of heels

(B)  a clean pair on heels

(C)  a clean pair down heels

(D)  No improvement

Answer: (A)

43. Not until did he receive her letter he fully realized her problem.

(A)  Not until received her letter did he fully realize

(B)  Not until had he received her letter that he

(C)  Not until he had received her letter that he

(D)  No improvement

Answer: (A)

44. I can’t think of anybody whom to invite.

(A)  anybody whom I should invite

(B)  anybody to invite

(C)  anybody who should be invited

(D)  No improvement

Answer: (A)

45. I hadn’t heard from him for nearly ten years in which time I had got married and had two children.

(A)  I hadn’t heard of him for nearly ten years, during which point I had got married and had two children.

(B)  I hadn’t heard of his for nearly ten years, in which point I had got married.

(C)  I hadn’t heard for him for nearly ten years, in which case I had got married.

(D)  No improvement

Answer: (D)

46. Our progress was slow because of having to search for them at frequent intervals.

(A)  at having

(B)  through having

(C)  in having

(D)  No improvement

Answer: (B)

47. One day a wonderful plate of gold fell into the courtyard of a temple from Heaven at Beneras.

(A)  One day at Beneras a wonderful plate of gold fell into the courtyard from Heaven of a temple.

(B)  One day fell into the courtyard of a temple at Beneras a wonderful plate of gold from Heaven.

(C)  One day a wonderful plate of gold fell from Heaven into the courtyard of a temple at Beneras.

(D)  No improvement

Answer: (C)

48. The man who approached me of dark complexion has disappeared.

(A)  The dark complexioned man who approached me has disappeared

(B)  Has disappeared who approached me of dark complexion.

(C)  Who of dark complexion approached me had disappeared

(D)  No improvement

Answer: (A)

49. He narrated what his brother had done in vivid detail.

(A)  He narrated in vivid detail what his brother had done.

(B)  In vivid detail, he narrated what his brother had done.

(C)  He narrated what his brother in vivid detail had done.

(D)  No improvement

Answer: (B)

50. He is looking for a flat for his son of 1200 sq. feet carpet area.

(A)  He is looking for a flat of 1200 sq. feet carpet area for his son.

(B)  He of 1200 sq. feet carpet area is looking for a flat for his son.

(C)  Of 1200 sq. feet carpet area he is looking for a flat for his son.

(D)  No improvement

Answer: (A)

51. He impressed with his words than with his acts rather.

(A)  He impressed with his words rather than with his acts.

(B)  He rather impressed with his words than with his acts.

(C)  Rather he impressed with his words than with his acts.

(D)  No improvement

Answer: (A)

52. John loves Mary so much that she can turn him round her finger.

(A)  turn a blind eye

(B)  turn him a cold shoulder

(C)  turn his head

(D)  No improvement

Answer: (D)

53. He has a good command over both English and French is known to all.

(A)  That he has a good command over both English and French is known to all.

(B)  That is known he has a good command over both English and French to all.

(C)  That a good command over both English and French he has is known to all.

(D)  No improvement

Answer: (A)

54. I want a nurse to look after my child of about fifty years.

(A)  I want a nurse to look of about fifty years after my child.

(B)  I want a nurse of about fifty years to look after my child.

(C)  To look after my child of about fifty years I want a nurse.

(D)  No improvement

Answer: (B)

Directions-(Q 55-81) A sentence has been given in Direct/Indirect. Out of the four alternatives suggested, select the one which best expresses the same sentence in Direct/Indirect and mark your answer in the Answer Sheet.

55. The project manager asked Vivek, “How much time will you take to finish this project?”

(A)  The project manager asked Vivek how much time would he take to finish that project.

(B)  The project manager enquired of Vivek how much time he would take to finish that project.

(C)  The project manager enquired of Vivek how much time he will be taking to finish that project.

(D)  The project manager asked Vivek how much time he would take to finish this project.

Answer: (B)

56. The ox asked the dog not to sit there.

(A)  The ox said to the dog. “Not to sit here.”

(B)  The ox said to the dog, “Do not sit here.”

(C)  The ox said to the dog, “Does not sit here.”

(D)  The ox said to the dog, “Did not sit here.”

Answer: (B)

57. He said, “I am writing a poem.”

(A)  He said that I am writing a poem.

(B)  He said that he is writing a poem.

(C)  He said that the poem is being written.

(D)  He said that he was writing a poem.

Answer: (D)

58. He said, “Asha washed the clothes.”

(A)  He said that Asha washed the clothes.

(B)  He said that clothes were washed by Asha.

(C)  He said that Asha had washed the clothes.

(D)  He said that Asha has washed the clothes.

Answer: (C)

59. Martha said, “I am going to Mexico next year.”

(A)  Martha said that she had been going to Mexico next year.

(B)  Martha said that she was going to Mexico next year.

(C)  Martha said that she had gone to Mexico next year.

(D)  Martha said that she will be going to Mexico next  year.

Answer: (B)

60. My mother said to me, “Wish you a happy and prosperous life !”

(A)  My mother wished me a happy and prosperous life.

(B)  My mother told me that I should lead a happy and prosperous life

(C)  My mother wished that I enjoy a happy and prosperous life.

(D)  My mother wished that I should be living a happy and prosperous life.

Answer: (A)

61. You said to me, “Why are you talking so much?”

(A)  You asked me why he was talking so much.

(B)  You asked me why I was talking so much.

(C)  You asked me why was I talking so much.

(D)  You asked me why I had been talking so much.

Answer: (B)

62. Sandy said, “I have already written a letter to my family.”

(A)  Sandy said that she had already written a letter to her family.

(B)  Sandy said that she wrote a letter to her family.

(C)  Sandy said that she was writing a letter to her family.

(D)  Sandy said that she will write a letter to her family.

Answer: (A)

63. You said, “They were busy the whole day.”

(A)  You said that they had busy the whole day.

(B)  You said that they were busy the whole day.

(C)  You said that they had been busy the whole day.

(D)  You said that they have been busy the whole day.

Answer: (C)

64. I said to her, “I will have written letters.”

(A)  I told her that she will have written letters.

(B)  I told her that she would have written letters.

(C)  I told her that I will have written letters.

(D)  I told her that I would have written letters.

Answer: (D)

65. I asked him, “Will you go to Agra tomorrow?”

(A)  I asked him whether I would go to Agra the next day.

(B)  I asked him whether he would go to Agra the next day.

(C)  I asked him whether you would go to Agra the next day.

(D)  I asked him whether he will go to Agra the next day.

Answer: (B)

66. I asked him where he would stay.

(A)  I said to him, “Where you will stay ?”

(B)  I asked him, “Where will you stay ?”

(C)  I said to him, “Where I will stay ?”

(D)  I said to him, “Where will I stay ?”

Answer: (B)

67. He said to his son, “Do not smoke.”

(A)  He advised his son do not smoke.

(B)  He advised his son to not smoke.

(C)  He advised his son not to smoke.

(D)  He advised his son that he should not smoke.

Answer: (C)

68. She said, “I will now take rest for some time.”

(A)  She said that I would now take rest for some time.

(B)  She said that she will then take rest for some time.

(C)  She said that she would then take rest for some time.

(D)  She said that he will then take rest for some time.

Answer: (C)

69. The receptionist said to Umesh, “Please be seated and wait for your tunr.”

(A)  The receptionist request Umesh that be seated and wait for his turn.

(B)  The receptionist told Umesh to please be seated and wait for his turn.

(C)  The receptionist requested Umesh that he should be seated and wait for his turn.

(D)  The receptionist requested Umesh to be seated and to wait for his turn.

Answer: (D)

70. He exclaimed that it was a very graceful animal.

(A)  He exclaimed, “What a graceful animal !”

(B)  He observed, “It is a very graceful animal !”

(C)  He said, “Alas ! What a graceful animal it is !”

(D)  He said, “It is indeed a graceful animal !”

Answer: (A)

71. Caroline said, “Will you come to my party on Saturday” ?

(A)  Caroline asked that I come to her party on Saturday.

(B)  Caroline told to come to her party on Saturday.

(C)  Caroline asked if I would come to her party on Saturday.

(D)  Caroline said I should come to her party on Saturday.

Answer: (C)

72. Babu said, “I’ve told my friends you’ll be here.”

(A)  Babu said that he had told his friends that I would be here.

(B)  Babu said that he has told his friends that I will be here.

(C)  Babu said that he told his friends that I will be there.

(D)  Babu said that he had told his friends that I would be there.

Answer: (D)

73. He said to me, “I will have been digging the ground for half an hour”.

(A)  He told me that he will have been digging the ground for half an hour.

(B)  He told me that I will have been digging the ground for half an hour.

(C)  He told me that he would have been digging the ground for half an hour.

(D)  He told me that I would have been digging the ground for half an hour.

Answer: (C)

74. Monica Cheng asked Roeun, “Are you going to visit Germany ?”

(A)  Monica Cheng asked Roeun if he is going to visit Germany.

(B)  Monica Cheng asked Roeun if he had visited Germany.

(C)  Monica Cheng asked Roeun if had gone to Germany.

(D)  Monica Cheng asked Roeun if he was going to visit Germany.

Answer: (D)

75. You said to me, “I am going to America”.

(A)  You told me that I was going to America.

(B)  You said me that you were going to America.

(C)  You told me that you were going to America.

(D)  You told me I was going to America.

Answer: (C)

76. You said to me, “She is my sister”.

(A)  You said to me that she was my sister.

(B)  You told me that she was your sister.

(C)  You said to me that I was her sister.

(D)  You said to me that she is my sister.

Answer: (B)

77. The Professor said, “Have you cleared your last semester papers ?”

(A)  The Professor asked whether I cleared my last semester papers.

(B)  The Professor asked if I had cleared my previous semester papers.

(C)  The Professor asked whether I have cleared my last semester papers.

(D)  The Professor asked if I have cleared my previous semester papers.

Answer: (B)

78. The traffic constable said to us, “Stop”.

(A)  The traffic constable ordered us to stop.

(B)  We were asked by the traffic constable to stop.

(C)  The traffic constable requested us to stop.

(D)  The traffic constable ordered that we were made to stop.

Answer: (A)

79. “I went for a long walk yesterday”, said Monikaviya.

(A)  Monikaviya said that I went for a long walk yesterday.

(B)  Monikaviya said that she had gone for a long walk the day before.

(C)  Monikaviya said that she had gone for a long walk yesterday.

(D)  Monikaviya said that she went for a long walk the day before.

Answer: (B)

80. “Alas ! How I have wasted my fortune !” said he.

(A)  He confessed with regret that he had been wasting his fortune.

(B)  He confessed with regret that he had been very extravagant.

(C)  He confessed with regret that I have wasted my fortune.

(D)  He confessed with regret that he had wasted his fortune.

Answer: (D)

81. Ronald said, “Don’t vote for Otis because he is irresponsible”.

(A)  Mr. Ronald said not vote for Otis because he was irresponsible

(B)  Mr. Ronald told us don’t vote for Otis because he isn’t irresponsible

(C)  Mr. Ronald told us not to vote for Otis because he lacked responsibility

(D)  Mr. Ronald told us not to vote for Otis because he had been irresponsible.

Answer: (C)

   Directions-(Q. 82-106) You have several passage where some of the words have been left out. Read the passages carefully and choose the correct answer to each blank out of the four alternatives and mark it in the Answer Sheet.

Passage-I

(Q. No. 82 to 91)

   It is difficult to believe that any man can be so spiritually dead as to have no …(82)… for his native country after travelling in …(83)… lands. But if such an …(84)… person does exist, take careful not of his career; and  you will find …(85)… he will never …(86)… poets to celebrate him in deathless song. He …(87)… be a man of …(88)… rank, of noble family and of …(89)… beyond the dream of avarice; but these great …(90)… will not save him from being forgotten. In spite of them all, he will win no …(91)… during his life time.

82. 

(A)  kindness

(B)  love

(C)  hatred

(D)  pity

Answer: (B)

83.

(A)  foreign

(B)  abroad

(C)  overseas

(D)  fine

Answer: (A)

84.

(A)  unmerciful

(B)  unsympathetic

(C)  unnatural

(D)  unpatriotic

Answer: (D)

85.

(A)  if

(B)  whether

(C)  that

(D)  since

Answer: (C)

86.

(A)  encourage

(B)  bribe

(C)  inspire

(D)  kindle

Answer: (C)

87.

(A)  may

(B)  can

(C)  should

(D)  will

Answer: (A)

88.

(A)  classic

(B)  high

(C)  first

(D)  smart

Answer: (B)

89.

(A)  riches

(B)  clothes

(C)  eatables

(D)  furniture

Answer: (A)

90.

(A)  benefits

(B)  donations

(C)  concessions

(D)  advantages

Answer: (D)

91.

(A)  position

(B)  job

(C)  fame

(D)  happiness

Answer: (C)

Passage-II

(Q. No. 92 to 101)

   Colour is …(92)… the soul of painting. Vishnudha-ramottara mentions four …(93)… colours, namely white, black, orange and orange; …(94)… five basic colouurs are …(95)… by other authorities. According to Bharata’s Natyashastra colour …(96)… realness. Specific colours are …(97)… for particular castes or classes of people or for the …(98)… of …(99)… characters. Colour is descriptive as well as …(100)… Human temperament is …(101)… by colour.

92.

(A)  valued as

(B)  truly

(C)  considered

(D)  really

Answer: (C)

93.

(A)  basic

(B)  important

(C)  ground

(D)  significant

Answer: (A)

94.

(A)  only

(B)  while

(C)  yet

(D)  when

Answer: (B)

95.

(A)  pointed

(B)  accepted

(C)  acceptable

(D)  mentioned

Answer: (D)

96.

(A)  means

(B)  shows

(C)  symbolises

(D)  represents

Answer: (D)

97.

(A)  utilised

(B)  meant

(C)  used

(D)  painted

Answer: (B)

98.

(A)  revelation

(B)  exposition

(C)  presentation

(D)  depiction

Answer: (D)

99.

(A)  coarse

(B)  bad

(C)  cruel

(D)  uncouth

Answer: (B)

100.

(A)  connotative

(B)  explanatory

(C)  expository

(D)  meaningful

Answer: (A)

101.

(A)  exposed

(B)  showed

(C)  reflected

(D)  transmitted

Answer: (C)

Passage-III

(Q. No. 102 to 106)

   Then think of other conveniences the city has to offer; the …(102)… buses to take  you from place to place; the bath-rooms fitted with gleaming …(103)… fittings; the shops and super markets where  you can buy all  your …(104)… desires and so many other things impossible to list. When  you live in the country long enough you are bound to miss all these …(105)… of …(106).. life.

102.

(A)  rickety

(B)  vintage

(C)  comfortable

(D)  uncomfortable

Answer: (C)

103.

(A)  pantry

(B)  ceramic

(C)  sanitary

(D)  plastic

Answer: (C)

104.

(A)  mind’s

(B)  heart’s

(C)  emotion’s

(D)  pocket’s

Answer: (B)

105.

(A)  goods

(B)  amenities

(C)  perks

(D)  points

Answer: (B)

106.

(A)  victorian

(B)  modern

(C)  ancient

(D)  rural

Answer: (B)

Directions-(Q. 107-136) You have 3 brief passage with 10 questions each following each passage. Read the passages carefully and choose the best answer to each question out of the four alternatives and mark it by blackening the appropriate oval in the Answer Sheet.

Passage-I

(Q. No. 107 to 116)

A pilgrimage is, of course, an expedition to some venerated place or a long and wonderful  history of human experience in divine matters, or a personal attraction affecting the soul impels one. This is, I say, it essence. But there is something more than to it than mere objective. I will visit the grave of a saint or a man whom I venerate privately for his virtues or deeds, but on the way I may wish to do something al little difficult to show at what price I hold communion with his resting place and also on the way I will see all I can of men and things, delight in the divine that is hidden in everything. Thus, I may go with nothing but my clothes and a stick but I must be open-minded and ready to give multitudinous praise to God.

A pilgrimage ought to be nothing but a nobler kind of travel, in which, according to our age and inclination we tell our tale. It is a very great error, and unknown before our most recent corruptions, that the religious spirit should be so superficial and self-conscious as to dominate our method of action at special times and to be absent at others. It is better occasionally to travel to some beloved place, haunted by our mission, yet falling into every ordinary levity, than to go about a common voyage on a chastened and devout spirit. There is another kind of pilgrimage which some few ad men undertake to ease a burdened mind. These are exempted from the rule and the adventures of the inns and foreign conversations, broaden their world and lighten their minds. The common sort, however, is a separate and human satisfaction of a need, the realization of imagined horizons and the reaching of a goal.

A pilgrimage however careless, must not be untroublesome. It would be a contradiction of pilgrimage to seek to make the journey short and rapid, merely consuming the mind for nothing, as is our modern habit; for they seem to think nowadays that to remain as near as possible to what one was at starting, and so one’s usual rut, is the great good of travel. The spirit of a pilgrimage is to comprehend the whole way, the people, their habits, the hills and clouds. And as to the method of doing this, we may go bicycling or driving but the best way is on foot, where one is a man like any other man, with the sky above one and the road beneath and the world on every side and time to see all.

107. A pilgrimage is not-

(A)  an expedition to a venerated place

(B)  a social journey for entertainment

(C)  a personal attraction affecting the soul

(D)  a journey to a historical place also known for divine matters

Answer: (B)

108. During a pilgrimage, one must-

(A)  carry a lot of clothes

(B)  see all one can of men and things

(C)  not delight in the divine that is hidden everywhere

(D)  fail to give multitudinous thanks to God

Answer: (B)

109. Pilgrimage ought to be-

(A)  a nobler kind of travel

(B)  an unparticular case of worship

(C)  nothing but a nobler kind of travel

(D)  a worship of common things

Answer: (C)

110. The corruption of the religious spirit-

(A)  occurs at special times

(B)  has been occurring for centuries

(C)  does not exist

(D)  is a recent phenomenon

Answer: (D)

111. Going on a common voyage on a chastened and devout spirit is worse than going to-

(A)  an uncommon place with a devout spirit

(B)  a beloved place and enjoying oneself

(C)  a beloved place with self-consciousness and superficiality

(D)  an uncommon place with a corrupted religious spirit

Answer: (B)

112. Few men undertake pilgrimages because-

(A)  they crave for adventure

(B)  they are overcome by a desire to visit sacred places

(C)  their minds are free from care

(D)  they have guilty consciences

Answer: (D)

113. The benefit of a pilgrimage is-

(A)  it enlarges one’s world

(B)  it can change one’s life

(C)  it tortures one’s mind

(D)  it narrows down topics of conversation

Answer: (A)

114. The most common reason for undergoing pilgrimage is to-

(A)  have rare and special experiences

(B)  realize a goal

(C)  visit extraordinary places

(D)  curb one’s instinct

Answer: (B)

115. A pilgrimage must be-

(A)  short and rapid

(B)  untroublesome

(C)  as near as possible to the starting place

(D)  different from one’s usual rut

Answer: (D)

116. The best way of travelling is-

(A)  bicycling

(B)  driving

(C)  walking

(D)  strolling

Answer: (C)

Passage-II

(Q. No. 117 to 126)

   Tagore was a man of extraordinary vision and progressive thinking. Spiritualism and rationalism were reconciled in Tagore. He had the visionary power of seeing the future in the seeds of the present events. In his book ‘Nationalism’, published in 1916. Tagore expressed his concerns about the dangers of religious fanaticism and racism which caused the most destructive wars in the history of the world. He was a seer who foresaw the destructive and dangerous effect of science and technologies on man and nature. Machines, he had predicted, would dominate man and destroy the beauty of nature and fine human qualities. Tagore’s prophecies have, to a great extent, come true.

The basic difference between the West and the East was clearly perceived by Tagore. Materialism characterizes the western civilization whereas the east is dominated by spiritualism. Tagore had known and prophesised it. However, he was optimistic of the mutual cultural enrichment through the association between the West and the East. Another important quality of Tagore as a thinker was revealed in his crusade against many superstitious beliefs of the Indians. He spoke out against caste discriminations which divided the Indian society. It required courage, intellectual as well as spiritual, to fight against the age-old customs such as child marriage and casteism. All this was possible because he was a thinker who thought and acted ahead of his own time.

117. Tagore had the visionary power to see the future in-

(A)  the seeds of the present events

(B)  western civilization

(C)  literary and social works

(D)  his books

Answer: (A)

118. Tagore expressed his concerns about the dangers of religious fanaticism and racism in his book-

(A)  ‘Materialism’

(B)  ‘Nationalism’

(C)  ‘Racism’

(D)  ‘Casteism

Answer: (B)

119. Tagore clearly perceived the basic difference between-

(A)  child marriage and casteism

(B)  progressive thinking and spirituality

(C)  culture and tradition

(D)  the West and the East

Answer: (D)

120. What was the destructive and dangerous effect that Tagore foresaw on man and nature?

(A)  Effect of science and technologies

(B)  Effect of wars in the history of the world

(C)  Cultural enrichment

(D)  Materialism

Answer: (A)

121. Which word in the passage means “violent enthusiasm in religion” ?

(A)  Sentiments

(B)  Spiritualism

(C)  Fanaticism

(D)  Crusade

Answer: (C)

122. Which of the following divides the Indian Society?

(A)  Western civilization

(B)  Intellectual division

(C)  Caste discrimination

(D)  Unprogressive thinking

Answer: (C)

123. What characterizes western civilization?

(A)  Racism

(B)  Materialism

(C)  Superstitious beliefs

(D)  Optimism

Answer: (B)

124. Tagore was optimistic of the mutual cultural ……….. through association between East and West.

(A)  enrichment

(B)  thinking

(C)  reconciliation

(D)  expression

Answer: (A)

125. Tagore’s prophecies have, to great extent come ……….

(A)  known

(B)  out

(C)  through

(D)  true

Answer: (D)

126. Tagore’s crusade was against………….

(A)  superstitious beliefs

(B)  child marriage

(C)  age-old customs

(D)  rationalism

Answer: (A)

Passage-III

(Q. No. 127 to 136)

   In his book about nutritional medicine Dr. Ray D. Strand points out that our food  industry, due to special transportation and storage techniques, has been able to make a wide variety of fruits and vegetables available nationwide throughout the year. The variety is good. But these are made available at a sacrifice. Green harvesting means picking fruits and vegetables before they mature. Shipping food over long distances requires cold storage and other preservation methods, which allow for depletion of vital nutrients. Our food is also for depletion of vital nutrients. Our food is also highly processed. For example, the refinement process of our flour to create white bread removes more than twenty-three essential nutrients, magnesium being one of the most important. Our food industry then puts about eight of these nutrients back into our bread and calls it ‘enriched’.

127. These days due to ……….. a wide variety of fruits and vegetables is available nationwide throughout the year.

(A)  cold storage facilities and preservation methods

(B)  refinement process

(C)  green harvesting

(D)  special transportation and storage techniques

Answer: (D)

128. In the passage, Magnesium is-

(A)  not an important essential nutrient in flour

(B)  one of the important essential nutrients in flour

(C)  one of the nutrients that is found in white bread

(D)  one of the most important essential nutrients in flour

Answer: (D)

129. Green harvesting is all about-

(A)  conserving the environment

(B)  picking fruits and vegetables before they mature

(C)  highly processed food

(D)  making fruits and vegetables available throughout the year

Answer: (B)

130. Shipping food over long distances allows for-

(A)  special transportation

(B)  special preservation methods

(C)  refinement process

(D)  depletion of vital nutrients

Answer: (D)

131. The refinement process of our flour to create white bread removes more than-

(A)  twenty-six essential nutrients

(B)  twenty-two essential nutrients

(C)  twenty-three essential nutrients

(D)  twenty-eight essential nutrients

Answer: (C)

132. The author is talking about-

(A)  the food industry in his own country

(B)  the universal food scenario

(C)  the food scenario in Europe

(D)  the food scenario in developed countries

Answer: (A)

133. The wide variety of fruits and vegetables from all over the world throughout the year, according to Dr. Strand,

(A)  is a sacrifice of all the nutrients

(B)  is good

(C)  shows green harvesting at its worst

(D)  shows green harvesting at its best

Answer: (B)

134. Shipping food over long distances requires-

(A)  constant processing

(B)  cold storage and other preservation methods

(C)  special transportation and storage techniques

(D)  picking fruits and vegetables before they mature

Answer: (B)

135. In the context of the passage ‘enriched’ bread indicates putting-

(A)  magnesium back into the bread

(B)  about eight of the important  nutrients back into the bread

(C)  flour back into the bread

(D)  vital nutrients back into the bread

Answer: (B)

136. By saying “food are made available at a sacrifice” the author exposes-

(A)  the depletion of vital nutrients in our food

(B)  the price of cold storage and other preservation methods

(C)  the lack of nutritional value in our food

(D)  All of the above

Answer: (D)

Directions-(Q. 137-156) Some parts of the sentences have errors and some are correct. Find out which part of a sentence has an error and blacken the oval corresponding to the appropriate letter (A, B, C). If a sentence is free from error, blacken the oval corresponding to (D) in the Answer Sheet.

137. Africa is the second largest continent (A)/ and it contain about one-fifth (B)/ of all the land in the world. (C)/ No error (D)

Answer: (B)

138. We had (A)/ hardly reached the (B)/ platform than the train came in. (C)/ No error (D)

Answer: (C)

139. When no individual is (A)/ responsible there are no (B)/ sense responsibility. (C)/ No error (D)

Answer: (B)

140. I am not sure (A)/ if (B)/ he will come. (C)/ No error (D)

Answer: (D)

141. I felt privileged (A)/ to recite a poem in an honour of my teacher (B)/ at a recent school function. (C)/ No error (D)

Answer: (B)

142. It is how (A)/ five years since (B)/ I visit the Delhi. (C)/ No error (D)

Answer: (C)

143. Prisoners, especially long term convict (A)/ have to suffer most (B)/ from emotional starvation. (C)/ No error (D)

Answer: (C)

144. This is not (A)/ a worth reading book (B)/ so don’t read it. (C)/ No error (D)

Answer: (B)

145. The beautiful, young girl (A)/ jumped in the river (B)/ in a state of depression. (C)/ No error (D)

Answer: (B)

146. The tree (A)/ is losing (B)/ its leaves. (C)/ No error (D)

Answer: (B)

147. People have (A)/ different ideas about (B)/ what makes a good holiday. (C)/ No error (D)

Answer: (D)

148. They decided (A)/ to consult (B)/ a marriage guidance counselor. (C)/ No error (D)

Answer: (D)

149. I saw (A)/ two deers (B)/ in the woods. (C)/ No error (D)

Answer: (B)

150. I enjoy jogging (A)/ and I enjoy (B)/ playing the piano. (C)/ No error (D)

Answer: (B)

151. For testing (A)/ the new microphone, (B)/I tried to record my voice. (C)/ No error (D)

Answer: (A)

152. My friends said (A)/ me that he was (B)/ unable to come. (C)/ No error (D)

Answer: (B)

153. She has married (A)/ a young tall (B)/Australian accountant. (C)/ No error (D)

Answer: (B)

154. The hurrying crowds (A)/ of people past (B)/ each other amazed him. (C)/ No error (D)

Answer: (B)

155. I have visited Mumbai two years ago (A)/ and I am planning to visit again (B)/ in the near future. (C)/ No error (D)

Answer: (A)

156. He neither talks too much (A)/ nor he causes any trouble (B)/ during the lesson. (C)/ No error (D)

Answer: (B)

Directions-(Q. 157-161) Sentences are given with blanks to be filled in with an appropriate word(s).Four alternatives are suggested for each question. Choose the correct alternatives out of the four and indicate it by blackening the appropriate oval in the Answer Sheet.

157. We always ………. your team at cricket.

(A)  overcome

(B)  beat

(C)  defeat

(D)  conquer

Answer: (D)

158. The recent ……….. in the size of the army is alarming.

(A)  built-up

(B)  built-in

(C)  build-up

(D)  build-on

Answer: (C)

159. Please ……… that the lights are switched off at night.

(A)  insure

(B)  assure

(C)  ensure

(D)  censure

Answer: (C)

160. A well-read man is quick to catch literary ………. .

(A)  allusions

(B)  illusions

(C)  delusions

(D)  diversions

Answer: (A)

161. A dog is a ……. animal.

(A)  quadruped

(B)  quartet

(C)  quadruple

(D)  quadrangle

Answer: (A)

Directions-(Q. 162-164) Out of the four alternatives, choose the one which best expresses the meaning of the given word and mark it in the Answer Sheet.

162. Affluent

(A)  Blossom

(B)  Flourish

(C)  Prosperous

(D)  Thrive

Answer: (C)

163. Peril

(A)  Danger

(B)  Roughness

(C)  Insecurity

(D)  Anger

Answer: (A)

164. August

(A)  Common

(B)  Ridiculous

(C)  Dignified

(D)  Petty

Answer: (C)

Directions-(Q. 165-167) Choose the word opposite in meaning to the given word and mark it in the Answer Sheet.

165. Servile

(A)  Defiant

(B)  Fawning

(C)  Sycophantic

(D)  Psychotic

Answer: (A)

166. Adept

(A)  Ignorance

(B)  Inept

(C)  Lacuna

(D)  Inexperience

Answer: (B)

167. Famous

(A)  Well-known

(B)  Unknown

(C)  Unfamiliar

(D)  Notorious

Answer: (B)

Directions-(Q. 168-170) Four words are given in each question, out of which only one word is correctly spelt. Find the correctly spelt word and mark your answer in the Answer Sheet.

168.

(A)  Tabaco

(B)  Tobacco

(C)  Tabocco

(D)  Toobaco

Answer: (B)

169.

(A)  Succesively

(B)  Acquaintance

(C)  Commissionner

(D)  Exaggerrate

Answer: (B)

170.

(A)  Pneumatic

(B)  Rhumatic

(C)  Emphatick

(D)  Symtomatic

Answer: (A)

Directions-(Q. 171-180) Four alternatives are given for the Idiom/Phrase underlined in the sentence. Choose the alternative which best expresses the meaning of the Idiom/Phrase and mark it in the Answer Sheet.

171. To get into hot waters.

(A)  To be impatient

(B)  To suffer loss

(C)  To get into trouble

(D)  To be in a confused state of mind

Answer: (C)

172. My student is one who know how many beans make five.

(A)  knows mathematical calculations

(B)  is fiercely loyal

(C)  well informed and intelligent

(D)  is accurate and precise

Answer: (C)

173. I can work for six hours at a stretch.

(A)  continuously

(B)  occasionally

(C)  maximum

(D)  minimum

Answer: (A)

174. It was the who put a spoke in my wheel.

(A)  tried to cause an accident

(B)  helped in the execution of the plan

(C)  thwarted in the execution of the plan

(D)  destroyed the plan

Answer: (C)

175. Our fleet bore down upon the enemy.

(A)  moved quickly backwards

(B)  moved quickly to the left

(C)  moved quickly to the right

(D)  moved quickly towards

Answer: (D)

176. The rock concert proved to be a big draw.

(A)  favourite

(B)  huge attraction

(C)  gain

(D)  big bang

Answer: (B)

177. Look sharp or you will miss the opportunity.

(A)  Be confident

(B)  Pay attention

(C)  Take control

(D)  Make haste

Answer: (C)

178. Our old Rolls Royee has become a white elephant.

(A)  a costly but useless possession

(B)  antique item or possession

(C)  any traditional possession

(D)  a unique item or possession

Answer: (A)

179. Since the children were young, their parents have always taught them to cut one’s coat according to one’s cloth.

(A)  be thrifty

(B)  live life to the full

(C)  live within one one’s means

(D)  be fair

Answer: (C)

180. The lawyer’s closing arguments were full of red herrings.

(A)  sound judgement

(B)  references and allusions to previous crimes

(C)  clues intended to distract or mislead

(D)  loopholes

Answer: (C)

Directions-(Q.181-200) The passage/sentence is split into four parts and named A, B, C, and (D). These four parts are not given in their proper order. Read the sentence and find out which of the four combinations is correct. Then find the correct answer and indicate it by blackening the appropriate oval in the Answer Sheet.

181. A explained angrily

B the bricklayer

C leaving the site

D his reasons for

(A)  B A D C

(B)  D C A B

(C)  C B A D

(D)  A D C B

Answer: (A)

182. A which had some

B had no use

C I found a firm

D components for which they

(A)  D A C B

(B)  C A D B

(C)  B D A C

(D)  C B D A

Answer: (B)

183. A the most suitable material

B because that is

C it of wood

D we made

(A)  C A D B

(B)  A C B D

(C)  D C B A

(D)  B A D C

Answer: (C)

184. A get some peace

B he left home

C his parents could

D in order that

(A)  B D C A

(B)  C B D A

(C)  D C A B

(D)  A D C B

Answer: (A)

185. A coal as a fuel

B in place of wood

C during this period

D people were beginning to use

(A)  C D A B

(B)  D C A B

(C)  B D A C

(D)  A C D B

Answer: (A)

186. A try to use the camera

B carefully

C before you

D read the manual

(A)  B D C A

(B)  D C A B

(C)  A C B D

(D)  C A D B

Answer: (D)

187. A are not allowed

B to change to a different class

C unless there is a special reason

D students

(A)  C D B A

(B)  B A D C

(C)  A C B D

(D)  D A B C

Answer: (D)

188. A she had taken a seat

B I put a woolen rug

C after

D over her knees

(A)  A D B C

(B)  C D A B

(C)  C A B D

(D)  B A C D

Answer: (C)

189. A undoubtedly to be

B our aim

C it ought

D to raise the minds of the natives

(A)  C A B D

(B)  D B A C

(C)  A D C B

(D)  B A D C

Answer: (A)

190. A indeed

B that he might have passed

C he was

D so eloquent

(A)  B A D C

(B)  D A C B

(C)  A B D C

(D)  C D A B

Answer: (D)

191. A She arrived where Gerard’s branch started.

B His heart stood still.

C The bear was mounting steadily uphill.

D But then he thanked God.

(A)  B C D A

(B)  A B C D

(C)  C A B D

(D)  D C A B

Answer: (C)

192. A while a student in London he began to take an interest in contemporary politics.

B Wells was a British author and one of the earliest writers of science fiction.

C But he later received a scholarship to study science in London.

D He came from a lower middle-class family and was apprentice to a draper at the age of fifteen.

(A)  A D C B

(B)  B D C A

(C)  B C A D

(D)  A C B D

Answer: (B)

193. A tends to create havoc

B even in small quantities

C the regular use of alcohol

D in many organs of the body

(A)  B D C A

(B)  D A C B

(C)  C B A D

(D)  A C B D

Answer: (C)

194. A I learned to walk and to talk.

B Time passed.

C I began to notice things.

D I remember my mother with her pretty hair and youthful figure  unlike Peggoty.

(A)  D C B A

(B)  B A C D

(C)  A C D B

(D)  D B A C

Answer: (B)

195. A I was about to examine the null which formed on deck a kind of horizon platform

B Daybreak appeared.

C Suddenly, I felt it gradually sinking.

D The morning mists surrounded us, but they soon cleared off.

(A)  C A B D

(B)  A B D C

(C)  D B A C

(D)  B D A C

Answer: (D)

196. A It came upon me and buried me deep in its own body and carried me swiftly towards the sea.

B I soon found it impossible to avoid it.

C I saw the sea come after me as high as a great hill and as furious as an enemy.

D I got upon my feet and endeavoured to make towards the land as fast as possible before another wave should return.

(A)  D C B A

(B)  A D B C

(C)  C B A D

(D)  B A D C

Answer: (A)

197. A At first glance all I could discern was a massive stone table running down its length.

B This apartment was not nearly so well lighted as the vast stalactite ante-cave.

C I stared down the passage and found myself in a gloomy apartment some 20 feet long, which in some past age had been hollowed out by hand out of the mountain.

D Next, I discovered a brown thing seated on the table in the centre.

(A)  B A D C

(B)  C B A D

(C)  D A B C

(D)  A B D C

Answer: (B)

198. A At last, having used up every tower, they wrote ‘H. East’ and ‘T. Brown’ on the minute hand of the great clock.

B So they climbed the walls to the top of the school, and found a number of tennis balls.

C In doing so, they held up the minute hand and so upset the clock’s timing.

D They liked it so much up there that they went back again and spent their time carving their names on the top of every tower.

(A)  B D A C

(B)  D A B C

(C)  C D B A

(D)  A C B D

Answer: (A)

199. A Tagore pointed out various evils of society of the time.

B And the beginning of the twentieth century were very tradition-bound.

C The Indian people in the nineteenth.

D Through the Brahmo Samaj he tried to abolish evil customs like child-marriage and caste system.

(A)  B A D C

(B)  C B A D

(C)  A B C D

(D)  B A C D

Answer: (B)

200. A The phantom head created by the witches warned him against Macduff.

B The second time they gave him some very ambiguous hope.

C But a bloody child and a child crowned with a golden crown encouraged him to be bold and proud.

D When Macbeth met the weird sisters or witches.

(A)  D B A C

(B)  B A C D

(C)  C A B D

(D)  D A B C

Answer: (A)

SSC CAPFs-Delhi Sub-Inspectors Examination-2014 Paper-I Question Paper With Answer Key

SSC CAPFs-Delhi Sub-Inspectors Examination-2014 Paper-I
SSC CAPFs-Delhi Sub-Inspectors Examination-2014 Paper-I Question Paper With Answer Key

SSC CAPFs/Delhi Sub-Inspectors Examination-2014 Paper-I Solved Papers

Part-A

General Intelligence & Reasoning

   Directions-(Q. 1 to 7) Select eh related word/ letter / number / figure from the given alternatives.

1. Saint : Meditation :: Scientist : ?

(A)  Research

(B)  Knowledge

(C)  Spiritual

(D)  Rational

Answer: (A)

2. King : Palace :: Eskimo : ?

(A)  Caravan

(B)  Asylum

(C)  Monastery

(D)  Igloo

Answer: (D)

3. AFKP : DINS :: WBGL : ?

(A)  ORUX

(B)  OSWA

(C)  OTYD

(D)  OQSU

Answer: (C)

4. SINGER : QGLECP :: MONSTER : ?

(A)  KLNQSCP

(B)  KLMQSCP

(C)  KMLQRCP

(D)  KLMQTDO

Answer: (C)

5. 18 : 5 :: 12 : ?

(A)  4

(B)  10

(C)  3

(D)  6

Answer: (C)

6. 12 : 20 :: ?

(A)  15 : 37

(B)  16 : 64

(C)  27 : 48

(D)  30 : 42

Answer: (D)

7.

Answer: (A)

8. In the following questions number of letters skipped in between adjacent letters in the series increased by one. Which of the following series observe the rule?

(A)  KORYBGJ

(B)  LMEYTPK

(C)  KMPTYEL

(D)  KPTYELM

Answer: (C)

9. In the following letter series how many Ms are followed by N, but not preceded by N?

NMWVMNMVWNMNMMNWVMN

(A)  1

(B)  2

(C)  3

(D)  4

Answer: (C)

Directions-(Q. 10 to 14) Find the odd word/letters/ number pair/ figure from the given alternatives.

10.

(A)  Tennis

(B)  Cricket

(C)  Volleyball

(D)  Football

Answer: (B)

11.

(A)  artificial

(B)  insincere

(C)  affected

(D)  naive

Answer: (D)

12.

(A)  68

(B)  85

(C)  153

(D)  174

Answer: (D)

13.

(A)  7: 98

(B)  9 : 162

(C)  12 : 288

(D)  17 : 572

Answer: (D)

14.

Answer: (C)

15. Which one of the given responses would be a meaningful order of the following?

(1) Orange, (2) Indigo, (3) Red (4) orange, (5), Green, (6) Yellow, (7) Violet

(A)  7, 2, 4, 5, 6, 1, 3

(B)  7, 2, 4, 6, 5, 1, 3

(C)  7, 2, 6, 4, 5, 1, 3

(D)  7, 2, 6, 4, 1, 5, 3

Answer: (A)

16. Arrange the following words as per order in the dictionary-

(1) Forecast, (2) Forget, (3) Foreign, (4) Forsook, (5) Force

(A)  3, 5, 1, 2, 4

(B)  5, 1, 3, 2, 4

(C)  5, 1, 3, 4, 2

(D)  5, 1, 2, 3, 4

Answer: (B)

17. Which one set of letters when sequentially placed at the gaps in the given letter series shall complete it?

rtx_sx_z_txy_ _ yz

(A)  y y r x s

(B)  y y s x r

(C)  y y r s x

(D)  y y x r z

Answer: (C)

Directions-(Q. 18 to 21) A series is given with one term missing. Choose the correct alternative from the given ones that will complete the series.

18. FAG, GAF, HAI, IAH, ……

(A)  JAK

(B)  HAK

(C)  JAI

(D)  HAL

Answer: (A)

19. 3, 6, 9, 15, 24, 39, 63, ?

(A)  100

(B)  87

(C)  102

(D)  99

Answer: (C)

20. −1, 0, ?, 8, 15, 24

(A)  4

(B)  3

(C)  2

(D)  1

Answer: (B)

21.

Answer: (C)

22. Govind is 48 years old. He is twice as old as his son Prem is now. How old was Prem seven years before?

(A)  16

(B)  17

(C)  13

(D)  18

Answer: (B)

23. Pointing to a man, a lady said “His mother is the only daughter of my mother”. How is the lady related to the man?

(A)  Mother

(B)  Daughter

(C)  Sister

(D)  Aunt

Answer: (A)

24. Five policemen are standing in arrow facing south. Shekhar is to the immediate right of Dhanush. Bala is between Basha and Dhanush. David is at the extreme right end of the row. Who is standing in the middle of the row?

(A)  Bala

(B)  Basha

(C)  Shekhar

(D)  Dhanush

Answer: (D)

25. From the given alternatives select the word which can be formed using the letters given in the word-

ULTRANATIONALISM

(A)  ULTRAMONTANE

(B)  ULTRAMODERN

(C)  ULTRAIST

(D)  ULULATE

Answer: (C)

26. From the given alternatives select the word which cannot be formed using the letters of the given word-

LEGALIZATION

(A)  ALERT

(B)  ALEGATION

(C)  GALLANT

(D)  NATAL

Answer: (A)

27. In a certain code DEPUTATION is written as ONTADEPUTI. How is DERIVATION written in that code?

(A)  ONVADERITI

(B)  ONDEVARITI

(C)  ONVAEDIRTI

(D)  ONVADEIRIT

Answer: (A)

28. If MADRAS is coded as 517916 and TENANT is coded as 432124, how would you encode RMATSN?

(A)  851353

(B)  951363

(C)  951462

(D)  941562

Answer: (C)

29. Find the missing number from the given responses-

(A)  12

(B)  10

(C)  9

(D)  8

Answer: (A)

30. If ‘−’ stands for ‘+’, ‘+’ stands for ‘×’, ‘×’ stands for ‘−’ then which one of the following is not correct?

(A)  22 + 7 – 3 × 9 = 148

(B)  33 × 5 – 10 + 20 = 228

(C)  7 + 28 – 3 × 52 = 127

(D)  44 – 9 + 6 × 11 = 87

Answer: (C)

31. Some equations are solved on the basis of a certain system. Find the correct answer for the unsolved equation on that basis-

5*6 = 35, 8 * 4 = 28, 6 * 8 = ?

(A)  46

(B)  34

(C)  23

(D)  38

Answer: (A)

32. Select the correct combination of mathematical signs to replace * signs and to balance the following equation-

12 * 3 * 4 = 6 * 8 * 8

(A)  +, ×, −, ×

(B)  ×, +, −, ×

(C)  ×, +, ×, −

(D)  ×, −, ×, +

Answer: (C)

Directions-(Q. 33 and 34) Select the missing number from the given responses-

33. 

(A)  17

(B)  23

(C)  47

(D)  73

Answer: (B)

34. 

(A)  1

(B)  8

(C)  6

(D)  16

Answer: (C)

35. A person walks towards his house at 8.00 am and observes his shadow to his right. In which direction he is walking?

(A)  North

(B)  South

(C)  East

(D)  West

Answer: (B)

36. A boat moves from a jetty towards East. After sailing for 9 nautical miles, the turns towards right and covers another 12 nautical miles. If she wants to go back to the jetty, what is the shortest distance now from her present position?

(A)  21 nautical miles

(B)  20 nautical miles

(C)  18 nautical miles

(D)  15 nautical miles

Answer: (D)

37. Find out the number of triangles in the given figure-

(A)  13

(B)  15

(C)  16

(D)  17

Answer: (C)

38. Four position for a dice are given below. Identify the number at the bottom when top is 6.

(A)  1

(B)  3

(C)  4

(D)  5

Answer: (A)

39. Identify the diagram that best represents the relationship among classes given below-

Food, Curd, Spoons

Answer: (D)

40. In the following figure, the boys who are cricketer and sober the indicated by which number?

(A)  6

(B)  5

(C)  4

(D)  2

Answer: (D)

Directions-(Q. 41 and 42) Some statements are given followed by three/two conclusions/assumptions respectively. You have to consider the statements to be true even if they seem to be at variance from commonly known facts. You have to decide which of the given conclusions/assumptions if any, follow from the given statements.

41. Statement :

Pictures can tell a story. All story books have pictures. Some story books have words.

Conclusions:

(I) Pictures can tell a story better than words can.

(II) The stories in story books are very simple.

(III) Some story books have both words and pictures.

(A)  Only conclusion I follows

(B)  Only conclusion II follows

(C)  Only conclusion III follows

(D)  Both conclusions I and II follow

Answer: (C)

42. Statement :

It is desirable to put the child in school at the age of 5 or so.

Assumptions:

(I) At that age the child reaches appropriate level of development and is ready to learn.

(II) The schools do not admit children after 6 years of age.

(A)  Only assumption I is implicit

(B)  Only assumption II is implicit

(C)  Neither assumption I nor II is implicit

(D)  Both assumption I and II are implicit

Answer: (A)

43. Which answer figure will complete the pattern in the question figure?

Answer: (B)

44. From the given answer figures, select the one in which the question figure is hidden/embedded-

Answer: (B)

45. A piece of paper is folded and cut as shown below in the question figures. From the given answer figures, indicate how it will appear when opened?

Answer: (B)

46. If a mirror is placed on the line MN, then which of the answer figures is the right image of the given figure?

Answer: (C)

47. A word is represented by only one set of numbers as given in any one of the alternatives. The sets of numbers given in the alternatives are represented by two classes of alphabets as in two matrices given below. The columns and rows of Matrix-I are numbered from 0 to 4 and that of Matrix-II are numbered from 5 to 9. A letter from these matrices can be represented first by its row and next by its column e.g., ‘A’ can be represented by 40, 01, 13, 32 etc., and ‘N’ can be represented by 56, 68, 89 etc. Similarly, you have to identify the set for the word given below-

(A)  86, 87, 99 – 40, 41, 86, 64

(B)  98, 96, 85 – 42, 78, 88, 77

(C)  77, 69, 76 – 22, 95, 28, 31

(D)  65, 55, 67 – 05, 25, 91, 40

Answer: (B)

48. A student is caught cheating in her test. What would you do in this situation?

(A)  Warn her and let her off

(B)  Warn her and give her counselling

(C)  Tell her parents

(D)  Put up her name on the board

Answer: (B)

49. An address has been given below, which has been reproduced against. A, B, C and D alternatives. Three of these have some mistake or the other. Identify the one without any mistake-

FG EUROFRED LIMITED

Centennial Park,

Centennial Avenue,

Elstree, Hertfordshire

United Kingdom

WD6 3SG

(A)  FG EUROFRED LIMITED

Cenetennial Park,

Elstree, Hertfordshire

United Kingdom

WD6 3SG

(B)  FG EUROFRED LIMITED

Centennial Park,

Cenetennial Avenue,

Elstree, Hertforbshire

United Kingdom

WD6 3SG

(C)  FG EUROFRED LIMITED

Centennial Park,

Centennial Avenue,

Elstree, Hertfordshire

United Kingdom

WD6 3SG

(D)  FG EUROFRED LIMITED

Centeninal Park,

Centeninal Avenue,

Elstee, Hertfordshire

United Kingdom

WD6 3SG

Answer: (C)

50. Given below are numbers in the first line and symbols in the second line. Numbers and symbols are code for each other. Choose the correct code for given symbols.

Which number can be decoded from the following-

(A)  5 8 6 3 7

(B)  5 6 8 7 3

(C)  5 7 8 6 3

(D)  5 8 3 6 7

bg_collapse view=”button-orange” color=”#4a4949″ icon=”arrow” expand_text=”Show Answer” collapse_text=”Hide Answer” ]

Answer: (A)

[/bg_collapse]

Part-B

General Knowledge and General Awareness

51. In the balance of payments account, unrequited receipts and payments are also regarded as-

(A)  bilateral transfers

(B)  unilateral transfers

(C)  capital account transfers

(D)  invisible transfers

Answer: (B)

52. Price and output are determinates in market structure other than-

(A)  monopoly

(B)  perfect competition

(C)  oligopoly

(D)  monopsony

Answer: (C)

53. A philosophy that the worker should share in industrial decisions is termed as-

(A)  industrial democracy

(B)  worker sovereignty

(C)  industrial socialism

(D)  worker dictatorship

Answer: (A)

54. If average cost falls, marginal cost-

(A)  increases at a higher rate

(B)  falls at the same rate

(C)  increases at a lower rate

(D)  falls at a  higher rate

Answer: (D)

55. Which one of the following disburses long term loans to private industry in India?

(A)  Food Corporation of India

(B)  Life Insurance Corporation of India

(C)  Primary Credit Society

(D)  Land Development Banks

Answer: (B)

56. Most important safeguard of liberty is-

(A)  bold and impartial judiciary

(B)  well-knit party system

(C)  decentralization of power

(D)  declaration of rights

Answer: (A)

57. What is t he ancient school of law?

(A)  The Philosophical school

(B)  The Historical School

(C)  The Analytical School

(D)  The Sociological School

Answer: (C)

58. Liberty stands for-

(A)  absence of restraint

(B)  consists in the presence of restraint

(C)  feeling enjoyed in a congenial atmosphere

(D)  the eager maintenance of that atmosphere in which men have the opportunity to be their best selves – Laski

Answer: (D)

59. When the Vice-President is acting as President he-

(I) will have all powers and functions of both President and Vice-President.

(II) gets all the allowances and privileges of the President

(III) should continue to work as the Chairman of the Rajya Sabha.

(A)  I, II and III

(B)  I and III

(C)  I and II

(D)  Only II

Answer: (D)

60. The Presidential Government, the President is-

(A)  independent of the Legislature

(B)  dependent on the Legislature

(C)  dependent on t he Judiciary

(D)  bound by the advice of the Council of Ministers

Answer: (A)

61. The method of amending the Constitution by popular veto is found in –

(A)  Britain

(B)  Switzerland

(C)  Russia

(D)  India

Answer: (B)

62. Which of the following is the inalienable attribute of the parliamentary system of government?

(A)  Flexibility of the Constitution

(B)  Fusion of Executive and Legislature

(C)  Judicial Supremacy

(D)  Parliamentary Sovereignty

Answer: (B)

63. Which one of the following kinds of equality is not compatible with the liberal notion of equality?

(A)  Legal Equality

(B)  Political Equality

(C)  Social Equality

(D)  Economic Equality

Answer: (D)

64. Which was the second capital of Akbar ?

(A)  Delhi

(B)  Agra

(C)  Fatehpur-Sikri

(D)  Patna

Answer: (C)

65. Thhe first country which discovered sea route to India was-

(A)  Portugal

(B)  Dutch

(C)  French

(D)  Britain

Answer: (A)

66. The unification of Karnataka was achieved in the year-

(A)  1956

(B)  1957

(C)  1958

(D)  1960

Answer: (A)

67. Who introduced the Indian University Act?

(A)  Lord Curzon

(B)  Lord Minto

(C)  Lord Morelay

(D)  Lord Rippon

Answer: (A)

68. Chinese travelers visited India primarily because-

(A)  they were interested in Buddhism

(B)  they were invited by the Indian kings

(C)  they were interested to study Indian culture

(D)  they were interested to stay in India

Answer: (A)

69. The first short based integrated steel plant in the country is-

(A)  Vijaynagar

(B)  Salem

(C)  Vishakhapatnam

(D)  Bhadravati

Answer: (C)

70. Name the condition which influences the development of plants into distinctive forms-

(A)  Climatic conditions

(B)  Soil conditions

(C)  Environmental conditions

(D)  Social conditions

Answer: (C)

71. Agriculture should serve as an instrument of income, livelihood and opportunity to the local community-this statement is given by-

(A)  Dr. Madhavan Nair

(B)  Dr. Manmohan Singh

(C)  Dr. Abdul Kalam

(D)  Dr. M.S. Swaminathan

Answer: (D)

72. When was the Geological Survey of India established?

(A)  1841

(B)  1851

(C)  1941

(D)  1951

Answer: (B)

73. Development that meets the needs of the present, without compromising the ability of future generations to meet their own needs was the focal point of Brundtland Commission is-

(A)  sustainable development

(B)  mitigation

(C)  disaster management

(D)  capacity building

Answer: (A)

74. The pancreas secretes-

(A)  Insulin

(B)  Bile juice

(C)  Peptic juice

(D)  None of these

Answer: (A)

75. When we touch leaves of “Touch me not plant”, they close, these movements are called-

(A)  photonastic movements

(B)  nyctinastic movements

(C)  seismonastic movements

(D)  chemonastic movements

Answer: (C)

76. The concept of tissue culture was introduced by-

(A)  Halfmeister

(B)  Hanstein

(C)  Haberlandt

(D)  Hanning

Answer: (C)

77. Beak is formed by-

(A)  cheeks

(B)  jaws

(C)  teeth

(D)  None of these

Answer: (B)

78. Pinna (external ear) is present in-

(A)  Amphibian

(B)  Fish

(C)  Mammal

(D)  Reptile

Answer: (C)

79. Purity of a metal can be determined with the help of-

(A)  Pascal’s law

(B)  Boyle’s law

(C)  Archimedes principle

(D)  Conservation of mass principle

Answer: (C)

80. If both the mass and the velocity of a body is increased to twice of their magnitude, the kinetic energy will increase by-

(A)  2 times

(B)  4 times

(C)  8 times

(D)  16 times

Answer: (C)

81. Two bodies kept at a certain distance feel a gravitational force F to each other. If the distance between them is made double the former distance, the force will be-

(A)  2F

(B) 

(C)  4F

(D) 

Answer: (D)

82. Stationary wave is formed by-

(A)  a transverse wave superposing a longitudinal wave

(B)  two waves of the same speed superposing

(C)  two waves of same frequency travelling in the same direction

(D)  two waves of same frequency travelling in the opposite direction

Answer: (D)

83. Which is not an extension of a picture file on a computer?

(A)  .jpeg

(B)  .png

(C)  .gif

(D)  .mdb

Answer: (D)

84. Which measure of memory is the largest?

(A)  MB Megabyte

(B)  GB Gigabyte

(C)  TB Terabyte

(D)  KB Kilobyte

Answer: (C)

85. Which of the following is the maximum number of electrons that can be present in M-shell?

(A)  2

(B)  8

(C)  18

(D)  32

Answer: (C)

86. In an oxygen molecule, two atoms are united by-

(A)  one bond

(B)  two bonds

(C)  three bonds

(D)  four bonds

Answer: (B)

87. The inert gas which is substituted for nitrogen in the air used by deep sea divers for breathing is-

(A)  Neon

(B)  Krypton

(C)  Argon

(D)  Helium

Answer: (D)

88. How many neutrons are there in 92U238 atom?

(A)  92

(B)  238

(C)  146

(D)  330

Answer: (C)

89. Root pressure is measured by-

(A)  Barometer

(B)  Atmometer

(C)  Manometer

(D)  Auxanometer

Answer: (C)

90. Cell becomes turgid because of-

(A)  Plasmolysis

(B)  Exosmosis

(C)  Endosmosis

(D)  Diffusion

Answer: (C)

91. The process of imbibition involves-

(A)  Diffusion

(B)  Capillary action

(C)  Absorption

(D)  Both (A) and (B)

Answer: (D)

92. A cell increases in volume when it is placed in-

(A)  Hypertonic solution

(B)  Hypotonic solution

(C)  Isotonic solution

(D)  None of these

Answer: (B)

93. Who launched National Peoples Party on 5th January, 2013?

(A)  Somnath Chatterjee

(B)  Shivraj Patil

(C)  K. S. Hegde

(D)  P. A. Sangma

Answer: (D)

94. Which Indian University bagged best Research Award at 100th Indian Science Congress?

(A)  University of Delhi

(B)  University of Kashmir

(C)  University of Mumbai

(D)  University of Sikkim

Answer: (B)

95. Who was the first woman winner of Kaun Banega Crorepati 6?

(A)  Kirand Bedi

(B)  Lata Mangeshkar

(C)  Sumeet Kaur Sawhney

(D)  Priety Zinta

Answer: (C)

96. Who was appointed new United State Defence Secretary in January 2013?

(A)  Donald Rumsfeld

(B)  Robert M Gates

(C)  Leon Panetta

(D)  Chuck Hagel

Answer: (D)

97. Who took the charge as new Deputy Governor of Reserve Bank of India in January 2013?

(A)  Urijit Patel

(B)  Dr. C. Rangarajan

(C)  S. Venktaramanam

(D)  Dr.  Y. Venugopal Reddy

Answer: (A)

98. Which political leader was sent to jail for teacher recruitment scam in January 2013?

(A)  Om Prakash Chautala

(B)  D. V. Sadananda Gowda

(C)  Raja Bhaiya

(D)  Suresh Kalmadi

Answer: (A)

99. Which player got stripped of his seven Tour de France titles and got banned for life after the International Cycling Union?

(A)  Nicola Adams

(B)  Chris Hoy

(C)  Lance Armstrong

(D)  Jason Kenny

Answer: (C)

100. Who won the Top Ranker’s Entrepreneurial Path Breaker Award 2013?

(A)  Prof. D. K. Bandyopadhyay

(B)  Ajay Shankar

(C)  B. Prasada Rao

(D)  Pratip Chaudhuri

Answer: (C)

Part-C

Quantitative Aptitude

101. 

(A)  2

(B)  3

(C)  4

(D)  5

Answer: (B)

102. Three tankers contain 403 litres, 434 litres, 465 litres of diesel respectively. Then the maximum capacity of a container that can measure the diesel of the three container exact number of times is-

(A)  31 litres

(B)  62 litres

(C)  41 litres

(D)  84 litres

Answer: (A)

103. The value of   is –

(A)  17/60

(B)  60/13

(C)  60/19

(D)  13/60

Answer: (B)

104. If ‘+’ means ‘÷’, ‘×’ means ‘−’, ‘÷’ means ‘×’ and ‘−’ means ‘+’, what will be the value of the following expression?

9 + 3 ÷ 4 – 8 × 2 = ?

(A) 

(B) 

(C) 

(D)  18

Answer: (D)

105. The next term of the sequence,   is

(A)  3

(B) 

(C)  5

(D) 

Answer: (A)

106. A can do 7/8 of work in 28 days, B can do 5/6 of the same work in 20 days. The number of days they will take to complete if they do it together is-

(A) 

(B) 

(C) 

(D) 

Answer: (D)

107. Seventy-five men are employed to lay down a railway line in 3 months. Due to certain emergency conditions, the work was to be finished in 18 days. How many more men should be employed to complete the work in the desired time?

(A)  300

(B)  325

(C)  350

(D)  375

Answer: (A)

108. A trader who marks his goods up to 50% offered a discount of 20%. What % profit the trader makes after offering the payment?

(A)  30%

(B)  70%

(C)  20%

(D)  50%

Answer: (C)

109. A retailer buys a sewing machine at a discount of 15% and sells it for Rs 1955. Thus he makes a profit of 15%. The discount is-

(A)  Rs 270

(B)  Rs 290

(C)  Rs 300

(D)  Rs 310

Answer: (C)

110. If x : y :: 2 : 3 and 2 : x : : 4 : 8 the value of y is-

(A)  6

(B)  8

(C)  4

(D)  12

Answer: (A)

111. Rs 730 were divided among A, B, C in such a way that if A gets Rs 3, then B gets Rs 4 and if B gets Rs 3.50 then C gets Rs 3. The share of B exceeds that of C by-

(A)  Rs 30

(B)  Rs 40

(C)  Rs 70

(D)  Rs 210

Answer: (B)

112. A boy found that the average of 20 numbers is 35 when he writes a number ‘61’ instead of ‘16’. The correct average 20 number is-

(A)  32.75

(B)  37.25

(C)  34.75

(D)  34.25

Answer: (A)

113. 11 friends went to a hotel and decided to pay the bill amount equally. But 10 of them could pay Rs 60 each, as a result 11th has to pay Rs 50 extra than his share. Find the amount paid by him-

(A)  Rs 105

(B)  Rs 110

(C)  Rs 115

(D)  Rs 120

Answer: (B)

114. A tea-merchant professes to sell tea at cost price but uses a false weight of 900 gram for a kilogram. The profit per cent in his transaction is-

(A) 

(B)  10%

(C) 

(D)  15%

Answer: (A)

115. Mahesh earned a profit of 20% by selling 60 apples at the rate of Rs 42.50 for 5 apples. Then the total cost, at which the apples were bought is-

(A)  Rs 452

(B)  Rs 425

(C)  Rs 450

(D)  Rs 485

Answer: (B)

116. In an examination 75% candidates passed in English and 60% passed in Mathematics. 25% failed in both and 240 passed the examination. Find the total number of candidates.

(A)  492

(B)  300

(C)  500

(D)  400

Answer: (D)

117. If 40% of 4/5 of 3/4 of a number is 48, then what is 1% of the same number?

(A)  20

(B)  2

(C)  10

(D)  1

Answer: (B)

118. A student goes to school at the rate of 5/2 km/hr and reaches 6 minutes late. If he travels at the speed of 3 km/hr, he reaches 10 minutes earlier. The distance of the school is-

(A)  45 km

(B)  20 km

(C)  10 km

(D)  4 km

Answer: (D)

119. A train 50 metre long passes a platform 100 metre long in 10 sec. The speed of the train in km/hr is-

(A)  10

(B)  54

(C)  15

(D)  100

Answer: (B)

120. A sum amounts double in 8 years b y simple interest. Then the rate of simple interest p.a. is-

(A)  10%

(B)  12.5%

(C)  15%

(D)  20%

Answer: (B)

121. Rekha invested a sum of Rs 12000 at 5% per annum compound interest. She received an amount of Rs 13230 after n years. Find n.

(A)  2.8 years

(B)  3.0 years

(C)  2.5 years

(D)  2.0 years

Answer: (D)

Directions-(Q. 122 to 125) The bar chart representing the number of first year B.Com. students of St. Xavier’s College using different companies’ smart phones. Study bar chart and answer question.

122. The ratio of the number of boys to the number of girls using the smart phones of Samsung and Sony together is-

(A)  12 : 13

(B)  13 : 12

(C)  14 : 11

(D)  11 : 14

Answer: (B)

123. What percentage of boys are using the smart phones of Samsung?

(A)  16.52%

(B)  17.52%

(C)  18.52%

(D)  15.52%

Answer: (C)

124. What percentage of girls are using the smart phones of Nokia?

(A)  33.58%

(B)  32.58%

(C)  30.58%

(D)  31.58%

Answer: (D)

125. The difference between the total number of students using smart phone of Samsung combined together and the total number of students using smart phone of Sony taken together is-

(A)  20

(B)  60

(C)  80

(D)  40

Answer: (B)

126. The radius of a right circular cone is 3 cm and its height is 4 cm. The total surface area of the cone is-

(A)  48.4 sq.cm

(B)  64.4 sq.cm

(C)  96.4 sq.cm

(D)  75.4 sq.cm

Answer: (D)

127. A wooden box of dimension 8 metre × 7 metre × 6 metre is to carry rectangular boxes of dimensions 8 cm × 7 cm × 6 cm. The maximum number of boxes that can be carried in 1 wooden box is-

(A)  7500000

(B)  9800000

(C)  1200000

(D)  1000000

Answer: (D)

128. Two circular cylinders of equal volume have their heights in the ratio 1 : 2; Ratio of their radii is (Take π = 22/7)-

(A)  1 : 4

(B)  1 : √2

(C)  √2 : 1

(D)  1 : 2

Answer: (C)

129. A rectangular piece of paper of dimensions 22 cm by 12 cm is rolled along its length to form a cylinder. The volume (in cm3) of the cylinder so formed is (use π = 22/7)-

(A)  562

(B)  412

(C)  462

(D)  362

Answer: (C)

130. A sphere is placed inside a right circular cylinder so as to touch the top, base and the lateral surface of the cylinder. If the radius of the sphere is R, the volume of the cylinder is-

(A)  2πR3

(B)  4πr3

(C)  8πR3

(D) 

Answer: (A)

131. If a = √6 + √5, b = √6 – √5, then 2a2 – 5ab + 2b2 =

(A)  38

(B)  39

(C)  40

(D)  41

Answer: (B)

132. If  then  is equal to

(A)  4/27

(B)  5/27

(C)  8/27

(D)  10/27

Answer: (C)

133. If 

(A)  0

(B)  1

(C)  −1

(D)  2

Answer: (D)

134. If a = 331, b = 336 and c = −667, then the value of a3 + b3 + c3 – 3abc is-

(A)  1

(B)  6

(C)  3

(D)  0

Answer: (D)

135. The simplified value of (√6 + √10 – √21 – √35) (√6 – √10 + √21 – √35) is-

(A)  13

(B)  12

(C)  11

(D)  10

Answer: (D)

136. In a ∆ABC  and ∠C = 50°, then ∠BAD = ?

(A)  60°

(B)  20°

(C)  30°

(D)  50°

Answer: (C)

137. In a ∆ ABC, AD, BE and CF are three medians. The perimeter of ∆ ABC is always-

(A) 

(B) 

(C) 

(D)  None of these

Answer: (B)

138. In a ∆ ABC,  are three medians. Then the ratio  is-

(A)  equal to 3/4

(B)  less than 3/4

(C)  greater than 3/4

(D)  equal to 1/2

Answer: (C)

139. Two circles with radii 25 cm and 9 cm touch each other externally. The length of the direct common tangent is-

(A)  34 cm

(B)  30 cm

(C)  36 cm

(D)  32 cm

Answer: (B)

140. If AB = 5 cm, AC = 12 and BA ⊥ AC then, the radius of the circumcircle of ∆ ABC is-

(A)  6.5 cm

(B)  6 cm

(C)  5 cm

(D)  7 cm

Answer: (A)

141. If  and sec2θ + tan2θ = 7, then θ is-

(A) 

(B) 

(C) 

(D) 

Answer: (B)

142. The simplest value of sin2x + 2tan2x – 2 sec2x is-

(A)  1

(B)  0

(C)  −1

(D)  2

Answer: (C)

143. A kite is flying at a height of 50 metres. If the length of string is 100 metres, then the inclination of string to the horizontal ground in degree measure is-

(A)  90

(B)  60

(C)  45

(D)  30

Answer: (D)

144. From the top of a light-house at a height 20 metres above sea-level, the angle of depression of a ship is 30°. The distance of the ship from the foot of the light-house is-

(A)  20 m

(B)  20√3 m

(C)  30 m

(D)  30√3 m

Answer: (B)

145. If x = a sin θ and y = b tan θ, then prove that  is-

(A)  1

(B)  2

(C)  3

(D)  4

Answer: (A)

Directions-(Q. 146 to 150) The following pie-chart shows the monthly expenditure of a family on food, house rent, clothing, education, fuel and miscellaneous. Study the pie-chart and answer question.

  1. If the expenditure for food is Rs 9000, then the expenditure for education is-

(A)  Rs 5,000

(B)  Rs 5,200

(C)  Rs 5,400

(D)  Rs 6,000

Answer: (C)

147. The central angle of the sector for the expenditure on fuel (in degrees) is-

(A)  50.4

(B)  54

(C)  57.6

(D)  72

Answer: (B)

148. If the expenditure on fuel is Rs 3,000, then the total expenditure excluding expenditure on house rent and education is-

(A)  Rs 11,600

(B)  Rs 12,000

(C)  Rs 12,400

(D)  Rs 12,500

Answer: (C)

149. If the percentage of expenditure on food is x% of the total percentage of expenditure on clothing, education and fuel, then x equals-

(A)  66

(B) 

(C) 

(D)  67

Answer: (C)

150. Total percentage of expenditure on house rent, clothing and fuel is greater than the percentage of expenditure on food by-

(A)  16

(B)  17

(C)  18

(D)  20

Answer: (B)

Part-D

English Comprehension

   Directions-(Q. 151 to 155) Some parts of the sentences have errors and some are correct. Find out which part of a sentence has an error and blacken the oval corresponding to the appropriate letter (A), (B), (C). If a sentence is free from error, blacken the oval corresponding to (D) in the Answer Sheet.

151. In 1906 a earthquake (A)/ destroyed much (B)/ of San Francisco.(C)/ No error (D)

Answer: (A)

152. His parents does not(A) / approve of (B)/ his business. (C)/ No error (D)

Answer: (B)

153. The college library is (A)/ not only equipped with (B/ very good books but also with the latest journals. (C)/ No error (D)

Answer: (B)

154. The lovers walked (A)/ besides each other (B)/ in silence. (C)/ No error (D)

Answer: (B)

155. Men are wanted (A)/ for the army/ and the navy, and the air force. (C)/ No error (D)

Answer: (C)

Directions-(Q. 156 to 160) Sentences are given with blanks to be filled in with an appropriate word(s). Four alternatives are suggested for each question. Choose the correct alternative out of the four and indicate it by blackening the appropriate oval in the Answer Sheet.

156. China is a big country, in area it is bigger than any other country ……. Russia.

(A)  accept

(B)  except

(C)  expect

(D)  access

Answer: (B)

157. The masks worn by the men helped them…….. their identity.

(A)  conceal

(B)  congeal

(C)  masquerade

(D)  cheat

Answer: (A)

158. On the …………. Occasion of Laxmi Puja the Mathurs bought a new car.

(A)  officious

(B)  auspicious

(C)  fortuitous

(D)  prosperous

Answer: (B)

159. Precautions are to be taken with any one who seems-

(A)  Contagious

(B)  Infectious

(C)  Diseased

(D)  Defiled

Answer: (A)

160. The treasure was hidden ……… a big shore.

(A)  on

(B)  underneath

(C)  toward

(D)  off

Answer: (B)

Directions-(Q. 161 to 165) Out of the four alternatives, choose the one which best expresses the meaning of the given word and mark it in the Answer Sheet.

161. Scorn

(A)  Concise

(B)  Despise

(C)  Bias

(D)  Fierce

Answer: (B)

162. Catastrophe

(A)  Tragedy

(B)  Anger

(C)  Violence

(D)  Hatred

Answer: (A)

163. Abjure

(A)  Renounce

(B)  Announce

(C)  Pronounce

(D)  Denounce

Answer: (A)

164. Assess

(A)  Overload

(B)  Measure

(C)  Permit

(D)  Enter

Answer: (B)

165. Elastic

(A)  Free

(B)  Liberal

(C)  Flexible

(D)  Broad

Answer: (C)

   Directions-(Q. 166 to 170) Choose the word opposite in meaning to the given word and mark it in the Answer Sheet.

166. Safe

(A)  Rash

(B)  Insecure

(C)  Beneficial

(D)  Harsh

Answer: (B)

167. Redundant

(A)  Repentant

(B)  Surplus

(C)  Singular

(D)  Required

Answer: (D)

168. Fair

(A)  Untrue

(B)  Unjust

(C)  Coarse

(D)  Harsh

Answer: (B)

169. Boisterous

(A)  Serenity

(B)  Calm

(C)  Cheerful

(D)  Courageous

Answer: (B)

170. Substantial

(A)  Flimsy

(B)  Hefty

(C)  Actual

(D)  Excess

Answer: (A)

Directions-(Q. 171 to 175) Four alternatives are given for the Idiom/Phrase bold in the sentence. Choose the alternative which best expresses the meaning of the Idiom/Phrase and mark it in the Answer Sheet.

171. We must work with all our might and main, otherwise we cannot succeed.

(A)  full force

(B)  complete trust

(C)  exceptional skill

(D)  full unity

Answer: (A)

172. The sailors nailed their colours to their mast.

(A)  put up a colourful mast

(B)  refused to climb down

(C)  took over the ship

(D)  decided to abandon the ship

Answer: (B)

173. We had (had) better batten down the hatches. The weather is unpredictable.

(A)  stay in-door

(B)  prepare for a difficult situation

(C)  go somewhere safe

(D)  face the obstacles

Answer: (B)

174. It is difficult to have a sensible discussion with her as she flies off at a tangent.

(A)  gets carried away

(B)  starts discussing something irrelevant

(C)  loses her temper easily

(D)  does not really understand anything

Answer: (B)

175. The students found it hard to go at equal speed with the professor.

(A)  get away from

(B)  put up with

(C)  keep up with

(D)  race against

Answer: (C)

Directions-(Q. 176 to 180) A sentence/a part of the sentence is bold. Below are given alternatives to the bold part at (A), (B), (C) which may improve the sentence. Choose the correct alternative. In case no improvement is needed your answer is (D). Mark your answer in the Answer Sheet.

176. He will revise it when he is comes back.

(A)  when he come back

(B)  on coming back

(C)  when he came back

(D)  No improvement

Answer: (B)

177. The members of the student’s union did not give the examination in protest.

(A)  did not write

(B)  did not sit for

(C)  did not show  up for

(D)  No improvement

Answer: (B)

178. She was kept from her assignment because of her loquacious room-mate.

(A)  noisy

(B)  irritating

(C)  talkative

(D)  No improvement

Answer: (C)

179. James epitomizes everything that a leader should be-

(A)  worships

(B)  loves

(C)  adores

(D)  No improvement

Answer: (D)

180. The businessman is respectively

(A)  respectfully

(B)  respectably

(C)  receptively

(D)  No improvement

Answer: (B)

Directions-(Q. 181 to 185) Out of the four alternatives, choose the one which can be substituted for the given words/sentences and indicate it by blackening the appropriate oval in the Answer Sheet.

181. One who offers his service of his own free will.

(A)  Worker

(B)  Slave

(C)  Volunteer

(D)  Servant

Answer: (C)

182. One who is always doubting.

(A)  Sceptic

(B)  Deist

(C)  Rationalist

(D)  Positivist

Answer: (A)

183. A collection of slaves.

(A)  Coffle

(B)  Crew

(C)  Company

(D)  Cortege

Answer: (A)

184. A professional soldier hired to serve in a foreign army.

(A)  Mercenary

(B)  Liquidator

(C)  Venal

(D)  Hireling

Answer: (A)

185. Not likely to be easily pleased.

(A)  Fastidious

(B)  Infallible

(C)  Fatalist

(D)  Communist

Answer: (A)

Directions-(Q. 186 to 190) Four words are given in each question, out of which only one word is correctly spelt. Find the correctly spelt word and mark your answer in the Answer Sheet.

186.

(A)  Resillient

(B)  Presuade

(C)  Dubious

(D)  Depplete

Answer: (C)

187.

(A)  Ineffectual

(B)  Iniffectual

(C)  Inefecttual

(D)  Inefictual

Answer: (A)

188.

(A)  Massacer

(B)  Massecre

(C)  Masacre

(D)  Massacre

Answer: (D)

189.

(A)  Conciance

(B)  Consience

(C)  Conscience

(D)  Connscience

Answer: (C)

190.

(A)  Nirvana

(B)  Nirvena

(C)  Nirvanna

(D)  Nyrvana

Answer: (A)

Directions-(Q. 191 to 200) You have two brief passages with 5 questions following each passage. Read the passages carefully and choose the best answer to each question out of the four alternative and mark it by blackening the appropriate oval in the Answer Sheet.

Passage-I

(Q. 191 to 195)

   Street theatre in India is a well established ancient art form. Despite the proliferation of modern means of entertainment and communication, street theatre continues to flourish in India.

Street theatre as a channel of communication has for centuries been propagating reforms by highlighting social, economic and political issues present in the society. Unlike in the olden days, its performance is no longer restricted to villages or small localities of the city. Today small groups of performers including students, would stage performances to mobilize public opinion or to help create or raise awareness over a particular issue of public importance. Themes on substance abuse, AIDS awareness, and domestic violence are some of the area highlighted by contemporary street theatre troupe. Unlike in regular drama street drama employ very little props and images. The human body becomes the main tool in which choreography, mime, dialogues, songs and slogans are extensively used.

Street theatre is one of the most intimate media. Its appeal is to the emotions leading to quick psychological impact on audiences. By being local and live they also are able to establish not only direct contact with the audience but by being cost-effective and flexible they are popular among all age groups.

191. Modern means of entertainment and communication ………. Street theatre.

(A)  does affect

(B)  does not affect

(C)  helps popularise

(D)  helps establish

Answer: (B)

192. In the olden days street theatre ………. To villages or small localities of the city.

(A)  was restricted

(B)  was not restricted

(C)  was opened

(D)  was entertained

Answer: (A)

193. Street theatre usually ………. With issues of public importance.

(A)  is distanced

(B)  is performed

(C)  deals

(D)  does not deal

Answer: (C)

194. Street theatre is ………. to stage.

(A)  nothing

(B)  costly

(C)  reasonable

(D)  affordable

Answer: (D)

195. Street theatre creates an/a ……….. impact on audiences.

(A)  intimate

(B)  emotional

(C)  mystical

(D)  physical

Answer: (B)

Passage-II

(Q. 196 to 200)

   Self-directed learning, in its broadest meaning, describes a process in which individuals take the initiative with or without the help of others, in diagnosing their learning needs formulating learning goals, identifying resources for learning, choosing and implementing learning strategies and evaluating learning outcomes. Thus, it is important to attain new knowledge easily and skillfully for the rest of his or her life.

What is the need for self-directed learning ? One reason is that there is convincing evidence that people, who take the initiative in learning, learn more things and learn better than people who sit at the feet of teachers passively waiting to be taught. The second reason is that self-directed learning is more in tune with our natural processes of psychological development; an essential aspect of maturing is developing the ability to take increasing responsibility of our own lives to become increasingly self-directed. The third reason is that many of the new developments in education put a heavy responsibility on the learners to take a good deal of initiative in their own learning. To meet the challenges in today’s instructive environment, self-directed learning is most essential.

196. In self-directed learning, an individual-

(A)  Takes initiative with or without the help of others

(B)  Is passive and waits for directions

(C)  Is helpless and dependent

(D)  Takes initiative, without an objective

Answer: (A)

197. There is need for self-directed learning because-

(A)  It is less challenging

(B)  It helps people to learn more things and learn better

(C)  It is a more cost-effective method

(D)  It is a modern method of learning

Answer: (B)

198. Which word best describes self-directed learning?

(A)  Active learning

(B)  Passive learning

(C)  Compulsory learning

(D)  Repulsive learning

Answer: (A)

199. The modern environment according to the author is-

(A)  Restrictive

(B)  Instructive

(C)  Less developed

(D)  Impracticable

Answer: (B)

200. The synonym of the word ‘diagnosing’ is-

(A)  Searching

(B)  Examining

(C)  Identifying

(D)  Complying

Answer: (C)

SSC CAPFs-Delhi Sub-Inspectors Examination-2015 Paper-I Question Paper With Answer Key

SSC CAPFs-Delhi Sub-Inspectors Examination-2015 Paper-I
SSC CAPFs-Delhi Sub-Inspectors Examination-2015 Paper-I Question Paper With Answer Key

SSC CAPFs/Delhi Sub-Inspectors Examination-2015 Solved Papers

Part-A

General Intelligence & Reasoning

   Directions-(Q. 1 to 5) Select the related word/letters/number from the given alternatives.

1. 9 : 24 :: ? : 6 –

(A)  5

(B)  1

(C)  2

(D)  3

Answer: (D)

2. DCBA : WXYZ :: IJKL : ?

(A)  PONM

(B)  SRQP

(C)  RQPO

(D)  QPON

Answer: (C)

3. hive : bee : : eyrie : ?

(A)  parrot

(B)  eagle

(C)  sparrow

(D)  pigeon

Answer: (B)

4. STAR : RATS : : WARD : ?

(A)  FAME

(B)  SHINE

(C)  MICE

(D)  DRAW

Answer: (D)

5. DHLPTX : BFJNRV :: CGKOSW : ?

(A)  AIOMUQ

(B)  IJKLMN

(C)  AEIMQU

(D)  UMQEIA

Answer: (C)

6. Choose the similar group of numbers on the basis of certain common properties they posses- (31, 19, 41)

(A)  (27, 29, 47)

(B)  (29, 17, 37)

(C)  (23, 33, 51)

(D)  (16, 27, 11)

Answer: (B)

   Direction-(Q. 7 to 12) Find the odd number/letters/number pair from the given alternatives.

7. 

(A)  Brass

(B)  Steel

(C)  Tin

(D)  Bronze

Answer: (C)

8. 

(A)  (47, 49)

(B)  (5, 7)

(C)  (11, 13)

(D)  (29, 31)

Answer: (A)

9.

(A)  QIEM

(B)  HIUL

(C)  ZOIV

(D)  TEAP

Answer: (B)

10.

(A)  Rose

(B)  Tulip

(C)  Marigold

(D)  Lotus

Answer: (D)

11.

(A)  1 1 8 9

(B)  1 2 3 4

(C)  9 1 0 0

(D)  8 1 1 0

Answer: (A)

12.

(A)  mmmqqqttt

(B)  ccc ggg kkk

(C)  kkk ooo sss

(D)  bbb fff jjj

Answer: (A)

13. Which one of the given responses would be a meaningful order of the following?

(1) Rivulet          (2) Ocean

(3) Tributary      (4) River

(5) Sea                (6) Rain

(A)  6 1 3 4 5 2

(B)  5 6 1 3 2 4

(C)  6 5 3 1 2 4

(D)  6 4 1 3 5 2

Answer: (D)

14. Arrange the following according to Dictionary-

(1) Fenestration              (2) Feather

(3) Feed head                 (4) Feature

(5) Feminine

(A)  4, 2, 3, 1, 5

(B)  2, 4, 1, 5, 3

(C)  2, 4, 3, 5, 1

(D)  4, 2, 3, 5, 1

Answer: (C)

Direction-(Q. 15 and 16) Which one set of letters when sequentially placed at the gaps in the given letter series shall complete it?

15. LU _ TUPLUBTU _ LUBT _ P _ UBTUP

(A)  LBPU

(B)  PBUL

(C)  BUPL

(D)  BPUL

Answer: (D)

16. B_ CCABB _ CABBC _ AB – CCA

(A)  BBBC

(B)  BCBC

(C)  BBCC

(D)  BCCB

Answer: (D)

   Direction-(Q. 17 and 18) Series is given with one/two term/s missing. Choose the correct alternative from the given ones that will complete the series.

17. 24, 35, 20, 31, 16, 27 ?, ?,

(A)  9, 9

(B)  12, 23

(C)  8, 25

(D)  5, 30

Answer: (B)

18. 

(A)  50/2

(B) 

(C) 

(D)  35

Answer: (A)

19. A man climbing up a wall of 24 metres high, climbs 16 m one day but slipped back by 3 m 40 cms in the evening. How far had the man reached on that day?

(A)  12.6m

(B)  11.4m

(C)  12 m 40 cm

(D)  19 m 40 cm

Answer: (A)

20. Two persons A and B get the same salary. Their basic pay are different. The allowances are 65% and 80% of the basic pay respectively. What is the ratio of the basic pay?

(A)  12 : 11

(B)  7 : 5

(C)  17 : 15

(D)  11 : 10

Answer: (A)

21. Two horses A and B run at a speed of 3 : 2 ratio in the first lap: during the second lap the ratio differs by 4 : 7; during the third lap the ratio differs by 8 : 9. What is the difference in ratio of speed altogether between the two horses?

(A)  1

(B)  3

(C)  4

(D)  2

Answer: (A)

   Directions- (Q. 22 and 23) From the given alternative words, select the word which cannot be formed using the letters of the given word-

22. CHARACTER

(A)  Hearty

(B)  Charter

(C)  Crate

(D)  Tracer

Answer: (A)

23. ADMINISTRATION

(A)  Station

(B)  Mind

(C)  Ration

(D)  Minister

Answer: (D)

24. In a language FIFTY is written as CACTY, CAR as POL, TAR as TOL how can TARIFF be written in that language?

(A)  TOEFDD

(B)  TOEFEL

(C)  TOLACC

(D)  TOLADD

Answer: (C)

25. Some equations are solved on the basis of certain system. Find out the correct answer for the unsolved equation on that basis-

7 × 6 × 4 = 674, 8 × 5 × 3 = 583, 9 × 1 × 2 = ?

(A)  462

(B)  727

(C)  192

(D)  292

Answer: (C)

26. If GOODNESS is coded as HNPCODTR, how can GREATNESS be written in that code?

(A)  HQFZUFRTM

(B)  HQFZUODTR

(C)  HQFZUMFRT

(D)  HQFZSMFRT

Answer: (C)

27. Change symbol and solve accordingly to find out correct answer from the alternatives given below-

9 × 8 × 7 = 24, 4 × 7 × 3 = 14, 2 × 1 × 9 = ?

(A)  18

(B)  12

(C)  10

(D)  11

Answer: (B)

28. If ‘a’ represents ÷, ‘b’ represents +, ‘c’ represents – and ‘d’ represents ×, then 24a6d4b9c8 = ?

(A)  6

(B)  20

(C)  19

(D)  17

Answer: (D)

   Directions-(Q. 29 to 31) Select the missing number from the given responses.

29. 

(A)  24

(B)  12

(C)  18

(D)  19

Answer: (D)

30. 

(A)  10

(B)  7

(C)  8

(D)  9

Answer: (B)

31. 

(A)  76

(B)  53

(C)  68

(D)  71

Answer: (C)

32. 

How many faces can  you count in this 3 – dimensional model?

(A)  14

(B)  18

(C)  12

(D)  16

Answer: (A)

33. Keeping her back towards the rising sun, Reshma starts walking. After a few minutes, she turns left and keeps on walking. Then a little later she turns right and then left. In which direction is she going at the moment?

(A)  South or West

(B)  East or South

(C)  West or North

(D)  North of South

Answer: (A)

34. Which of the following states the relationship between Sociology, Humanities and Psychology?

Answer: (A)

35. From her home facing south-west, Kavita walks 15 m and turns northward and walks 12 m. How far was she from her home, which was her starting point?

(A)  15 m

(B)  12 m

(C)  10 m

(D)  9 m

Answer: (D)

36. Which figure best represents the relationship between Editor, Newspaper and Journalist?

Answer: (D)

37. Which venn diagram truly represents teachers, scholars and students?

Answer: (D)

38. How many triangles are there in the following square?

(A)  18

(B)  6

(C)  7

(D)  9

Answer: (A)

Directions-(Q. 39 and 40) One or two statements are given followed by two conclusions I and II.  You have to consider the statement to be true, even if it seems to be at variance from commonly known facts. You are to decide which of the given conclusions can definitely be drawn from the given statement. Indicate your answer.

39. Statements : (a) Some schools are Laboratories.

(b) All Laboratories are theatres.

Conclusions : (I) Some theatres are schools

(II) No schools is a theatre

(A)  neither conclusion (I) nor (II) follows

(B)  conclusion I(I) follows

(C)  conclusion (II) follows

(D)  conclusion (I) and (II) follows

Answer: (B)

40. Statements : Cactus plants are fleshy but need least water

Conclusions : (I) All fleshy plants are cactus.

(II) Cactus plants need least water.

(A)  Both (I) and (II) follow

(B)  Only (I) follows

(C)  Neither (I) or (II) follows

(D)  Only (II) follows

Answer: (D)

Directions-(Q. 41 and 42) Which answer figure will complete the pattern in the question figure?

41.

Answer: (B)

42.

Answer: (D)

43. A piece of paper is folded and cut as shown below in the question figures. From the given answer figures, indicate how will it appear when opened.

Answer: (B)

44. A circular sheet of paper is folded in a particular manner, punched once and then unfolded. Find out the manner in which the paper was folded and punched from amongst the answer figures.

Answer: (C)

45. From the given answer figures, select the one in which the question figure is hidden/embedded-

Answer: (D)

46. Select the answer figure which is the same as the questions figure.

Answer: (A)

47. Which is the correct image if the picture is held in front of a mirror?

Answer: (B)

48. If a mirror is placed on the line MN, then which of the answer figures is the right image of the given figure?

Answer: (C)

Directions-(Q. 49 and 50) A word is represented by only one set of numbers as given in any one of the alternatives. The sets of numbers given in the alternatives are represented by two classes of alphabets as in two matrices given below. The columns and rows of Matrix I are numbered from 0 to 4 and that of Matrix II are numbered from 5 to 9. A letter from these matrices can be represented first by its row and next by its column e.g., ‘E’ can be represented by 01, 13 etc., and ‘L’ can be represented by 56, 77 etc. Similarly, you have to identify the set for the word given in each question.

49.

(A)  24, 69, 56, 78

(B)  00, 68, 78, 88

(C)  12, 76, 99, 78

(D)  43, 55, 86, 95

Answer: (D)

50.

(A)  11, 66, 33, 96

(B)  11, 76, 32, 97

(C)  31, 87, 32, 97

(D)  31, 66, 33, 97

Answer: (C)

Part-B

General Knowledge and General Awareness

51. The market equilibrium for a commodity is determined by-

(A)  the intervention of the Government

(B)  the market demand of the commodity

(C)  the market supply of the commodity

(D)  the balancing of the forces of demand and supply for the commodity

Answer: (D)

52. A clone is a group of individuals obtained through-

(A)  micropropagation

(B)  hybridisation

(C)  cross pollination

(D)  self pollination

Answer: (A)

53. The forests which act as barriers against cyclones are-

(A)  Mangrove forests

(B)  Monsoon forests

(C)  Alpine forest

(D)  Evergreen forests

Answer: (C)

54. What does the word ‘amphibian’ mean?

(A)  Four lives

(B)  Three lives

(C)  One life

(D)  Two lives

Answer: (D)

55. Who among the following was named as “Haryana Hurricane”?

(A)  Nawab Pataudi

(B)  Kapil Dev

(C)  Ajay Jadeja

(D)  Ajay Ratsa

Answer: (B)

56. Regarding money supply situation in India, it can be said that the-

(A)  Currency with the public is inconvertible only

(B)  Currency with the public is more than the deposits with the banks

(C)  Currency with the public is less than the deposits with the blanks

(D)  Currency with the public is almost equal to the deposits

Answer: (C)

57. The largest forest which covers 25 per cent of the world’s Forest land is-

(A)  Temperature forests of Europe

(B)  The tropical Rain forest

(C)  Taiga forest of Siberia

(D)  Monsoon Forest

Answer: (B)

58. Cripps Mission came to India in-

(A)  1946

(B)  1940

(C)  1942

(D)  1945

Answer: (C)

59. In a computer system, which device is functionally opposite of a keyboard?

(A)  Trackball

(B)  Mouse

(C)  Printer

(D)  Joystick

Answer: (C)

60. To whom the President has to submit his resignation?

(A)  Chief Justice

(B)  Prime Minister

(C)  Vice President

(D)  Speaker

Answer: (C)

61. Reservation for the Scheduled Castes and Scheduled tribes in the Service has been provided in the Indian Constitution under-

(A)  Article 375

(B)  Article 315

(C)  Article 335

(D)  Article 365

Answer: (C)

62. The most abundant element is-

(A)  Calcium

(B)  Nitrogen

(C)  Oxygen

(D)  Silicon

Answer: (C)

63. If the Union Parliament is to assume legislative power over and subject included in the State List, the resolution to the effect has to be passed by which of the following?

(A)  Both Lok Sabha and Rajya Sabha

(B)  Loka Sabha

(C)  Rajya Sabha

(D)  Loka Sabha, Rajya Sabha and legislatures of the Concerned States

Answer: (C)

64. A low interest policy is also known as-

(A)  income generating policy

(B)  dear money policy

(C)  cheap money policy

(D)  investment policy

Answer: (C)

65. Nitrogen in water is commonly found in the form of-

(A)  Nitrous oxide

(B)  Nitrate

(C)  Nitrite

(D)  Nitric oxide

Answer: (B)

66. Name the Kingdom which first used elephants in wars?

(A)  Champa

(B)  Magadha

(C)  Kosala

(D)  Avanti

Answer: (B)

67. Immunization technique was developed by-

(A)  Robert Koch

(B)  Joseph Lister

(C)  Edward Jenner

(D)  Louis Pasteur

Answer: (C)

68. Interferons are synthesized in response to

(A)  Gungi

(B)  virus

(C)  bacteria

(D)  mycoplasma

Answer: (B)

69. Government of India has decided to integrate _________ with recent launched Pradhan Mantri Krishi Sinchayee Yojana.

(A)  Haryali

(B)  National Rural Livelihood Mission

(C)  Mahatma Gandhi National Rural Employment Guarantee Act

(D)  Accelerated Irrigation Benefit Programme

Answer: (C)

70. Who constructed Humayun’s Tomb in Delhi?

(A)  Babar

(B)  Humayun

(C)  Akbar

(D)  Haji Begam

Answer: (D)

71. Who among the following Sultans tried to prohibit Sati?

(A)  Firoz Tuglaq

(B)  Jalaluddin Khilji

(C)  Alauddin Khilji

(D)  Muhammad Bin  Tuglaq

Answer: (D)

72. The fleshy Thalamus is edible in-

(A)  Orange

(B)  Tomato

(C)  Apple

(D)  Mango

Answer: (C)

73. Aluminium salt commonly used to stop bleeding is-

(A)  Aluminium nitrate

(B)  Aluminium sulphate

(C)  Potash alum

(D)  Aluminium chloride

Answer: (C)

74. Who among the following Mughal rulers banned music and dancing?

(A)  Babur

(B)  Aurangzeb

(C)  Humayun

(D)  Jahangir

Answer: (B)

75. Which of the following is used in the treatment of cancer?

(A)  Psychotherapy

(B)  Chemotherapy

(C)  Physiotherapy

(D)  Electrotherapy

Answer: (B)

76. Who was the governor-general during the Second Anglo-Mysore War?

(A)  Sir John Shore

(B)  Lord Cornwallis

(C)  Warren Hastings

(D)  Lord Wellesley

Answer: (D)

77. Name the acid attack victim who was honoured with prestigious “International women of courage” award on 4 march, 2014 at the state department ceremony in Washington by Michelle Obama, first lady of the U.S.-

(A)  Shivani

(B)  Nirbhaya

(C)  Laxmi

(D)  Malini

Answer: (C)

78. ‘Economies’ of a firm are-

(A)  its dominance of the market

(B)  saving in its production costs

(C)  a reduction in its selling expenses

(D)  an increase in its profits

Answer: (B)

79. A reversible and an irreversible engines are working between the same limits of temperature. The efficiency of-

(A)  each engine is 100%

(B)  the two engines are equal

(C)  the irreversible engine is greater than the reversible engine

(D)  the reversible engine is greater than the irreversible engine

Answer: (D)

80. Which of the following is the major copper producing country?

(A)  Russia

(B)  Sri Lanka

(C)  Chile

(D)  Indonesia

Answer: (C)

81. The equilibrium price of a commodity will definitely rise if there is a/an-

(A)  increase in demand accompanied by a decrease in supply

(B)  decrease in both demand and supply

(C)  increase in both demand and supply

(D)  increase in supply combined with a decrease in demand

Answer: (A)

82. The state reorganization in 1956 created-

(A)  17States and 9 Union Territories

(B)  14 States and 6 Union Territories

(C)  15 States and 9 Union Territories

(D)  17 States and 6 Union Territories

Answer: (B)

83. The larges current, because of its dark waters is known as the “black current” and that is-

(A)  Antarctic current

(B)  Gulf Stream

(C)  Kuroshio current

(D)  California current

Answer: (C)

84. Chemical name of Gammaxaine is-

(A)  Toluene

(B)  Benzene hexachloride

(C)  Chloro benzene

(D)  Aniline

Answer: (B)

85. The resources which are obtained from biosphere and have life are-

(A)  Potential resources

(B)  Biotic resources

(C)  Abiotic resources

(D)  Renewable resources

Answer: (B)

86. Mac Iver says ‘kinship creates society and society at length creates ________.

(A)  City State

(B)  Nation

(C)  State

(D)  Association

Answer: (C)

87. Total internal reflection cannot take place when light goes from-

(A)  water to air

(B)  glass to air

(C)  glass to water

(D)  water to glass

Answer: (D)

88. In a solar or lunar eclipse, the region of earth’s shadow is divided into-

(A)  Five parts

(B)  Two parts

(C)  Three parts

(D)  Four parts

Answer: (C)

89. Which Article of the Indian Constitution defines the duties of the Chief Minister?

(A)  Article 166

(B)  Article 167

(C)  Article 163

(D)  Article 164

Answer: (B)

90. Which one of the following chapters in the Indian Constitution guarantees fundamental rights to the people?

(A)  Part IV

(B)  Part I

(C)  Part II

(D)  Part III

Answer: (D)

91. Red rot of sugarcane is caused by

(A)  Cercospora personata

(B)  Alternaria alternata

(C)  Phylophthora Infestans

(D)  Colletotrichum falcatum

Answer: (D)

92. The sense of balance is achieved by-

(A)  Thalamus equilibrium

(B)  Cerebrum equilibrium

(C)  Spinal cord equilibrium

(D)  Cerebellum equilibrium

Answer: (D)

93. The first law of thermodynamics is simply the case of-

(A)  the law of conservation of energy

(B)  the law of heat exchange

(C)  Newton’s law of cooling

(D)  Charle’s law

Answer: (A)

94. Tulsidas wrote Ramacharitamanas during the reign of-

(A)  Krishnadeva Raya

(B)  Jahangir

(C)  Rama  Raya

(D)  Akbar

Answer: (D)

95. An electron microscope gives higher magnifications than an optical microscope because-

(A)  The electron microscope uses more powerful lenses

(B)  The wavelength of electrons is smaller as compared to the wavelength of visible light

(C)  The velocity of electrons is smaller than that of light

(D)  The electrons have more energy than the light particles

Answer: (B)

96. The hydrophilic nature of DNA is due to the presence of-

(A)  phosphate group

(B)  deoxyribose sugar

(C)  thymine base

(D)  a number of hydrogen bonds

Answer: (D)

97. The total number f planets revolving around the sun are-

(A)  Eight

(B)  Five

(C)  Six

(D)  Seven

Answer: (A)

98. The Members of the Constituent Assembly which enacted the constitution of India were-

(A)  Nominated by the Political Parties

(B)  Elected by the Legislative Assemblies of various Provinces

(C)  Directly elected by the People

(D)  Nominated by Governor-General

Answer: (B)

99. RAM is used as a short memory in a computer because it-

(A)  is programmable

(B)  is volatile

(C)  has less capacity

(D)  is very expensive

Answer: (B)

100. The founder of the ‘Arya Samaj’ was

(A)  Raja Ram Mohan Roy

(B)  Vivekananda

(C)  Annie Besant

(D)  Dayananda Saraswati

Answer: (D)

Part-C

Quantitative Aptitude

101. 2 km 5 m is equal to-

(A)  2.0005 km

(B)  2.5 km

(C)  2.05 km

(D)  2.005 km

Answer: (D)

102. In a factory, the production of cycles rose to 48,400 from 40,000 in 2 years. The rate of growth per annum is-

(A)  9%

(B)  10.5%

(C)  10%

(D)  8%

Answer: (C)

103. The mean of x and 1/x is M. Then t he mean of x2 and 1/x2 is-

(A)  M2 – 2

(B)  4M2 – 2

(C)  2M2 – 1

(D)  M2

Answer: (C)

104. Incomes of x and y are in the ratio 4 : 3. Their expenditures are in the ratio 12 : 7. Both save Rs 3200 at the end of the month, then the income of x is-

(A)  Rs 8000

(B)  Rs 2000

(C)  Rs 4000

(D)  Rs 6000

Answer: (A)

105. The smallest whole number that is to be multiplied with 59535 to make a perfect square number is x. The sum of digits of x is-

(A)  6

(B)  5

(C)  7

(D)  9

Answer: (A)

106. 20 men can do piece of work in 18 days. They worked together for 3 days, then 5 men joined them. In how many more days is the work completed?

(A)  14

(B)  15

(C)  13

(D)  12

Answer: (D)

107. How many seconds will a train 120 metres long running at the rate of 36 km/hr take to cross a bride of 360 metres in length?

(A)  36 second

(B)  40 second

(C)  46 second

(D)  48 second

Answer: (D)

108. If a shopkeeper purchases cashew nut at Rs 250 per kg and sells it at Rs 10 per Rs 50 grams, then he will have-

(A)  20% profit

(B)  20% loss

(C)  25% loss

(D)  25% profit

Answer: (B)

109. A number is increased by x%, to get back to the original number, it is to be reduced by-

(A) 

(B)  x%

(C) 

(D) 

Answer: (A)

110. 

(A)  1/2

(B)  2

(C)  4

(D)  8

Answer: (B)

111. The height of a triangle is increased by 10%. To retain the original area of the triangle, its corresponding base must be decreased by-

(A) 

(B)  10%

(C) 

(D) 

Answer: (A)

112. A bus moving at 40 km per hours covers a distance in 6 hours 15 minutes. If it travels the same distance at 50 km per hour how long will it take to cover the distance?

(A)  2 hours

(B)  4 hours

(C)  5 hours

(D)  6 hours

Answer: (C)

113. The ratio of the ages of A and B at present are 3 : 1. Four years earlier the ratio was 4 : 1. The present age of A is-

(A)  32 years

(B)  36 years

(C)  48 years

(D)  40 years

Answer: (B)

114. A man bought a watch at 25% discount on the original price. He got Rs 40 more than the original price by selling it at 140% of the price at which he bought. The price of buying the watch was-

(A)  Rs 600

(B)  Rs 800

(C)  Rs 700

(D)  Rs 900

Answer: (B)

115. A fruit seller buys 240 apples for Rs 600. Some of these apples are bad and are thrown away. He sells the remaining apples at Rs 3.50 each and makes a profit of Rs 198. The % of apples thrown away are-

(A)  7%

(B)  6%

(C)  5%

(D)  8%

Answer: (C)

116. If 32x – y = 3x+y = √27, then the value of 3x – y will be-

(A)  1/√27

(B)  √3

(C)  3

(D)  1/√3

Answer: (B)

117. A trader marks his goods 20% above C.P. but allows his customers a discount of 10%. The C.P. of a blackboard, which is sold for Rs 216, is-

(A)  Rs 180

(B)  Rs 108

(C)  Rs 196

(D)  Rs 200

Answer: (D)

118. The average of some natural numbers is 15. If 30 is added to first number and 5 is subtracted from the last number the average becomes 17.5 then the number of natural number is-

(A)  30

(B)  15

(C)  10

(D)  20

Answer: (C)

119. The simplified value of  is-

(A)  599.6

(B)  0.5996

(C)  5.996

(D)  59.96

Answer: (C)

120. A certain amount of money earns Rs 540 as simple interest in 3 years. If it earns a compound interest of Rs 376.20 at the same rate of interest in 2 years, find the amount. (in rupees)

(A)  1600

(B)  1800

(C)  2000

(D)  2100

Answer: (C)

121. 12 monkeys can eat 12 bananas in 12 minutes. In how many minutes can 4 monkeys eat 4 bananas?

(A)  10

(B)  4

(C)  8

(D)  12

Answer: (D)

Directions-(Q. 122 to 125) Study the bar graph

122. The approximate percentage increase in quantity from 1997 to 1998 was-

(A)  27.8%

(B)  26.5%

(C)  26.9%

(D)  15.38%

Answer: (D)

123. Percentage fall in value from 2000 to 2001 is-

(A)  76%

(B)  25%

(C)  40%

(D)  50%

Answer: (A)

124. The difference between the bags exported in 1999 and 2000 was-

(A)  1,00,000,00

(B)  1,50,000,00

(C)  2,00,000,00

(D)  50,000,00

Answer: (A)

125. Value per bag was minimum in the year-

(A)  1999

(B)  1996

(C)  1997

(D)  2001

Answer: (B)

126. The total surface area of a regular triangular pyramid with each edges of length 1 cm is-

(A) 

(B)  4 sq. cm

(C)  4√3 sq. cm

(D)  √3 sq. cm

Answer: (D)

127. The shadow of a tower standing on a level plane is found to be 40 m longer when the sun’s altitude is 45° than when it is 60°. The height of the tower is-

(A)  40(3 + √3) m

(B)  10(3 + √3) m

(C)  20(3 + √3) m

(D)  30(3 + √3) m

Answer: (C)

128. If 999x + 888y = 1332

888x + 999y = 555

Then the value of x + y is-

(A)  1

(B)  999

(C)  555

(D)  888

Answer: (A)

129. From two points on the ground and lying on a straight line through the foot of a pillar, the two angles of elevation of the top of the pillar are complementary to each other. If the distances of the two points from the foot of the pillar are 12 metres ad 27 metres and the two points lie on the same side of the pillar, then the height (in metres) of the pillar is-

(A)  12

(B)  15

(C)  18

(D)  16

Answer: (C)

130.  If α is an acute angle and 2 sin α + 15cos2 α = 7, the value of cot α is-

(A)  5/4

(B)  4/3

(C)  3/4

(D)  4/5

Answer: (C)

131. The perimeters of two similar triangles are 30 cm and 20 cm respectively. If one side of the first triangle is 9 cm. Determine the corresponding side of the second triangle-

(A)  5 cm

(B)  6 cm

(C)  13.5 cm

(D)  15 cm

Answer: (B)

132. The number of paving stones each measuring 2.5 m × 2 m required to pave a rectangular courtyard 30 m long and 17.5 m wide, are-

(A)  105

(B)  33

(C)  80

(D)  99

Answer: (A)

133. The perimeter of sheet of paper in the shape of a quadrant of a circle is 75 cm. Its area would be (π = 22/7)-

(A)  100 cm2

(B)  693 cm2

(C)  512.25 cm2

(D)  346.5 cm2

Answer: (D)

134. O is the circumcentre of ∆ If ∠BAC = 50°, then ∠OBC is-

(A)  100°

(B)  50°

(C)  40°

(D)  130°

Answer: (A)

135. If the base of right prism remains same and the measures of the lateral edges are halved, then its volume will be reduced by-

(A)  25%

(B)  66%

(C)  33.33%

(D)  50%

Answer: (D)

136. The term that should be added to (4x2 + 8x) so that resulting expression be a perfect square is-

(A)  1

(B)  2x

(C)  2

(D)  4

Answer: (D)

137. Provided sin (A – B) = sin A cos B – cos A sin B, sin 15° will be-

(A) 

(B) 

(C) 

(D) 

Answer: (A)

138. If  , then the value of  is-

(A)  212

(B)  2

(C)  27

(D)  25

Answer: (B)

139. If in a triangle ABC, BE and CF are two medians perpendicular to each other and if AB = 19 cm and AC = 22 cm, then the length of BC is-

(A)  19.5 cm

(B)  13 cm

(C)  26 cm

(D)  20.5 cm

Answer: (B)

140. The angle between the graph of the linear eqeuation 239x – 239y + 5 = 0 and the x-axis is-

(A)  60°

(B)  0°

(C)  30°

(D)  45°

Answer: (D)

141. Two isosceles triangles have equal vertical angles and their areas are in the ratio 9 : 16. Then the ratio of their corresponding heights is-

(A)  8 : 4.5

(B)  3 : 4

(C)  4 : 3

(D)  4.5 : 8

Answer: (B)

142. The diagonal of a quadrilateral shaped field is 24 m and the perpendiculars dropped on it from the remaining opposite vertices are 8 m and 13 m. The area of the field is-

(A)  1152 m2

(B)  96 m2

(C)  156 m2

(D)  252 m2

Answer: (D)

143. If sin θ + sin2 θ = 1, then the value of cos2θ + cos4θ is-

(A)  0

(B)  1

(C)  −1

(D)  2

Answer: (B)

144. A man was born in the year 1896 A.D. If in the year x2D. his age is x – 4, the value of x is-

(A)  36

(B)  42

(C)  40

(D)  44

Answer: (D)

145. Two circles of radii 10 cm and 8 cm intersect and the length of the common chord is 12 cm. Then the distance between their centres is-

(A)  10 cm

(B)  8 cm

(C)  13.3 cm

(D)  15 cm

Answer: (C)

   Directions-(Q. 146-150) The expenses of a country for a particular year is given I the pie chart. Read the pie chart.

146. The per cent of less money spent on non-plan than that on defence is-

(A)  15%

(B)  5%

(C)  10%

(D)  12%

Answer: (B)

147. The per cent of the total spending that is spent on health is-

(A)  30%

(B)  15%

(C)  20%

(D)  25%

Answer: (B)

148. If the total amount spent by Government during the year was Rs 3,00,000 crores, the amount spent on state development exceeds that on sports by-

(A)  Rs 45,000 crores

(B)  Rs 35,000 crores

(C)  Rs 30,000 crores

(D)  Rs 25,000 crores

Answer: (C)

149. The per cent of more money spent on others than that on sports is-

(A)  25%

(B)  28%

(C)  27%

(D)  26%

Answer: (A)

150. If the total amount spent by the Government during the year was Rs 1,00,000 crores, the amount spent on health and education together was-

(A)  Rs 30,000 crores

(B)  Rs 15,000 crores

(C)  Rs 20,000 crores

(D)  Rs 25,000 crores

Answer: (D)

Part-D

English Comprehension

   Directions-(Q. 151 to 155) Some part of the sentences have errors and some are correct. Find out which part of a sentence has an error.

151. Every one were happy (A) / to hear (B)/ about his success. (C)/ No error (D)

Answer: (A)

152. The lions (A)/ kill the animals (B)/ and eat his (C)/ No error (D)

Answer: (C)

153. Hearing these strange noises above (A) / the thought at once occurred to me (B)/ that thieves had entered the house. (C)/ No error (D)

Answer: (B)

154. Our beloved Prime-Minister (A)/ is known to (B)/ the prominent figures all over the world. (C)/ No error (D)

Answer: (A)

155. The team was (A)/ now in the field and (B)/ about to take their place. (C)/ No error (D)

Answer: (C)

   Directions-(Q. 156 to 160) Sentences given with blanks are to be filled in with an appropriate word(s). Four alternatives are suggested for each question. Choose the correct alternative out of the four.

156. There were so many ________ points in his speech that the audience ________ to get the message.

(A)  relevant – hastened

(B)  interesting – tried

(C)  irrelevant – failed

(D)  superficial – listened

Answer: (C)

157. He was so __________ at his job that he was asked to give a ______ to the visitors.

(A)  able – disclosure

(B)  agile – demolition

(C)  accurate – display

(D)  adept – demonstration

Answer: (D)

158. She was so ________ that she was ________ by all.

(A)  excellent : warned

(B)  talented : praised

(C)  stupid : admired

(D)  proud : respected

Answer: (B)

159. The _______ of the ‘chief’ was _______ upon

(A)  name, dismissed

(B)  title, imposed

(C)  appointment, thrown

(D)  title, bestowed

Answer: (D)

160. The little boy ran ______ fast that he was _______ for breath.

(A)  too – fighting

(B)  so – inhaling

(C)  very – struggling

(D)  so – gasping

Answer: (D)

   Directions-(Q. 161 to 165) Out of the four alternatives, choose the one which best expresses the meaning of the given word and mark it in the Answer Sheet.

161. Quarry

(A)  denounce

(B)  victim

(C)  quake

(D)  quest

Answer: (B)

162. Verisimilitude

(A)  fantastic

(B)  grotesque

(C)  authenticity

(D)  festive

Answer: (C)

163. Conjecture

(A)  guess

(B)  truth

(C)  bias

(D)  knowledge

Answer: (A)

164. Annihilate

(A)  destroy

(B)  solve

(C)  deduce

(D)  initiate

Answer: (A)

165. Interference

(A)  deference

(B)  obstruction

(C)  fearful

(D)  honour

Answer: (B)

   Directions-(Q. 166 to 70) Choose the word opposite in meaning to the given word and mark in the Answer Sheet.

166. Vacillate

(A)  relieve

(B)  decide

(C)  repel

(D)  injure

Answer: (B)

167. Tranquil

(A)  temporal

(B)  unruffled

(C)  disturbed

(D)  perpetual

Answer: (C)

168. Enlarge

(A)  glorify

(B)  condense

(C)  augment

(D)  amplify

Answer: (B)

169. Unscrupulous

(A)  superfluous

(B)  dedicated

(C)  conscientious

(D)  single – minded

Answer: (C)

170. Obsequious

(A)  clever

(B)  opaque

(C)  careful

(D)  domineering

Answer: (D)

   Directions-(Q. 171 to 175) Four alternatives are given for the Idiom/Phrase bold in the sentence. Choose the alternative which best expresses the meaning of the Idiom/Phrase and mark it in the Answer Sheet.

171. We must draw the line

(A)  put the screw on

(B)  fix a limit

(C)  aim at the sky

(D)  turn the tables

Answer: (B)

172. To cut the Gordiann knot.

(A)  to cut a piece of cloth

(B)  to perform an easy task

(C)  to perform a difficult task

(D)  to do a job perfunctorily

Answer: (C)

173. The shopping mall is a beehive of activity.

(A)  a sweet place

(B)  a busy place

(C)  a costly place

(D)  a secure place

Answer: (B)

174. To win laurels.

(A)  To memorise

(B)  To be victorious in a game

(C)  To earn great prestige

(D)  To be disappointed

Answer: (C)

175. The officer was in the soup when there was a raid in his office.

(A)  distributing soup

(B)  to be in trouble

(C)  drinking soup

(D)  preparing soup

Answer: (B)

   Directions-(Q. 176 to 180) A part of the sentence is bold. Below are given alternatives to the bold part at (A), (B), (C) which may improve the sentence. Choose the correct alternative. In case no improvement is needed your answer is (D). Mark your answer in the Answer Sheet.

176. The workers should have been more

(A)  precise

(B)  punctual

(C)  committed

(D)  no improvement

Answer: (C)

177. She is very eccentric

(A)  unusual

(B)  impatient

(C)  generous

(D)  no improvement

Answer: (D)

178. I was angry at myself for making such a big mistake.

(A)  about

(B)  with

(C)  on

(D)  no improvement

Answer: (B)

179. The chickens in his farm are fatted up nicely.

(A)  are fattening

(B)  are fattying

(C)  are fattened

(D)  no improvement

Answer: (A)

180. Torture and trauma has made her a mental patient.

(A)  mad

(B)  mania

(C)  lunatic

(D)  no improvement

Answer: (C)

   Directions-(Q. 181 to 185) Out of the four alternatives, choose the one which can be substituted for the given words/sentences.

181. Walking in sleep-

(A)  somnolence

(B)  somnambulism

(C)  insomnia

(D)  sleepy-head

Answer: (B)

182. Showing a dislike of anything improper-

(A)  prim

(B)  strict

(C)  rude

(D)  crude

Answer: (B)

183. A person who leaves his own country in order to go and live in another-

(A)  refugee

(B)  immigrant

(C)  expatriate

(D)  emigrant

Answer: (D)

184. Artistic, musical or dramatic interpretation-

(A)  report

(B)  imitation

(C)  rendition

(D)  reparation

Answer: (C)

185. That which cannot be avoided-

(A)  indifferent

(B)  inestimable

(C)  infallible

(D)  inevitable

Answer: (D)

   Directions-(Q. 186 to 190) Four words are given in each question, out of which only one word is correctly spelt. Find the correctly spelt word and mark y our answer in the Answer Sheet.

186.

(A)  quriocity

(B)  curiousity

(C)  curioucity

(D)  curiosity

Answer: (D)

187.

(A)  antecedant

(B)  antecedent

(C)  entecedent

(D)  intecedent

Answer: (B)

188.

(A)  maditation

(B)  delibarate

(C)  aberrance

(D)  naseating

Answer: (C)

189.

(A)  encestral

(B)  ansastral

(C)  ancestral

(D)  ansestral

Answer: (C)

190.

(A)  cemetery

(B)  cemetary

(C)  cemetry

(D)  symetry

Answer: (A)

   Directions-(Q. 191 to 195) You have a passage with 5 questions. Read the passage carefully and choose the best answer to each question out of the four alternatives.

   Research is a detailed study of a subject undertaken on a systematic basis in order to increase the stock of knowledge, including knowledge of man, culture and society, and the use of this stock of knowledge to devise new applications. It is used to establish or confirm facts, re-affirm the results of previous work, solve new or existing problems, or develop new theories. To test the validity of instruments, procedures or experiments research may replicate elements of prior projects, or the project as a whole. The primary purposes of basic research are documentation, discovery, interpretation or the research and development of methods and systems for the advancement of human knowledge. There are several forms of research : scientific, humanities, artistic, economic, social, business, marketing etc.

   Academic publishing describes a system that is necessary in order for academic to review the work and make it available for a wider audience. Most academic work is published in book form. There is also a large body of research that exists in either a thesis or dissertation form. Many researchers spend their time applying for grants for research funds. These grants are necessary not only for researchers to carry out their research, but also as a source of merit.

191. What is research ?

(A)  Research is the creation of new forms of knowledge

(B)  Research is a process having no practical use

(C)  Research is the attempt to limit the growth of knowledge

(D)  Research is the destruction of previous works

Answer: (A)

192. Choose the most appropriate answer from this passage.

(A)  Academic publishing is meant only for professionals

(B)  Academic publishing is meant to benefit the general public

(C)  Academic publishing is meant only for experts

(D)  Academic publishing is meant only for academicians

Answer: (D)

193. Why is research conducted?

(A)  Research is conducted in order to destroy facts

(B)  Research is conducted in order to develop new problems

(C)  Research is conducted in order to verify information

(D)  Research is conducted in order to minimize the result of previous works

Answer: (C)

194. Select the answer which best reflect the view expressed in the passage.

(A)  Researchers never apply for grants

(B)  Research can thrive without grants

(C)  Documentation is important in research

(D)  Grants are not based on merit

Answer: (C)

195. How many kinds of research are there?

(A)  There are different kinds of research

(B)  There is only one kind of research

(C)  There are two different kinds of research

(D)  There are seven different kinds of research

Answer: (A)

   Directions-(Q. 196 to 200) In the following passage some of the words have been left out. Read the passage carefully and choose the correct answer to each question out of the four alternatives and fill in the blanks.

   A team of Indian engineers …. 196 …. Shown that digital photographs of the hand may … 197 … cholesterol levels in the body and serve as an early warning signal … 198 … poor lipid health without a standard blood test. Researchers believe the programme may be … 199 … in the future and … 200 … into mobile phones to enable it to analyse digital images of the hand captured in the camera on a mobile phone.

196.

(A)  Are

(B)  Have

(C)  Has

(D)  Has been

Answer: (C)

197.

(A)  Conjecture

(B)  Reveal

(C)  Conduct

(D)  Conceal

Answer: (B)

198.

(A)  Before

(B)  At

(C)  By

(D)  Of

Answer: (D)

199. 

(A)  Refined

(B)  Retracted

(C)  Recorded

(D)  Resolved

Answer: (A)

200. 

(A)  Featured

(B)  Loaded

(C)  Fixed

(D)  Formulated

Answer: (B)

SSC CAPFs-Delhi Sub-Inspectors Examination-2016 Paper-I Question Paper With Answer Key

SSC CAPFs-Delhi Sub-Inspectors Examination-2016 Paper-I
SSC CAPFs-Delhi Sub-Inspectors Examination-2016 Paper-I Question Paper With Answer Key

SSC CAPFs/Delhi Sub-Inspectors Examination-2016 Solved Papers

Part-A

General Intelligence & Reasoning

Directions-(Q. 1-6) Find the odd word/letters/number pair from the given alternatives.

1. Find the odd number pair-

(A)  183-54

(B)  99-63

(C)  108-97

(D)  243-132

Answer: (C)

2. Find the odd number-

(A)  495

(B)  583

(C)  286

(D)  166

Answer: (D)

3. Find the odd word-

(A)  Air Marshal

(B)  Wing Commander

(C)  Captain

(D)  Group captain

Answer: (C)

4. Find the odd letters-

(A)  D F I M

(B)  N P S W

(C)  K M P T

(D)  D F O U

Answer: (D)

5. Find the odd word-

(A)  Rectangle

(B)  Rhombus

(C)  Square

(D)  Kite

Answer: (C)

6. Find the odd letters-

(A)  L R X

(B)  I N U

(C)  A G M

(D)  M S Y

Answer: (B)

7. Sanjith faces towards north turning to his right, he walks 10 m. He then turn to left and walks 30 m. He moves 25 m to his right and again to right through 50 m. Finally he turns to right and moves 40 m. In which direction is he now from h is starting point?

(A)  South-East

(B)  North-East

(C)  South-West

(D)  North-West

Answer: (C)

Directions-(Q. 8-13) Select the related word/letters/number from the given alternatives.

8. Nurture : Neglect :: Denigrate : ?

(A)  Recognise

(B)  Reveal

(C)  Extol

(D)  Reply

Answer: (C)

9. 43 : 57 :: 111 : ?

(A)  136

(B)  135

(C)  133

(D)  134

Answer: (C)

10. Modesty : Arrogance :: ?

(A)  Speak : Lecture

(B)  Obese : Fat

(C)  Ship : Fleet

(D)  Debility : Strength

Answer: (D)

11. 23 : 0827 :: 45 : ?

(A)  64125

(B)  12025

(C)  12125

(D)  08125

Answer: (A)

12. AZBY : CXDW :: EVFU : ?

(A)  FUVE

(B)  BXWD

(C)  GTHS

(D)  ZYEU

Answer: (C)

13. ABDG : ZYWT :: BCEH : ?

(A)  YXVS

(B)  QPRU

(C)  YXTS

(D)  SVXY

Answer: (A)

14. Triangle represents school teachers. Square represents married persons. Circle represents persons living in joint families.

Married persons living in joint families but not working as school teachers are represented by-

(A)  C

(B)  D

(C)  F

(D)  A

Answer: (B)

Directions-(Q. 15 and 16) Which answer figure will complete the pattern in the question figure?

15. 

Answer: (A)

16.

Answer: (C)

17. Two statements are given, followed by two conclusions I and II. You have to consider the statements to be true even if they seems to be at variance from commonly known facts. You have to decide which of the given conclusions, if any follow from the given statements.

Statements :

(1) All Pens are Erasers.

(2) All Erasers are Pencils.

Conclusions:

(I) All Pencils are Pens.

(II) All Pens are Pencils.

(A)  Only conclusion I follows

(B)  Neither conclusion I nor II follows

(C)  Both conclusions I and II follows

(D)  Only conclusion II follows

Answer: (D)

   Directions-(Q. 18 and 19)  A word is represented by only one set of numbers as given in any one of the alternatives. The sets of numbers given in the alternatives are represented by two classes of alphabets as in two matrices given below. The columns and rows of Matrix I are numbered from 0 to 4 and that of Matrix II are numbered from 5 to 9. A letter from these matrices can be represented first by its row and next by its column, e.g., ‘A’ can be represented by 03, 44 etc, and ‘Q’ can be represented by 55, 78 etc. Similarly, you have to identify the set for the word given in questions.

18. RETAIL

(A)  20,77, 85, 21, 13, 57

(B)  77, 85, 21, 13, 57, 20

(C)  77, 21, 20, 85, 57, 13

(D)  20, 85, 77, 21, 57, 13

Answer: (D)

19. BIRDS

(A)  87, 75, 12, 68, 23

(B)  23, 75, 12, 87, 23

(C)  86, 12, 75, 23, 68

(D)  87, 12, 75, 68, 23

Answer: (A)

Directions (Q. 20 and 21) From the given alternative words, select the word which cannot be formed using the letters of the given word.

20. PERAMBULATOR

(A)  MARBLE

(B)  RAMPANT

(C)  LABOUR

(D)  RAMBLE

Answer: (B)

21. ORGANISATION

(A)  ORATION

(B)  ORGAN

(C)  ORGANISE

(D)  NATION

Answer: (C)

22. If 678 = 83, 476 = 75 and 567= 80, what is 369 = ?

(A)  40

(B)  99

(C)  18

(D)  72

Answer: (A)

   Directions-(Q. 23-26) Select the missing number from the given responses.

23. 

(A)  24

(B)  27

(C)  18

(D)  9

Answer: (D)

24. 

(A)  128

(B)  131

(C)  130

(D)  129

Answer: (C)

25. 

(A)  29

(B)  37

(C)  39

(D)  41

Answer: (B)

26.

16          25        48        8

2             5          3          6

8             25        36        16

4             5          4          ?

(A)  6

(B)  12

(C)  8

(D)  3

Answer: (D)

27. A square sheet of paper has been folded and punched as shown by the question figures. You have to figure out from amongst the four answer figures how it will appear when unfolded.

Answer: (D)

28. Which numbers complete the series?

3 1 5 3_5 9_11

(A)  7, 7

(B)  7, 6

(C)  8, 7

(D)  6, 6

Answer: (A)

   Directions-(Q. 29 and 30) If a mirror is placed on the line MN, then which of the answer figures in the right image of the given figure?

29. 

Answer: (B)

30.

Answer: (B)

31. In a peculiar mathematical operation, plus means multiplication, minus means plus, divided mean minus and multiplication means sum of digits of two numbers. Follow these rules and solve the following example-

(6 × 7) – (8 × 9) – (10 × 11) = ?

(A)  51

(B)  33

(C)  224

(D)  04

Answer: (A)

32. Which one of the given responses would be a meaningful order of the following?

(1) estimate        (2) contract

(3) concept         (4) plan

(5) execute

(A)  3, 1, 2, 4, 5

(B)  3, 4, 1, 2, 5

(C)  1, 4, 3, 5, 2

(D)  5, 2, 4, 3, 1

Answer: (A)

33. The structure shown in the figure is made of small cubes. First, third and fifth layers are made of black cubes and the others are white cubes. How many cubes are there in the structure?

(A)  25

(B)  75

(C)  45

(D)  95

Answer: (D)

   Directions-(Q. 34 and 35) A series is given, with one term missing. Choose the correct alternative from the given options that will complete the series.

34. 3, 17, 45, 87, ….?….

(A)  153

(B)  143

(C)  123

(D)  183

Answer: (B)

35. AZWD, CXUF, …..?…..GTQJ

(A)  EVSH

(B)  EUTH

(C)  EVPI

(D)  EUSH

Answer: (A)

36. Which of the following diagrams represents the correct relationship between Doctors, Specialist Doctors and Patients?

Answer: (B)

   Directions-(Q. 37 and 38) From the given answer figures, select the one of the which the question figure is hidden/embedded.

37.

Answer: (C)

38. 

Answer: (D)

39. A 7 metre long piece of cloth is cut into three smaller pieces. How long is the longest of the three pieces? Given that

Statements:

(I) One piece is 3.70 metre long.

(II) One piece is 1.50 metre longer than another piece and the remaining piece is 90 cm long.

(A)  Statement I alone is sufficient and also statement II alone is sufficient to answer the problem

(B)  Statement I alone but not Statement II alone is sufficient to answer the problem

(C)  Statement I and II both are needed to answer the problem

(D)  Statement II alone but not Statement I alone is sufficient to answer the problem

Answer: (A)

40. Grouping of words are given. Choose the next words in the series from the given options.

AFGHAN, INDEFINITLY, SYNOPSIS, STUPENDOUS, BURST

(A)  COMPANION

(B)  GLACIAL

(C)  RESCRIPT

(D)  HIJACKER

Answer: (D)

41. If BHASHA is coded as 154754, BRAIN is coded as 13408, AHINSA will be coded as-

(A)  450874

(B)  450847

(C)  405847

(D)  458074

Answer: (A)

42. In a certain language GAMBLE is coded as FBLCKF, how is FLOWER coded in that code?

(A)  GKPVFQ

(B)  GMPVDS

(C)  EMNXDS

(D)  EKNVDQ

Answer: (C)

43. A is elder to B while C and D are elder to E who lies between A and C. If C is elder to B, who is the youngest?

(A)  A

(B)  C

(C)  D

(D)  B

Answer: (D)

44. Mona is 6 years younger to her husband and he is 5 times as old as his daughter Rina. If Rina was 5 years old 3 years back, what is the present age of Mona?

(A)  34

(B)  30

(C)  35

(D)  40

Answer: (A)

45. Four position of a dice are given below. Identify the number at the bottom when top is 5.

(A)  2

(B)  3

(C)  1

(D)  4

Answer: (B)

46. The information collected through a survey conducted among the public is represented in the Venn diagram given below. Study the diagram and answer the question.

Which of the following represents Govt. servants who are tax payers, but are neither females nor graduates?

(A)  7

(B)  5

(C)  6

(D)  12

Answer: (A)

47. X is sister of Y . Y is brother of Z. Z is husband of P. O is father of Y. How is P related to O?

(A)  Uncle

(B)  Daughter

(C)  Sister

(D)  Daughter-in-law

Answer: (D)

48. Indu starts travelling towards east and stops after 35 km and turns right and walks another 15 km and again turns right and walk 15 km and then stops. What is the minimum distance Indu has to walk to reach the starting point?

(A)  25 km

(B)  30 km

(C)  35 km

(D)  20 km

Answer: (A)

49. A circular piece of paper is folded and punched as shown in the question figure. How will it appear when opened?

Answer: (C)

50. Which one set of letters when sequentially placed at the gaps in the given letter series shall complete it?

…qpx…rq…xxr…pxx…qp…x

(A)  pxrqxr

(B)  rsqprx

(C)  rxpqrx

(D)  xrprqx

Answer: (C)

Part-B

General Knowledge & General Awareness

51. Which one of the following Pacts sought to be resolved the Hindu-Muslim differences?

(A)  Gandhi-Irwin Pact

(B)  Poona Pact

(C)  Lucknow Pact

(D)  Lahore Pact

Answer: (C)

52. Who among the following was the Prime Minister of England when India was given independence?

(A)  Mountbatten

(B)  Churchill

(C)  Wavell

(D)  Attlee

Answer: (D)

53. Which of the following countries has become the first country in the world to receive funds from United Nations for its fast growing Solar Home Systems?

(A)  Burma

(B)  Afghanistan

(C)  Bangladesh

(D)  Switzerland

Answer: (C)

54. A new molecule Heat Shock Protein 90 (HSP90) was discovered in 2014 by the Didier Picard. The new discovery could help in effective treatment of…..

(A)  TB

(B)  Malaria

(C)  AIDS

(D)  None of these

Answer: (D)

55. A dispersion indicates-

(A)  the value of standard deviation

(B)  the value of mean

(C)  spread of data around central measure

(D)  the value of mode

Answer: (C)

56. Who among the following Indian writers have been long listed among the 13 international authors for the Man Booker Prize of 2015?

(A)  Anuradha Roy and Sanjeev Sahatta

(B)  Arvind Adiga and Shashi Tharoor

(C)  Chetan Bhagat and Jhumpa Lahiri

(D)  Amitav Ghosh and Amrita Pritam

Answer: (A)

57. The original jurisdiction of the Supreme Court covers-

(A)  Disputes between different units of the Indian Union

(B)  National Integrity of India

(C)  Terrorism related cases

(D)  Civil and Criminal cases

Answer: (A)

58. Antigen presenting cells are specialized cells present in all of the following, except-

(A)  Spleen

(B)  Lymph node

(C)  Skin

(D)  Kidney

Answer: (B)

59. An organic compound which produces bluish green coloured flame on heating in presence of copper is-

(A)  Benzaldehyde

(B)  Benzoic Acid

(C)  Aniline

(D)  Chlorobenzene

Answer: (D)

60. One strategy of allocating kernel memory is known as……… .

(A)  register allocation

(B)  resource allocation

(C)  frames allocation

(D)  slab allocation

Answer: (C)

61. In the absence of the president and Vice-President, who discharges the functions of the President?

(A)  Advocate General

(B)  Comptroller and Auditor General

(C)  Attorney General

(D)  Chief Justice of India

Answer: (D)

62. Which of the following is not be essential element of the State?

(A)  Government

(B)  Population

(C)  Sovereignty

(D)  Institutions

Answer: (D)

63. In a simple Keynesian model equilibrium is characterized by the equality of-

(A)  realized saving and realized investment

(B)  planned saving and realized investment

(C)  planned saving and planned investment

(D)  realized saving and planned investment

Answer: (C)

64. Political Science is a science of-

(A)  Cultural Engineering

(B)  Social Engineering

(C)  Economic Engineering

(D)  Political Engineering

Answer: (D)

65. The Vedic literature was composed between-

(A)  3500 B.C. and 1000 B.C.

(B)  3500 B. C. and 2500 B.C.

(C)  2500 B.C. and 500 B.C.

(D)  3000 B.C. and 1000 B.C.

Answer: (C)

66. In which type of vulnerability, an unauthorized host pretends to be authorized host by meeting some authorization criterion?

(A)  phishing

(B)  spoofing

(C)  tunnel

(D)  keystroke logger

Answer: (B)

67. Who of the following is not one of the recipients of Arjuna Award 2015?

(A)  P.R. Sreejesh (Hockey)

(B)  Anoop Singh ( Wrestling)

(C)  Dipa Karmakar (Gymnastics)

(D)  Jitu Rai(Shooting)

Answer: (B)

68. A closed vessel containing some gas at atmospheric pressure and room temperature is placed in a fast moving train. The temperature of the gas will-

(A)  increase or decrease depending on the chemical composition of the gas

(B)  remain unchanged

(C)  increase

(D)  decrease

Answer: (C)

69. The Rath temples at Mahabalipuram were built by ……. .

(A)  Chedis

(B)  Cholas

(C)  Chalukyas

(D)  Pallavas

Answer: (D)

70. Which of the following has become the first airport in the world to fully operational on Solar Power?

(A)  Amritsar International Airport

(B)  Cochin International Airport Ltd. (Kerala)

(C)  Netaji Subhash Chandra Bose International Airport

(D)  Lokpriya Gopinath Bardoli International Airport

Answer: (B)

71. The capsulte of Anthoceros differs from Funaria in that in the latter-

(A)  elaters are absent

(B)  columella is absent

(C)  elaters are present

(D)  columella is present

Answer: (A)

72. The concept of Liberty, Equality and Fraternity enshrined in the Indian Constitution is inspired from-

(A)  Irish Constitution

(B)  French Constitution

(C)  US Constitution

(D)  UK Constitution

Answer: (B)

73. If the pressure in a closed vessel is reduced by drawing out some gas, the mean free path of the gas molecules will-

(A)  increase

(B)  increase or decrease depending on the nature of the gas

(C)  decrease

(D)  remain unchanged

Answer: (A)

74. A plant with green leaves placed in red light will appear-

(A)  Black

(B)  Violet

(C)  Green

(D)  Red

Answer: (A)

75. Which type of molecular motion does not contribute towards internal energy?

(A)  None of these

(B)  Rotational

(C)  Vibrational

(D)  Translational

Answer: (D)

76. The minor irrigation schemes have Cultural Command Area (CCA) upto-

(A)  1500 hectares

(B)  2000 hectares

(C)  3000 hectares

(D)  2500 hectares

Answer: (B)

77. Viruses that infect bacteria are called-

(A)  Basal body

(B)  Basophils

(C)  Basidiospores

(D)  Bacteriophages

Answer: (D)

78. Which of the following Islands is under direct threat of rising sea levels due to climate change has appealed for help from European leaders?

(A)  Nauru

(B)  Tuvalu Islands

(C)  Palau

(D)  Marshall Islands

Answer: (B)

79. In terms of value, India’s export in Descending order are-

(A)  Textiles, Gems and Jewellery, Engineering goods

(B)  Gems & Jewellery, Textiles, Engineering goods

(C)  Textiles, Engineering Goods, Gems and Jewellery

(D)  Engineering goods, Gems and Jewellery, Textiles

Answer: (D)

80. The lethal does required to kill 50% of the lab animals tested under standard is referred as-

(A)  LD50

(B)  ID50

(C)  MLD

(D)  ID

Answer: (A)

81. Comptroller and Auditor General of India is not responsible for-

(A)  Regulating Reserve Bank of India

(B)  Issue of money from the Consolidated Fund

(C)  Accounting of States & UTs, and Auditing of all Government Institutions

(D)  Making payments to the States

Answer: (A)

82. Which of the following determines whether a group of organisms that is from the same genus and species arise from a common source or from different sources?

(A)  Biotyping

(B)  Phage typing

(C)  DNA hybridization

(D)  Serotyping

Answer: (C)

83. The method of age determination called “radioactive uranium dating” cannot be used to determine the age of-

(A)  minerals on earth

(B)  fossils and plant bodies

(C)  rocks

(D)  the earth

Answer: (B)

84. How many times did Babur invade India before 1526 A.D.?

(A)  Five times

(B)  Two times

(C)  None of these

(D)  Four times

Answer: (D)

85. The directive principles of state policy of the Indian Constitution is an idea borrowed from the Constitution of-

(A)  Australian

(B)  Ireland

(C)  Russian

(D)  American

Answer: (B)

86. Demand curve is indeterminate under-

(A)  monopoly

(B)  oligopoly

(C)  duopoly

(D)  pure competition

Answer: (B)

87. Altocumulus clouds occur at altitude of-

(A)  1000 m – 1800 m

(B)  2000 m – 6000 m

(C)  10000 m – 12000 m

(D)  6000 m – 10000 m

Answer: (B)

88. Professor Ramesh Chand has been appointed as new full time member of National Institution for Transforming India (NITI) Aayog. He is a-

(A)  Renowned English Professor

(B)  None of these

(C)  Renowned Agriculture Expert

(D)  Renowned Physics Professor

Answer: (C)

89. Basic problems of an economy is/are-

(A)  Decide as to what, how and for whom to produce

(B)  Providing Social Security and employment to all

(C)  Providing basic requirements of life to all

(D)  Elimination of poverty and reduction of inequalities of income and wealth

Answer: (A)

90. Parliamentary Government is a form of Constitutional democracy in which-

(A)  the legislature emerge from and is responsible to the executive

(B)  the legislatures emerge from and is responsible to the judiciary

(C)  the executives emerge from and is responsible to the judiciary

(D)  the executive emerge from and is responsible to the legislature

Answer: (D)

91. U-235 belongs to which member of the series?

(A)  Actinium series

(B)  Uranium series

(C)  Thorium series

(D)  Neptunium series

Answer: (B)

92. The book “Globalization Democratization and Distributive Justice” has been authored by-

(A)  Karan Bajaj

(B)  Mool Chand Sharma

(C)  Anita Nair

(D)  Salman Rushdies

Answer: (B)

93. The principle involved in the absorption of water by soil is-

(A)  Suction action

(B)  Capillary action

(C)  Principle of absorption

(D)  Condensation

Answer: (B)

94. Which of the following is present in maximum amount in acid rain?

(A)  H2CO3

(B)  HCl

(C)  H2SO4

(D)  HNO3

Answer: (A)

95. In which of the following year the Gandhi-Irwin Pact was signed?

(A)  1931

(B)  1932

(C)  1935

(D)  1929

Answer: (A)

96. Which of the following Football teams won UEFA Super Cup of Georgia on August 12, 2015?

(A)  Atlatico Madrid

(B)  Sevilla

(C)  Real Madrid

(D)  Barcelona

Answer: (D)

97. If the selling price of a product under perfect competition is Rs 25, the Marginal Revenue will be-

(A)  less than Rs 25

(B)  more than Rs 25

(C)  zero

(D)  equal to Rs 25

Answer: (D)

98. Starch is insoluble in water but still it is stored in large quantity in potato because-

(A)  it is synthesized in potato root

(B)  soil microorganisms deposit it in the tuber

(C)  it is translocated in the form of sugar from leaves

(D)  it is useful for human

Answer: (A)

99. Marginal Revenue is-

(A)  Revenue realized on every unit sold

(B)  The average revenue of a firm

(C)  Revenue realized on the sale of an extra units

(D)  Revenue realized from the sale of all units

Answer: (C)

100. In Indian architecture ‘Surkhi’ was introduced by-

(A)  Guptas

(B)  Kushans

(C)  Sultanate Sultans

(D)  Mughals

Answer: (C)

Part-C

Quantitative Aptitude

101. A trader lists his article 20% above the cost price and allows a discount of 10% on cash payment. His gain per cent is-

(A)  6%

(B)  5%

(C)  8%

(D)  10%

Answer: (C)

102. A chord of length 24 cm is at a distance of 5 cm from the centre of a circle. The length of the chord of the same circle which is at a distance of 12 cm from the centre is-

(A)  12 cm

(B)  11 cm

(C)  17 cm

(D)  10 cm

Answer: (D)

   Directions-(Q. 103-106) The bar graph shows the results of an annual examination in a secondary school in a certain year. Answer the following questions based on this chart.

103. The class having the highest number of passed students, is-

(A)  VII

(B)  IX

(C)  VIII

(D)  X

Answer: (D)

104. The average number of boys passed per class is-

(A)  72

(B)  70

(C)  75

(D)  78

Answer: (B)

105. The ratio of the total number of boys passed to the total number of girls passed in the three classes VII, VIII and IX is-

(A)  20 : 23

(B)  19 : 25

(C)  18 : 21

(D)  21 : 26

Answer: (A)

106. The class in which the number of boys passed is nearest to the average number of girls passed per class, is-

(A)  VIII

(B)  X

(C)  IX

(D)  VI

Answer: (D)

107. A sum of Rs x was put at simple interest at a certain rate for 2 years. Had it been put at 3% higher rate, it would have fetched Rs 300 more. The value of 4x is-

(A)  Rs 16,000

(B)  Rs 24,000

(C)  Rs 20,000

(D)  Rs 36,000

Answer: (C)

108. If 3x+y = 81 and 81x – y = 3, then the value of x/y is=

(A)  17/15

(B)  15/34

(C)  15/17

(D)  17/30

Answer: (A)

109. The average monthly income of A and B is Rs 15,050, the average monthly income of B and C is Rs 15,350 and the average income of A and C is Rs 15,200. The monthly income of A is-

(A)  Rs 14,900

(B)  Rs 15,500

(C)  Rs 15,900

(D)  Rs 15,200

Answer: (A)

110. If p, q, r are all real numbers, then (p – q)3 + (q – r)3 + (r – p)3 is equal to-

(A)  (p – q) (q – r) (r – p)

(B)  1

(C)  3(p – q) (q – r) (r – p)

(D)  0

Answer: (C)

111. A number is increased by 15% and then decreased by 25% and the number becomes 22 less than the original number. The original number is-

(A)  140

(B)  160

(C)  100

(D)  120

Answer: (B)

112. By selling an article at 3/4 of selling price, a trader incurred a loss of 10%. The profit/loss percentage, when it is sold at the original selling price, is-

(A)  20% profit

(B)  120% profit

(C)  20% loss

(D)  32.5% loss

Answer: (A)

113. AB and AC are two chords of a circle. The tangents at B and C meet at P. If ∠BAC = 54°, then the measure of ∠BPC is-

(A)  72°

(B)  108°

(C)  54°

(D)  36°

Answer: (A)

114. The average age of a class is 15.8 years. The average age of the boys in the class is 16.4 years while that of the girls is 15.4 years. The ratio of boys to girls in the class is-

(A)  3 : 4

(B)  2 : 3

(C)  1 : 2

(D)  3 : 5

Answer: (B)

115. If 0 < A < 90°, then the value of  is-

(A)  1/2

(B)  2

(C)  1

(D)  0

Answer: (C)

116. If  where x ≠ 0, then the value of  is-

(A)  7

(B)  2

(C)  3

(D)  5

Answer: (C)

117. A man travelled a distance of 72 km in 12 hours. He travelled partly on foot at 5 km/hour and partly on bicycle at 10 km/hour. The distance travelled on foot is-

(A)  46 km

(B)  50 km

(C)  48 km

(D)  52 km

Answer: (C)

118. If 4 men and 6 women can complete a work in 8 days, while 3 men and 7 women can complete it in 10 days, then 10 women will complete it in-

(A)  40 days

(B)  45 days

(C)  50 days

(D)  35 days

Answer: (A)

119. A can is full of a mixture of two liquids A and B in the ratio of 7 : 5. When 9 litres of mixture are drawn off from the can and replaced by the same quantity of liquid B, the ratio of A and B in the can becomes 7 : 9. The capacity of the can is-

(A)  10 litres

(B)  20 litres

(C)  21 litres

(D)  36 litres

Answer: (D)

120. A sum of money placed at compound interest double itself at 2 y ears. The year it will take to amount 4 times itself is-

(A)  3

(B)  4

(C)  6

(D)  8

Answer: (B)

121. From the top and bottom of a straight hill, the angle depression and elevation of the top of a pillar of 10 m. height are observed to b 60° and 30° The height (in metres) of the hill is-

(A)  30

(B)  60

(C)  40

(D)  80

Answer: (C)

   Directions-(Q. 122-126) Study the following table which shows the amount of money invested (Rupees in crore) in the core infrastructure areas of two districts. A and B of a State, and answer the given question.

122. By approximately what per cent was the total investment in the ratio districts A and B more in 1996 as compared to 1995?

(A)  14%

(B)  21%

(C)  24%

(D)  18%

Answer: (D)

123. The total investment in electricity and thermal energy in 1995, in these two districts A and B formed approximately what per cent of the total investment made in that year?

(A)  55%

(B)  41%

(C)  52%

(D)  47%

Answer: (D)

124. In district B, the investment in which area in 1996 did show the highest percentage increase over the investment in that area in 1995?

(A)  Electricity

(B)  Solar

(C)  Nuclear

(D)  Chemical

Answer: (D)

125. Approximately how many times was the total investment in 1995 and 1996 in district B was that of total investment of district A in the same years?

(A)  2.8

(B)  1.9

(C)  2.4

(D)  1.7

Answer: (C)

126. If the total investment in district B shows the same rate of increase in 1997, as it had shown from 1995 to 1996, what approximately would be the total investment in B in 1997?

(A)  Rs 9,170 crore

(B)  Rs 8,540 crore

(C)  Rs 9,850 crore

(D)  Rs 10,020 crore

Answer: (C)

127. A man sells an article at 5% above the cost price. If he had bought it at 5% less than what he paid for it and sold it for Rs 2 less, he would have gained 10%. The cost price of the article is-

(A)  Rs 250

(B)  Rs 400

(C)  Rs 200

(D)  Rs 350

Answer: (B)

128. If  then each of them is equal to-

(A) 

(B) 

(C) 

(D) 

Answer: (D)

129. The radii of the base of a cylinder and a cone are equal and their volumes are also equal. Then the ratio of their height is-

(A)  1 : 4

(B)  2 : 1

(C)  1 : 2

(D)  1 : 3

Answer: (D)

130. If x + 1 = √y+ 3, y > 0, then the value of  is-

(A)  1/2

(B)  0

(C)  1

(D)  −1

Answer: (B)

131. PQRS is a cyclic quadrilateral, such that ratio of measures of ∠P, ∠Q and ∠R is 1 : 3 : 4, then the measures of ∠S is-

(A)  144°

(B)  36°

(C)  72°

(D)  108°

Answer: (C)

132. In a ∆ABC a straight line parallel to BC intersects the sides AB and AC at P and Q respectively such that AP : PB = 3 : 2, then the ratio of the area of ∆APQ : ∆ABC is-

(A)  4 : 9

(B)  25 : 4

(C)  9 : 4

(D)  9 : 25

Answer: (D)

133. If a person spends 40% of his income on food, 20% on house rent and 70% of the remaining on children’s education, then the percentage of his income left is-

(A)  8%

(B)  10%

(C)  12%

(D)  6%

Answer: (C)

134. The value of (√6 + √10 – √21 – √35) (√6 – √10 + √21 – √35) is-

(A)  27

(B)  10

(C)  18

(D)  40

Answer: (B)

135. The sides of a triangle are 7 cm, 8 cm, 9 cm, then the area of the triangle (in cm2) is-

(A)  24√5

(B)  12√5

(C)  6√5

(D)  30√5

Answer: (B)

136. If tan θ + sec θ = 2, then the value of tan θ is-

(A)  2/3

(B)  3/5

(C)  3/4

(D)  4/5

Answer: (C)

137. The value of  is-

(A)  0.03

(B)  0.02

(C)  0.01

(D)  0.04

Answer: (B)

138. The length of the diagonal BD of the parallelogram ABCD is 12 cm. P and Q are the centroids of the ∆ABC and ∆ADC respectively. The length (in cm) of the line segment PQ is-

(A)  5

(B)  6

(C)  4

(D)  3

Answer: (C)

139. Let a be a positive integer. When 89 and 125 are divided by a, the remainders are 4 and 6 respectively. Then the value of a is-

(A)  15

(B)  17

(C)  9

(D)  7

Answer: (B)

140. Points A and B are 100 km apart on a highway. One car starts from A and another from B at the same time. If the cars travel in the same direction, they meet in 5 hours. If the cars travel towards each other, they meet in 1 hours. What is the speed of the faster car?

(A)  80 km/hour

(B)  40 km/hour

(C)  70 km/hour

(D)  60 km/hour

Answer: (D)

141. A cylindrical rod of radius 30 cm and length 40 cm is melted and made into spherical balls of radius 1 cm. The number of spherical balls is-

(A)  40000

(B)  27000

(C)  36000

(D)  90000

Answer: (B)

142. If x = 1 + √2 + √3 and y = 1 + √2 – √3, then the value of  is-

(A)  6

(B)  2(1 + √2)

(C)  1

(D)  2√2

Answer: (A)

143. A merchant purchases a wrist watch for Rs 1,200 and fixes its list price in such a way that after allowing a discount of 10%, he earns a profit of 20%. The list price of the watch is-

(A)  Rs 1,400

(B)  Rs 1,600

(C)  Rs 1,800

(D)  Rs 1,200

Answer: (B)

144. The sum of two numbers is  and their difference is  The product of the numbers is-

(A) 

(B)  60

(C) 

(D)  50

Answer: (A)

145. A and B can separately finish a piece of work in 20 days and 15 days respectively. They worked together for 6 days, after which B was replaced by C. If the work was finished in next 4 days, then the number of days in which C alone could do the work is-

(A)  40 days

(B)  60 days

(C)  30 days

(D)  50 days

Answer: (A)

146. The value of sin22° + sin24° + sin26° + …… + sin2 90° is-

(A)  44

(B)  0

(C)  22

(D)  23

Answer: (D)

147. The curved surface area of a cylinder with its height equal to the radius, is equal to the curved surface area of a sphere. The ratio of volume of the cylinder to that of the sphere is-

(A)  √2 : 3

(B)  3 : √2

(C)  2√2 : 3

(D)  3 : 2√2

Answer: (B)

148. If  then the value of a + b + c is –

(A)  6

(B)  9

(C)  3

(D)  12

Answer: (A)

149. The ratio of the radii of two cylinders is 2 : 1 and their heights are in the ratio 3 : 2. Then their volumes are in the ratio-

(A)  6 : 1

(B)  4 : 3

(C)  6 : 5

(D)  3 : 1

Answer: (A)

150. If 0 < θ < 90°, tan θ + sin θ = m and tan θ – sin θ = n where m ≠ n, then the value of m2 – n2 is-

(A)  4mn

(B)  2(m2 + n2)

(C)  2(tan2 θ + sin2 θ)

(D)  4√mn

Answer: (D)

Part-D

English Comprehension

   Directions-(Q. 151 to 155) Choose the word opposite in meaning to the given word and mark it in the answer sheet.

151. Fortify

(A)  strengthen

(B)  support

(C)  undermine

(D)  intensify

Answer: (C)

152. Misanthropist

(A)  philanthropist

(B)  zealot

(C)  pragmatist

(D)  pedant

Answer: (A)

153. Unapproachable

(A)  unclear

(B)  accessible

(C)  casual

(D)  withdrawn

Answer: (B)

154. Ostracize

(A)  crucify

(B)  discard

(C)  patronize

(D)  shun

Answer: (C)

155. Abstain

(A)  repel

(B)  indulge

(C)  disgrace

(D)  dismiss

Answer: (B)

   Directions-(Q. 156 to 160) Four words are given in the each question, out of which only one word is correctly spelt. Find the correctly spelt word and mark your answer in the answer sheet.

156. Find the correctly spelt word.

(A)  Conteinment

(B)  Confinment

(C)  Conceilment

(D)  Consignment

Answer: (D)

157. Find the correctly spelt word.

(A)  Obeisence

(B)  Perservance

(C)  Turbulance

(D)  Surveillance

Answer: (D)

158. Find the correctly spelt word.

(A)  Congrruous

(B)  Auspiscious

(C)  Audacious

(D)  Hillarious

Answer: (C)

159. Find the correctly spelt word.

(A)  Dysfunction

(B)  Dastitution

(C)  Divienation

(D)  Divarsion

Answer: (A)

160. Find the correctly spelt word.

(A)  Hygeinic

(B)  Hygienic

(C)  Hyigeinic

(D)  Hyegienic

Answer: (B)

   Directions-(Q. 161 to 165) Some parts of the sentences have errors and some are correct. Find out which part of a sentence has an error and blacken the circle corresponding to the appropriate option. If a sentence is free from error, blacken the circle corresponding to ‘No Error’ in the answer sheet.

161. The car that he is using these days is belonging to his employer-

(A)  The car that he is using

(B)  to his employer

(C)  these days is belonging

(D)  No error

Answer: (C)

162. Three years have elapsed since I had gone to visit my aunt in the city-

(A)  visit my aunt in the city

(B)  since I had gone to

(C)  No error

(D)  Three years have elapsed

Answer: (B)

163. I reached two hours before he had came-

(A)  he had came

(B)  I reached

(C)  two hours before

(D)  No error

Answer: (A)

164. He needs not have shouted at me that way-

(A)  at me that way

(B)  not have shouted

(C)  No error

(D)  He needs

Answer: (D)

165. He assured me that he will return in an hour-

(A)  in an hour

(B)  that he will return

(C)  No error

(D)  He assured me

Answer: (C)

   Directions-(Q. 166 to 170) Read the passage carefully and choose the best answer to each question out of the four alternatives and mark it in the answer sheet.

Passage

   The destructive process of Mountain Top Removal mining (MTR) has caused permanent damage to Appalachia. Although the law requires that mining companies restore the mountaintops after the mining has been completed, the 1.5 million acres of mountains that have already been removed cannot be re-grown, re-built, or replaced. The companies do secure the rock formations to prevent erosion and landslides, but their efforts cannot recreate the once beautiful mountain landscape. Furthermore, while companies are usually vigilant about securing the rock formations, they seem less interested in restoring the native vegetation. MTR operations clear enormous tracts of forest. Environmental hazards are not only created in preparing a mountaintop for mining, they also continue once the coal has been extracted. After the blast, the excess mountaintop –which miners refer to as ‘overburden’ – is usually dumped into nearby valleys or streams. The overburden contains a variety of toxic substances, including explosive residue, silica and coal dust.

166. After the MTR operation, the mining companies-

(A)  beautify the mountains

(B)  replace the mountaintops

(C)  secure rock formations to prevent erosion

(D)  restore native vegetation

Answer: (C)

167. The word opposite in meaning to ‘Vigilant’ is-

(A)  careless

(B)  annoyed

(C)  displeased

(D)  lenient

Answer: (A)

168. In the Appalachian region MTR has caused-

(A)  landslides

(B)  permanent beautification

(C)  floods

(D)  widespread damage

Answer: (D)

169. The term “overburden” means-

(A)  weeds planted by mining companies

(B)  excess mountaintop left after the extraction of coal through the blast

(C)  debris from landslides

(D)  remnants of natural forests

Answer: (B)

170. MTR operations cause environmental hazards because-

(A)  it causes landslides

(B)  it causes explosion

(C)  it destroys natural vegetation

(D)  mountaintops dumped in valleys & streams contain toxic substance

Answer: (D)

   Directions-(Q. 171 to 175) A sentence/a part of the sentence is underlined. Below are given alternatives to the underlined part which may improve the sentence. Choose the correct alternative. In case no improvement is required, choose “No Improvement” option.

171. Try to lower the amount of fat in your diet-

(A)  Lose

(B)  No improvement

(C)  Cut

(D)  Reduce

Answer: (D)

172. You should cut off on the amount of cigarettes you smoke-

(A)  cut out

(B)  cut down

(C)  No improvement

(D)  cut up

Answer: (B)

173. You must accustom yourself with new ideas-

(A)  No improvement

(B)  accustomed yourself with

(C)  get accustom to

(D)  accustom yourself to

Answer: (D)

174. I contradicted against him-

(A)  No improvement

(B)  contradicted with

(C)  contradicted

(D)  contradicted over

Answer: (C)

175. No sooner than it stopped raining, the children went out-

(A)  No improvement

(B)  had it stopped raining than

(C)  did it stopped raining than

(D)  did it stop raining than

Answer: (D)

   Directions-(Q. 176 to 180) Out of the four alternatives, choose the one which best expresses the meaning of the given word and mark it in the answer sheet.

176. Synopsis

(A)  preview

(B)  discussion

(C)  summary

(D)  report

Answer: (C)

177. Contentious

(A)  extravagant

(B)  precious

(C)  benevolent

(D)  controversial

Answer: (D)

178. Conjecture

(A)  strife

(B)  guess

(C)  critic

(D)  gathering

Answer: (B)

179. Brazen

(A)  shameless

(B)  trashy

(C)  modest

(D)  melodramatic

Answer: (A)

180. Jinx

(A)  spell

(B)  taunt

(C)  juncture

(D)  tour

Answer: (A)

   Directions-(Q. 181 to 185) Out of the four alternatives, choose the one which can be substituted for the given words/sentences and indicate it by blackening the appropriate circle in the answer sheet.

181. Scientific study of Earthquakes-

(A)  Geography

(B)  Seismology

(C)  Astrology

(D)  Anthropology

Answer: (B)

182. The study of growing garden plants-

(A)  nomenclature

(B)  nursery

(C)  orchard

(D)  horticulture

Answer: (D)

183. One who copies from other writers-

(A)  Offender

(B)  Plagiarist

(C)  Contender

(D)  Antagonist

Answer: (B)

184. The belief that God is in everything, including nature-

(A)  Naturalism

(B)  Pantheism

(C)  Polytheism

(D)  Mysticism

Answer: (B)

185. Gradually advanced-

(A)  evaded

(B)  evoluted

(C)  advantaged

(D)  evolved

Answer: (D)

   Directions-(Q. 186-190) Read the passage carefully and choose the best answer to each question out of the four alternatives and mark it in the answer sheet.

Passage

   Flattery means ‘praising insincerely in order to please’. Every flatterer says words in which he himself does not believe. While flattering he is insincere to the man he is praising and to himself. In doing so he does not  mind if he corrupts the minds of those whom he flatters.

   Flattery is immoral because it stains the human conscience. It creates a world of falsehood and thus an outrage of man’s sense of decency and gentlemanly behaviour.

   A man who feels happy when flattered lives in a fool’s paradise. Flattery is the ready weapon of the opportunist. This weapon easily conquers the weak willed man. It work on the general weakness of human beings. We all love to be told what we are not rather than what we are. Flattery is equally bad for him who is flattered and for him who flatters.

   Flattery deceives us by giving us false notions about ourselves. By falling a victim to it, we show lack of character. By accepting flattery we make ourselves small beings. It is an evil which ruins social and moral values by claiming what is not rightfully its own. It thrives on corruption and leads to human bankruptcy. It is thus the greatest of disease which can plague humanity.

186. How does the weapon of flattery work?

(A)  A man does not like it

(B)  It conquers the man with a weak will

(C)  A man feels sad

(D)  It conquers the man with a strong will

Answer: (B)

187. ‘Thrives’ in the passage means-

(A)  collects

(B)  provides

(C)  fills

(D)  prospers

Answer: (D)

188. How does flattery deceive us?

(A)  It gives us false ideas about ourselves

(B)  It makes us feel indecent

(C)  It makes use bankrupt

(D)  It makes us more corrupt

Answer: (A)

189. Flattery can stain the-

(A)  human conscience

(B)  heart

(C)  emotion

(D)  mind

Answer: (A)

190. Flattery means-

(A)  insincere praise in order to please

(B)  claiming what is not ours

(C)  being anti-social

(D)  being immoral

Answer: (A)

   Directions-(Q. 191-195) Four alternatives are given for the Idiom/Phrase underlined. Choose the alternative which best expresses the meaning of the Idiom/Phrase and mark it in the answer sheet.

191. To roll out the red carpet-

(A)  to give a warning signal

(B)  to give a grand welcome

(C)  to buy a gift

(D)  to decorate the room

Answer: (B)

192. To have an axe to grind-

(A)  To have a selfish end to serve

(B)  To fail to arouse interest

(C)  To criticize someone

(D)  To work for both sides

Answer: (A)

193. To put his foot down-

(A)  Withdraw

(B)  Concede

(C)  Not to yield

(D)  Risign

Answer: (C)

194. Have a foot in the grave-

(A)  be afraid to die

(B)  be close to death

(C)  have not interest in life

(D)  have an incurable disease

Answer: (B)

195. A hornet’s nest-

(A)  a dilemma

(B)  among thorns

(C)  a comfortable position

(D)  an unpleasant situation

Answer: (D)

   Directions-(Q. 196-200) Sentences given with blanks to be filled in with an appropriate word(s). Four alternatives are suggested for each question. Choose the correct alternative out of the four and indicate it by blackening the appropriate circle in the answer sheet.

196. The demonstration passed off without……..

(A)  incident

(B)  incidence

(C)  accident

(D)  coincidence

Answer: (B)

197. As the driver swerved violently at the turning, the wheel came off, as it was already…….

(A)  lost

(B)  loose

(C)  loss

(D)  lose

Answer: (B)

198. His profession is teaching but his ……… is photography.

(A)  vocation

(B)  vocative

(C)  vacation

(D)  avocation

Answer: (D)

199. He quarreled with his boss,……..of the consequences.

(A)  heedless

(B)  devoid

(C)  weary

(D)  confident

Answer: (A)

200. Some verbs need a …… to convey the full meaning.

(A)  complement

(B)  component

(C)  complimentary

(D)  compliment

Answer: (A)

SSC CAPFs-Delhi Sub-Inspectors Examination-2017 Paper-I Question Paper With Answer Key

SSC CAPFs-Delhi Sub-Inspectors Examination-2017 Paper-I
SSC CAPFs-Delhi Sub-Inspectors Examination-2017 Paper-I Question Paper With Answer Key

SSC CAPFs/Delhi Sub-Inspectors Examination-2017 Paper-I Solved Papers

Part-A

General Intelligence & Reasoning

Directions-(Q. 1-6) Find the odd word/letters/numbers from the given alternatives

1.

(A)  Yacht

(B)  Submarine

(C)  Boat

(D)  Ship

Answer: (B)

2.

(A)  Autobiography

(B)  Trapezium

(C)  Cylinder

(D)  Parallelogram

Answer: (B)

3.

(A)  Square

(B)  Trapezium

(C)  Cylinder

(D)  Parallelogram

Answer: (C)

4.

(A)  42 : 4

(B)  48 : 6

(C)  32 : 2

(D)  15 : 5

Answer: (A)

5.

(A)  Year 2012

(B)  Year 1998

(C)  Year 2005

(D)  Year 1997

Answer: (A)

6.

(A)  MIGE

(B)  XTQO

(C)  RNKI

(D)  HDAY

Answer: (A)

7. Arrange the following words as per order in the dictionary-

(1) Euphrasy       (2) Eupepsy

(3) Euphonic      (4) Eugenic

(5) Euphony

(A)  4, 3, 2, 1, 5

(B)  3, 4, 1, 2, 5

(C)  4, 2, 3, 5, 1

(D)  3, 5, 2, 4, 1

Answer: (C)

8. Which one of the given responses would be a meaningful order of the following?

(1) Absorption    (2) Digestion

(3) Nutrition       (4) Excretion

(A)  3, 1, 2, 4

(B)  2,1, 3, 4

(C)  3, 4, 2, 1

(D)  3, 2, 1, 4

Answer: (D)

9. Four villages A, B, C and D lie in a straight line. D is 10 km from B. A is exactly between D and C and C from B is 2 km more than it is from D. How far is C from B?

(A)  4 km

(B)  6 km

(C)  8 km

(D)  2 km

Answer: (B)

10. Select the alternative inference which is most appropriate.

“All professors are learned; learned people are always gentle.”

Inference : All professors are gentle persons.

(A)  The inference is true

(B)  The inference is false

(C)  The inference is probably true or probably false

(D)  The inference is irrelevant

Answer: (A)

11. Statement is given followed by two inferences I and Ii. You have to consider the statement to be true even if it seems to be at variance from commonly known facts. You have to decide which of the given inferences, if any, follow from the given statement.

Statements: “Electric supply in Anand Colony  will be tomorrow after 12 O’clock for three hours because repairing work will be carried out.”

Inferences : (I) Residents of Anand Colony may use their electrical appliances before 12 O’clock tomorrow.

(II) Residents of Anand Colony need training for using electricity economically.

(A)  Only inference I follows

(B)  Only inference II follows

(C)  Both the inferences follow

(D)  None of the inferences follows

Answer: (A)

12. Anand travels 10 km from his home to the east to reach his school. Then he travels 5 km to the south to reach his father’s shop, after school. He then travels 10 km to the west to help his uncle. How far and in which direction is he from his home?

(A)  10 km North

(B)  5 km South

(C)  5 km East

(D)  10 km West

Answer: (B)

13.

 

Answer: (D)

14.

Answer: (B)

15. Which of the following answer figures cannot occur when the question figure given below is rotated?

Answer: (C)

Directions-(Q. 16 and 17) Which of the following Venn diagrams best represents the relation between given classes?

16. Humans, Birds, Animals

Answer: (A)

17. Fruits, Apples, Oranges

Answer: (B)

18. In the diagram given below which letter(s) represents the students who play Cricket as well as Football and Hockey?

(A)  S + T + U

(B)  V

(C)  S

(D)  P + R + U

Answer: (C)

Directions (Q. 19 and 20) Which answer figure will complete the pattern in the question figure?

19.

Answer: (A)

20.

Answer: (A)

21. From the given answer figures, select the one in which the question figure is hidden/embedded.

Answer: (A)

22. A piece of paper is folded and cut as shown below in the question figures. From the given answer figures, indicate how it will appear when opened.

Answer: (C)

23. If a mirror is placed on the line MN, then which of the answer figures is the right image of the given figure?

Answer: (C)

24. A word is represented by only one set of numbers as given in any one of the alternatives. The sets of numbers given in the alternatives are represented by two classes of alphabets as in two matrices given below. The columns and rows of Matrix I are numbered from 0 to 4 and 2 to 6 respectively and that of Matrix II are numbered from 2 to 6 and 7 to 0 respectively. A letter from these matrices can be represented first by its row and next by its column, e.g., ‘H’ can be represented by 04, 25, 32 etc., and ‘N’ can be represented by 21, 40, 59, etc. Similarly, you have to identify the set of the word given below :

(A)  23, 27, 15, 61

(B)  16, 38, 15, 32

(C)  34, 31, 32, 28

(D)  45, 50, 36, 29

Answer: (A)

Directions-(Q. 25  and 26)  The given alternative words, select the word which cannot be formed using the letters of the given word.

25. INSTITUTIONALISE

(A)  NUTRITION

(B)  INTUTION

(C)  TUITION

(D)  INSULATION

Answer: (A)

26. STIMULATION

(A)  STATION

(B)  NATION

(C)  MOTION

(D)  MOUTH

Answer: (D)

27. If TRANSFER is coded as RTNAFSRE, then how ELEPHANT be coded in that code language?

(A)  LEPEHATN

(B)  LEPEAHTN

(C)  LEEPAHTN

(D)  LEPEAHNT

Answer: (B)

28. If MONKO is coded as 57637, then how KLJMN be coded in the same code?

(A)  32456

(B)  34256

(C)  35156

(D)  32546

Answer: (B)

29. In a certain code, TRIPLE is written as SQHOKD. How is DISPOSE written in that code?

(A)  CHRONRD

(B)  DSOESPI

(C)  ESJTPTF

(D)  ESOPSID

Answer: (A)

30. Which letter in the word ‘Vertex’ should be changed to mean spiral movement?

(A)  1st

(B)  2nd

(C)  4th

(D)  Last

Answer: (B)

31. Which of the following interchange of signs would make the equation correct?

8 × 6 + 2 = 22

(A)  +, ×, 2 and 6

(B)  +, ×, 2 and 8

(C)  +, ×, 6 and 8

(D)  +, ×, 2 and 22

Answer: (C)

Directions-(Q. 32-35) A series is given, with one term missing. Choose the correct alternative from the given ones that will complete the series.

32. DHK, GKN, JNQ, ?

(A)  MRU

(B)  MQT

(C)  LPS

(D)  NRU

Answer: (B)

33. 1, 5, 25, 125, ?, ?, ?

(A)  245, 485, 965

(B)  225, 325, 425

(C)  625, 3225, 15605

(D)  625, 3125, 15625

Answer: (D)

34. 0, ?, 8, 27, 64, 125.

(A)  5

(B)  4

(C)  2

(D)  1

Answer: (D)

35. BMX, DNW, FOU, ?

(A)  GHO

(B)  GPS

(C)  HPS

(D)  HPT

Answer: (D)

36. If a number is greater than 5 but less than 9 and greater than 7 but less than 11, the number is-

(A)  5

(B)  6

(C)  7

(D)  8

Answer: (D)

37. In a row, 25 trees are planted at equal distance from each other. The distance between 1st and 25th tree is 30 m. What is the distance between 3rd and 15th tree?

(A)  8 m

(B)  15 m

(C)  16 m

(D)  18 m

Answer: (B)

38. In a class composed of x girls y boys what part of the class is composed of girls?

(A)  y(x + y)

(B)  x/xy

(C)  x/(x+y)

(D)  y/xy

Answer: (C)

39. If ‘−’ stands for addition, ‘+’ stands for subtraction, ‘÷’ stands for multiplication and ‘×’ stands for division, then which one of the following equation is correct?

(A)  50 × 5 ÷ 2 – 30 + 25 = 25

(B)  50 – 30 + 5 ÷ 2 × 30 = 25

(C)  40 + 35 × 2 – 50 ÷ 30 = 95

(D)  30 × 2 – 25 + 50 ÷ 5 = 100

Answer: (A)

Directions – (Q. 40 and 41)  Some questions are solved on the basis of certain system. Find out the correct answer for the unsolved equation on that basis.

40. 2 × 4 × 6 = 4; 9 × 3 × 7 = 13;

4 × 7 × 6 = 3; 9 × 7 × 8 = ?

(A)  10

(B)  09

(C)  08

(D)  07

Answer: (A)

41. 3 × 5 × 7 × 2 = 24, 2× 4 × 6 × 8 = 22, 4 × 4 × 8 × 9 = ?

(A)  33

(B)  25

(C)  144

(D)  1152

Answer: (A)

Directions-(Q. 42-44) Find the missing number from the given responses.

42.

7     8          5

6      9          ?

2       3          6

84     216      900

(A)  90

(B)  70

(C)  65

(D)  30

Answer: (D)

43.

2           3          4          ?

3             1          4          2

4             2          2          6

5             5          2          2

196         121      144      225

(A)  4

(B)  5

(C)  6

(D)  3

Answer: (B)

44.

96           ?          168

32           48        56

16           24        28

(A)  52

(B)  144

(C)  64

(D)  38

Answer: (B)

Directions-(Q. 45-50) Select the related word/letter/numbers from the given alternatives.

45. Macabre : Lovely : : Baneful : ?

(A)  Unharmful

(B)  Churlish

(C)  Filter

(D)  Ugly

Answer: (A)

46. AYRRJC : CATTLE : NCPDCAR : ?

(A)  SUBJECT

(B)  NEGLECT

(C)  PERFECT

(D)  OPERATE

Answer: (C)

47. SOCIAL : OCIALS : DRIVEN : ?

(A)  VENRID

(B)  NEVIRD

(C)  RIVEND

(D)  VIREND

Answer: (C)

48. 

(A)  128

(B)  126

(C)  144

(D)  132

Answer: (A)

49. 29 : 71 : : 79 : ?

(A)  120

(B)  192

(C)  197

(D)  131

Answer: (D)

50. Embarrassed : Humiliated :: Frightened : ?

(A)  Terrified

(B)  Agitated

(C)  Courageous

(D)  Reckless

Answer: (A)

Part-B

General Awareness

51. When is International ‘Yoga Divas’ is celebrated ?

(A)  21st June

(B)  25th April

(C)  21st May

(D)  21 July

Answer: (A)

52. Cyanide poisoning causes death in seconds because-

(A)  It breaks the electron transport chain

(B)  It denatures enzymes of the heart/muscle

(C)  It causes cardiac arrest

(D)  It causes Lysis of red blood cells

Answer: (C)

53. Who Directed the film on the dacoit queen Phoolan Devi?

(A)  Shashi Kapoor

(B)  Shekhar Kapoor

(C)  Abbas Mastan

(D)  Anil Kapoor

Answer: (B)

54. In which year did Dad Saheb Phalke produce the first Feature film?

(A)  1911

(B)  1910

(C)  1912

(D)  1913

Answer: (D)

55. The earth completes one rotation on its axis in-

(A)  23 hrs. 56 min. 4.9 sec.

(B)  23 hrs. 10min. 2 sec

(C)  24 hrs.

(D)  23 hrs. 30 min.

Answer: (A)

56. The technology used in the electronic printer is called-

(A)  Micro encapsulation

(B)  Micro technology

(C)  Micro array

(D)  Micro millimetric

Answer: (B)

57. For a missile launched with a velocity less than the earth’s escape velocity, the total energy is-

(A)  Positive

(B)  Negative

(C)  Either positive or negative

(D)  Zero

Answer: (A)

58. Which one of the following is the correct sequence of ecosystem in the order of decreasing productivity-

(A)  Mangroves, oceans, grassland, lakes

(B)  Oceans, lakes, grassland, mangroves

(C)  Mangroves, grasslands, lakes, oceans

(D)  Oceans, mangroves, lakes, grassland

Answer: (C)

59. The classical Dance of Andhra Pradesh is-

(A)  Kathakali

(B)  Odissi

(C)  Bharatanatyam

(D)  Kuchipudi

Answer: (D)

60. The Mediterranean region are characterized by heavy rain in-

(A)  Spring

(B)  Autumn

(C)  Winter

(D)  Summer

Answer: (C)

61. Which was the first hydel power project in India?

(A)  Siva Samudram in Karnataka

(B)  Nizamnagar in Andhra Pradesh

(C)  Paikara in Tamilnadu

(D)  Palli vassal in Kerala

Answer: (A)

62. Molybdenum deficiency affects the activity of-

(A)  Nitrogenase

(B)  Chlorate reductase

(C)  Nitrate reductase

(D)  All of the given options

Answer: (C)

63. Autocracy means-

(A)  Absolute rule by one

(B)  Rule by few

(C)  Rule by the representatives of the People

(D)  Rule by King

Answer: (A)

64. Waksman go the Noble prize for the discovery of-

(A)  Streptomycin

(B)  Chloromycetin

(C)  Penicillin

(D)  Neomycin

Answer: (A)

65. Spring tides occur on-

(A)  New moon day only

(B)  Full moon day only

(C)  The day when the moon’s position is in its first

(D)  Full moon days as well as on new moon day

Answer: (D)

66. What is the fascist view of state?

(A)  Nation state is unquestionably sovereign

(B)  State is the instrument of exploitation in the hands of few

(C)  State is under the control of a king

(D)  State enhances the ideal of individualism

Answer: (B)

67. Which of the following can be found as pollutants in the drinking water in some parts of India-

Select the correct answer using the code given below.

(1) Arsenic         (2) Sorbitol      (3) Fluoride     (4) Formaldehyde       (5) Uranium

(A)  1 and 3

(B)  1, 2, 3, 4 and 5

(C)  2, 4 and 5

(D)  1, 3 and 5

Answer: (A)

68. Who is considered founder of the Gupta Empire?

(A)  Chandra Gupta

(B)  Srigupta

(C)  Samudra Gupta

(D)  Chandra Gupta II

Answer: (B)

69. When one gene pair hides the effect of the other unit, the phenomenon is referred to as-

(A)  Mutation

(B)  Epistasis

(C)  Dominance

(D)  None of the options

Answer: (C)

70. State Bank of India was previously known as-

(A)  Canara Bank

(B)  Imperial Bank of India

(C)  Syndicate Bank

(D)  Co-operative Bank of India

Answer: (B)

71. “Don’t Laugh : We are Police”, This book was compiled by-

(A)  K.P.S. Gill D.G.P.

(B)  Bishan Lal Vohra I.G.P.

(C)  Shanti Swaroop I.G. Police

(D)  None of the options

Answer: (B)

72. Indian first talkie film produced in 1931 was-

(A)  Almara

(B)  Indra Sabha

(C)  Shakuntala

(D)  Neel Kamal

Answer: (A)

73. The name of the upper house of the Indian Parliament is-

(A)  Legislative assembly

(B)  Rajya Sabha

(C)  House of Lords

(D)  Senate

Answer: (B)

74. Microbial Type Culture Collection Centre is situated at-

(A)  Chandigarh

(B)  New Delhi

(C)  Hyderabad

(D)  Bangalore

Answer: (A)

75. Who discovered the first antibiotic?

(A)  Louis Pastour

(B)  A Fleming

(C)  W Fleming

(D)  C Waksman

Answer: (B)

76. In the grass lands, trees do not replace the grasses as a part of an ecological succession because of-

(A)  Water limits and fire

(B)  Limited sun light and paucity of nutrients

(C)  Insect and fungi

(D)  None of the above

Answer: (A)

77. The business in Stock Market and other securities markets is regulated by-

(A)  Stock and Exchange Bank of India

(B)  Securities and Exchange Board of India

(C)  Sole Trade and Exchange Bank of India

(D)  State and exchange Bank of India

Answer: (B)

78. The Women’s Reservation Bill seeks how much reservation for women in the State assemblies and Lok Sabha?

(A)  30%

(B)  36%

(C)  25%

(D)  33%

Answer: (D)

79. The oldest form of composition of Hindustani vocal music is-

(A)  Ghazal

(B)  Thumri

(C)  Dhrupad

(D)  None of these

Answer: (C)

80. G E F, an international aid- giving agency has the full form-

(A)  Global Environment Fund

(B)  Global Energy Fund

(C)  Global Economic Fund

(D)  Global Educational Fund

Answer: (A)

81. On 2nd June 2015 a Conference on Make in India-Indigenisation of currency was organized by the department of Economic Affairs, Ministry of Finance, govt. of India. Who inaugurated the Conference?

(A)  Finance Secretary

(B)  Finance Minister

(C)  Prime Minister

(D)  Governor R.B.I.

Answer: (B)

82. A Fuse wire is characterized by-

(A)  Low resistance and high melting point

(B)  High resistance and low melting point

(C)  Low resistance and low melting point

(D)  High resistance and high melting point

Answer: (B)

83. The ore of Aluminium is-

(A)  Hematite

(B)  Bauxite

(C)  Fluorspar

(D)  Chalco pyrites

Answer: (B)

84. Heavy metals got their name because compared to other atoms they have-

(A)  Higher atomic masses

(B)  Higher denisties

(C)  Higher atomic numbers

(D)  Higher atomic radii

Answer: (B)

85. Fascism believes in the application of the principle of-

(A)  Dictatorship

(B)  Utilitarianism

(C)  Democracy

(D)  Totalitarianism

Answer: (D)

86. Raga Kameshwari was composed by-

(A)  Pandit Ravi Shankar

(B)  Ustad Amzad Ali Khan

(C)  Uday Shankar

(D)  None of these

Answer: (A)

87. The Dynamo converts-

(A)  Electrical energy into Mechanical Energy

(B)  Mechanical energy into Electrical Energy

(C)  Mechanical energy into Magnetic Energy

(D)  None of these

Answer: (B)

88. What is ordinary Law?

(A)  Laws made by the common people

(B)  Laws made and enforced by the Government

(C)  Laws made by the High Court

(D)  Laws made by the Supreme Court

Answer: (B)

89. Photo chemical smog is a resultant of the reaction among-

(A)  High concentration of NO2, O3 and CO in the evening

(B)  NO2, O3 and peroxy acetyl nitrate in the presence of Sunlight

(C)  CO, CO2 and NO2 at low temperature

(D)  CO, O2 and peroxy acetyl nitrate in the presence of Sunlight

Answer: (B)

90. The father of computer is-

(A)  Charles Babbage

(B)  Oliver twist

(C)  Love lice

(D)  Charles Dickens

Answer: (A)

91. Which of the following costs is related to marginal cost?

(A)  Fixed Cost

(B)  Implicit Cost

(C)  Prime Cost

(D)  Variable Cost

Answer: (D)

92. Laberalism stands for-

(A)  A movement & an attitude

(B)  Self-emancipation

(C)  Freedom in social, political and economic aspects

(D)  Religious orthodoxy

Answer: (C)

93. Mahatma Gandhi began his political activities in India first from-

(A)  Kheda

(B)  Champaran

(C)  Dandi

(D)  Sabarmati

Answer: (B)

94. When a helium atom loses an electron it becomes-

(A)  A proton

(B)  A positive helium ion

(C)  A negative helium ion

(D)  An alpha particle

Answer: (B)

95. The Environment (Protection) Bill was passed by the Parliament of India in-

(A)  1986

(B)  1981

(C)  1984

(D)  1972

Answer: (A)

96. The Battle of Plassey was fought in year-

(A)  1775

(B)  1757

(C)  1576

(D)  1761

Answer: (B)

97. Xenobiotics which are inherently resistant to microbial attack are called as-

(A)  Biodegradable

(B)  Persistent

(C)  Recalcitrant

(D)  All of the given options

Answer: (C)

98. PSW stands for-

(A)  Program status word

(B)  Process status word

(C)  Primitive status word

(D)  Processor status word

Answer: (A)

99. Earthquakes are caused by-

(A)  Denudation

(B)  Earth revolution

(C)  Tectonism

(D)  Earth rotation

Answer: (C)

100. The Liquidity Preference Theory of Interest was propounded by-

(A)  Adam Smith

(B)  David Ricardo

(C)  Alfred Marshall

(D)  J.M. Keynes

Answer: (D)

Part-C

Quantitative Aptitude

101. If angle bisector of a triangle bisect the opposite side, then what type of triangle is-

(A)  Right angled

(B)  Scalene

(C)  Similar

(D)  Isosceles

Answer: (D)

102. If x[−2{−4(−a)}] + 5[−2{−2(−a)}] = 4a, then x =

(A)  −2

(B)  −3

(C)  −4

(D)  −5

Answer: (B)

103. A can complete a work in ‘m’ days and B can complete it in ‘n’ days. How many days will it take to complete the work if both A and B work together?

(A)  (m + n)days

(B) 

(C) 

(D) 

Answer: (D)

104. Pipe A is an inlet pipe filling the tank at 8000 litre/hr. Pipe B empties the tank in 3 hours. The capacity of the tank is-

(A)  12000 litres

(B)  8000 litres

(C)  6000 litres

(D)  4000 litres

Answer: (*)

105. A tap drips at a rate of one drop/ sec. 600 drops make 100 ml. The number of litres wasted in 300 days is-

(A)  4320000

(B)  432000

(C)  43200

(D)  4320

Answer: (A)

106. ABC is a right angled triangle, B being the right angle. Mid-points of BC and AC are respectively B’ and A’. Area of ∆A’B’C is-

(A) 

(B) 

(C) 

(D) 

Answer: (C)

107. Three numbers are in the ratio 1 : 2 : 3 and their HCF is 12. The numbers are-

(A)  12, 24, 36

(B)  5, 10, 15

(C)  4, 8, 12

(D)  10, 20, 30

Answer: (A)

108. The perimeters of a circle, a square and an equilateral triangle are same and their areas are C, S and T respectively. Which of the following statement is true?

(A)  C = S= T

(B)  C > S > T

(C)  C < S < T

(D)  S < C < T

Answer: (B)

109. The list price of a shirt is Rs 440 and a customer pays, Rs 396 for it. The discount rate is-

(A)  10%

(B) 

(C)  20%

(D)  12%

Answer: (A)

110. A plate was sold for Rs 6,300 after giving two successive discounts of  and 10%. Find the marked price-

(A)  Rs 7,300

(B)  Rs 7,700

(C)  Rs 8,000

(D)  Rs 7,250

Answer: (C)

111. To attract more visitors, Zoo authority announces 20% discount on every ticket which costs 25 p. For this reason, sale of ticket increases by 28%. Find the percentage of increase in the number of visitors.

(A)  40%

(B)  50%

(C)  60%

(D)  No change

Answer: (C)

112. Which of the following represents a correct proportion?

(A)  12 : 9 = 16 : 12

(B)  13 : 11 = 5 : 4

(C)  30 : 45 = 13 : 24

(D)  3 : 5 = 2 : 5

Answer: (A)

113. A sphere is cut into two hemispheres. One of them is used as bowl. It takes 8 bowlfuls of this to fill a conical vessel of height 12 cm and radius 6 cm. The radius of the sphere (in centimeter) will be-

(A)  3

(B)  2

(C)  4

(D)  6

Answer: (A)

114. If sin θ = 3/5, then the value of  is equal to-

(A)  29/60

(B)  31/60

(C)  34/60

(D)  37/60

Answer: (B)

115. If acos θ + b sin θ = p and asin θ – bcos θ = q, then the relation between a, b, p and q is-

(A)  a2 – b2 = p2 q2

(B)  a2 + b2 = p2 + q2

(C)  a + b = p + q

(D)  a – b = p – q

Answer: (B)

116. From a point P on the ground the angle of elevation of the top of a 10 m tall building is 30°, A flag is hoisted at the top of the building and the angle of elevation of the top of the flagstaff from P is 45°. Find the length of the flagstaff from P is 45°. Find the length of the flagstaff. (Take √3 = 1.732)

(A)  10(√3 + 2) m

(B)  10(√3 + 1) m

(C)  10√3

(D)  7.32 m

Answer: (D)

117. The value of  is equal to-

(A)  0

(B)  1

(C)  2

(D)  3/2

Answer: (A)

118. 60 students were asked to choose their favourite sport. Their distribution is as follows-

Football – 15

Cricket – 12

Basketball – 11

Athletics – 10

The data is illustrated in a Pie Chart. What angle should be used for football?

(A)  15°

(B)  60°

(C)  90°

(D)  180°

Answer: (C)

119. If in a triangle ABC as drawn in the figure, AB = AC and ∠A is equal to-

(A)  50°

(B)  60°

(C)  70°

(D)  80°

Answer: (B)

Directions-(Q. 120-125) Study the graph and answer.

120. In which year the sale of cool-sip is minimum?

(A)  1990

(B)  1992

(C)  1993

(D)  None of these

Answer: (D)

121. In case of which soft drink was the average annual sale maximum during the period 1988-1993?

(A)  Pep-up only

(B)  Pep-up and Dew-drop

(C)  Cool-sip only

(D)  Cool-sip and Pep-up

Answer: (A)

122. What was the approximate per cent drop in the sale of Pep-up in 1990 over its sale in 1989?

(A)  5

(B)  14

(C)  12

(D)  20

Answer: (B)

123. What was the approximate per cent increase in sales of Cool-sip in 1990 over its sales in 1989?

(A)  100

(B)  50

(C)  171

(D)  150

Answer: (C)

124. In which year sale of Dew-drop is maximum?

(A)  1988

(B)  1992

(C)  1989

(D)  1993

Answer: (B)

125. In case of which soft drink was the average annual sale minimum during the period 1988-1993?

(A)  Pep-up only

(B)  Cool-sip only

(C)  Dew-drop only

(D)  Dew-drop and Cool-sip

Answer: (C)

126. In a Mathematics examination the numbers scored by 5 candidates are 5 successive odd integers. If their total marks is 185, the highest score is-

(A)  39

(B)  43

(C)  41

(D)  37

Answer: (C)

127. In two successive years, 80 and 60 students of a school appeared at the final examination of which 60% and 80% passed respectively. The average rate of students passed (in per cent) is-

(A)  68%

(B) 

(C)  70%

(D) 

Answer: (B)

128. Nisha bought a number of oranges at 2 for a rupee and an equal number at 3 for a rupee. To make a profit of 20% she should sell a dozen for-

(A)  Rs 6

(B)  Rs 8

(C)  Rs 10

(D)  Rs 12

Answer: (A)

129. If A’s salary is 50% more than that of B, then B’s salary is less than A’s by-

(A)  33%

(B) 

(C) 

(D) 

Answer: (D)

130. A and B are 20 km apart. A can walk at an average speed of 4 km/hour and B at 6 km/hr. If they start walking towards each other at 7 a.m., when they will meet?

(A)  8.00 a.m.

(B)  8.30 a.m.

(C)  9.00 a.m.

(D)  10.00 a.m.

Answer: (C)

131. A policeman starts to chase a thief. When the thief goes 10 steps the policeman moves 8 steps. 5 steps of the policeman is equal to 7 steps of the thief. The ratio of the speeds of the policeman and the thief is-

(A)  25 : 28

(B)  25 : 56

(C)  28 : 25

(D)  56 : 25

Answer: (C)

132. The compound interest on a certain sum of money for 2 years at 5% per annum is Rs 410. The simple interest on the same sum at the same rate and for the same time is-

(A)  Rs 400

(B)  Rs 300

(C)  Rs 350

(D)  Rs 405

Answer: (A)

133. The graphs of x = a and y = b intersect at-

(A)  (a, b)

(B)  (b, a)

(C)  (−a, b)

(D)  (a, −b)

Answer: (A)

134. What is the value of 

(A)  10

(B)  2

(C)  1

(D)  100

Answer: (B)

135. If  is-

(A)  12098

(B)  12048

(C)  14062

(D)  12092

Answer: (A)

136. If 5√5 + 53 ÷ 53/2 = 5a+2, then value of a is-

(A)  4

(B)  5

(C)  6

(D)  8

Answer: (A)

137. If x2 – 3x + 1 = 0, then the value of  will be-

(A)  18

(B)  15

(C)  21

(D)  30

Answer: (C)

138. A boat goes 24 km upstream and 28 km downstream in 6 hours. It goes 30 km upstream and 21 km downstream in 6 hours and 30 minutes. The speed of the boat in still water is

(A)  8 km/hr

(B)  9 km/hr

(C)  12 km/hr

(D)  10 km/hr

Answer: (D)

139. If  and x > 1, then find the positive value of 

(A)  25

(B)  27

(C)  36

(D)  49

Answer: (C)

140. The value of (3 + 2√2)3 + (3 – 2√2)3 is-

(A)  198

(B)  180

(C)  108

(D)  189

Answer: (A)

141. ‘O’ is the centre of the circle, AB is a chord of the circle, OM ⊥ If AB = 20 cm, OM = 2√11 cm, then radius of the circle is-

(A)  15 cm

(B)  12 cm

(C)  10 cm

(D)  11 cm

Answer: (B)

142. If the angles of a triangle ABC are in the ratio 2 : 3 : 1, then the angles ∠A, ∠B, ∠C is-

(A)  ∠A = 60°, ∠B = 90°, ∠C = 30°

(B)  ∠A = 40°, ∠B = 120°, ∠C = 20°

(C)  ∠A = 20°, ∠B = 60°, ∠C = 60°

(D)  ∠A = 45°, ∠B = 90°, ∠C = 45°

Answer: (A)

143. In ∆ABC, ∠ABC = 70°, ∠BCA = 40°. O is the point of intersection of the perpendicular bisectors of the sides, then the angle ∠BOC is-

(A)  100°

(B)  120°

(C)  130°

(D)  140°

Answer: (D)

144. If the measures of the sides of triangle are (x2 – 1). (x2 + 1) and 2x cm, then the triangle would be-

(A)  equilateral

(B)  acute-angled

(C)  isosceles

(D)  right-angled

Answer: (D)

145. 2x = 4y = 8z and xyz = 288, then value of  is-

(A)  11/12

(B)  11/96

(C)  29/96

(D)  27/96

Answer: (B)

146. If each angle of a triangle is less than the sum of the other two, then the triangle is-

(A)  obtuse angled

(B)  right angled

(C)  acute angled

(D)  equilateral

Answer: (C)

147. A, B, C are three points on the circumference of a circle and if   and ∠BAC = 90°, find the radius-

(A)  10 cm

(B)  5 cm

(C)  20 cm

(D)  15 cm

Answer: (B)

148. The value of  is-

(A) 

(B) 

(C) 

(D) 

Answer: (A)

149. If  then the value of cos4 θ + sin4 θ is-

(A)  1/3

(B)  2/3

(C)  1/9

(D)  2/9

Answer: (A)

150. If  then the value of  is-

(A)  3/4

(B)  4/5

(C)  5/6

(D)  6/7

Answer: (C)

Part-D

English Comprehension

Directions-(Q. 151-155) You have a passage with 5 questions following. Read the passage carefully and choose the best answer to each question out of the four alternatives and mark it by blackening the appropriate circle in t he Answer Sheet.

PASSAGE

   The public distribution system, which provides food at low prices, is a subject of vital concern. There is a growing realization that though India has enough food to feed its masses two square meals a day, the monster of starvation and food insecurity continues to haunt the poor in our country.

Increasing the purchasing power of the poor through providing productive employment leading to rising income, and thus good standard of living is the ultimate objective of public policy. However, till then there is a need to provide assured supply of food through a restructured, more efficient and decentralized public distribution system (PDS). Although the PDS is extensive it is one of the largest such systems in the world- it hasn’t reached the rural poor and the remote places. It remains an urban phenomenon, with the majority of the rural poor still out of its each due to lack of economic and physical access. The poorest in the cities and the migrants are left out, for they generally do not possess ration cards. The allocation of PDS supplies in big cities is larger than in rural areas. In view of such deficiencies in the system, the PDS urgently needs to be streamlined. Also, considering the large foodgrain production combined with food subsidy on one hand and the continuing slow starvation and dismal poverty of rural population on the other, there is a strong case for making PDS target-group oriented. By making PDS target-group oriented, not only the poorest and the neediest would be reached without additional cost but we can also reduce the overall costs incurred.

151. What should be an appropriate step to make the PDS effective?

(A)  To reduce administrative cost

(B)  To decrease the allotment of food grains

(C)  To make it target-group oriented

(D)  To increase the amount of foodgrain per ration card

Answer: (C)

152. The full form of PDS is-

(A)  Public distribution system

(B)  Party distribution system

(C)  Private distribution system

(D)  Partial distribution system

Answer: (A)

153. Which of the following is true of public distribution system?

(A)  It has improved its effectiveness over the year.

(B)  It has reached the remotest corner of the country.

(C)  It is unique in the world because of its effectiveness.

(D)  It has remained effective only in the cities.

Answer: (D)

154. The public distribution system, which provides food at ……. is a subject of vital concern.

(A)  fair prices

(B)  high prices

(C)  low prices

(D)  as per capital income

Answer: (C)

155. What according to the passage, would be the rationale of making the PDS target-group oriented?

(A)  It will abolish the imbalance of urban and rural sectors.

(B)  It will remove poverty.

(C)  It will give food to the poorest section without additional cost.

(D)  It will motivate the target-group population to work more.

Answer: (C)

Directions-(Q. 156-158) Choose the word opposite in meaning to the given word and mark it in the answer sheet.

156. BOLD

(A)  Nervous

(B)  Fearful

(C)  Timid

(D)  Coy

Answer: (C)

157. AUTHENTIC

(A)  Genuine

(B)  False

(C)  Factual

(D)  Real

Answer: (B)

158. URBANE

(A)  Crude

(B)  Rural

(C)  Native

(D)  Loud

Answer: (A)

Directions-(Q. 159-161) Out of the four alternatives, choose the one which best expresses the meaning of the given word and mark it in the answer sheet.

159. FRUGAL

(A)  Economical

(B)  Plain

(C)  Miserly

(D)  Simple

Answer: (A)

160. DIMINISH

(A)  Prohibit

(B)  Worsen

(C)  Shorten

(D)  Reduce

Answer: (D)

161. PLEBISCITE

(A)  Renunciation

(B)  Reservation

(C)  Representation

(D)  Referendum

Answer: (D)

Directions-(Q. 162 and 163)  Four words are given in each question, out of which only one word is correctly spelt. Find the correctly spelt word and mark your answer in the Answer Sheet.

162. 

(A)  aneurysm

(B)  disipate

(C)  carcas

(D)  annonymous

Answer: (D)

163. 

(A)  tommorow

(B)  commendation

(C)  coherant

(D)  advicable

Answer: (B)

Directions-(Q. 164-168) Sentences are given with blanks to be filled in with an appropriate word(s). Four alternative are suggested for each question. Choose the correct alternative out of the four and indicate it by blackening the appropriate circle in the answer sheet.

164. By morning, the fury of the floods……..

(A)  retired

(B)  ebbed

(C)  receded

(D)  abated

Answer: (D)

165. She pipped her rival ……… the gold medal.

(A)  for

(B)  with

(C)  to

(D)  near

Answer: (A)

166. Ships are ………… by giant engines.

(A)  dragged

(B)  sailed

(C)  driven

(D)  pulled

Answer: (C)

167. Work should be guided by some ……….. desire of altruism and …………

(A)  shallow; ability

(B)  false; honesty

(C)  genuine; philanthropy

(D)  good; liberty

Answer: (C)

168. Krisda’s stories…………..me very strangely.

(A)  afflicted

(B)  effected

(C)  changed

(D)  affected

Answer: (B)

Directions-(Q. 169-173) You have a passage with 5 questions following. Read the passage carefully and choose the best answer to each question out of the four alternatives and mark it by blackening the appropriate circle in the Answer Sheet.

PASSAGE

The desert floras shame us with their cheerful adaptations to the seasonal limitations. Their whole duty is to flower and fruit, and they do it hardly, or with tropical luxuriance, as the rain admits. It is recorded in the report of the Death Valley expedition that after a year of abundant rains, on the Colorado desert was found a specimen of Amaranthus ten feet high. A year later the same species in the same place matured in the drought at four inches. Seldom does the desert herb attain the full stature of the type. Extreme aridity and extreme altitude have the same dwarfing effect, so that we find in the high Sierras and in Death Valley related species in miniature that reach a comely growth in mean temperatures. Very fertile are the desert plants in expedients to prevent evaporation, turning their foliage edge-wise toward the sun, growing silky hairs, exuding thick gum. The wind, which has a long sweep, harries and helps them. It rolls up dunes about the stocky stems, encompassing and protective, and above the dunes, which may be, as with the mesquite, three times as high as a man, the blossoming twigs flourish and bear fruit.

169. The mesquite is a-

(A)  a sand dune

(B)  a tribe of people

(C)  a type of desert animal

(D)  a desert flora

Answer: (D)

170. What stops the desert floras from performing their duty well?

(A)  The rain

(B)  The desert animals

(C)  The desert sand

(D)  The people who pluck them

Answer: (C)

171. What lesson do the desert floras have to teach us?

(A)  The rain

(B)  The desert animals

(C)  The desert sand

(D)  The people who pluck them

Answer: (A)

172. How does the wind keep the desert floras to grow?

(A)  By rolling up protective sand dunes

(B)  By blowing gently

(C)  By blowing the clouds away

(D)  By blowing the heat away

Answer: (A)

173. The desert plants face the danger of……….from extreme aridity and extreme altitude.

(A)  early death

(B)  loss of reproduction

(C)  dwarfism

(D)  painful growth

Answer: (C)

   Directions-(Q. 174-178) Some parts of the sentences have errors and some are correct. Find out which part of a sentence has an error and blacken the circle corresponding to the appropriate correct option. If a sentence is free from error, blacken the circle corresponding to ‘No error’ option in the Answer Sheet.

174. The ruins of the spillway are a vital clue to the epic struggle that unfold as generations of Khmer engineers coped with a water system that grew complex and unruly.

(A)  To the epic struggle that unfold as generations of Khmer engineers

(B)  Coped with a water system that grew complex and unruly

(C)  No error

(D)  The ruins of the spillways are a vital clue

Answer: (D)

175. The sum and substance of this poem is as follows-

(A)  No error

(B)  The sum and substance

(C)  Is as follows

(D)  Of this poem

Answer: (A)

176. I watched how the pianoist used her left hand-

(A)  No error

(B)  Used her left hand

(C)  I watched how

(D)  The pianoist

Answer: (B)

177. Bobby leant the alphabets at the age of two-

(A)  Age of two

(B)  No error

(C)  Alphabets at the

(D)  Bobby learnt the

Answer: (C)

178. You are required to give an explanation for your conduct within two days of the receipt of this letter-

(A)  You are required to give an explanation

(B)  For your conduct

(C)  No error

(D)  Within two days of the receipt of this letter

Answer: (D)

Directions-(Q. 179-183) Four alternatives  are given for the idiom/phrase bold in the sentence. Choose the alternative which best expresses the meaning of the idiom/phrase and mark it in the answer sheet.

179. I shall always remember my alma mater with gratitude.

(A)  Kindergarten days

(B)  Teacher who inspired me

(C)  Institution where I got education

(D)  Mother’s loving care

Answer: (C)

180. He’s as daft as a brush. Don’t believe a word of what he says.

(A)  Unreliable

(B)  Very funny

(C)  Really silly

(D)  A liar

Answer: (C)

181. I hope you will back me at the meeting.

(A)  Support

(B)  Follow

(C)  Criticise

(D)  Speak after I do

Answer: (A)

182. I and my fried always god Dutch when we eat out.

(A)  Drive together

(B)  Divide the costs

(C)  Pay for each other’s meal

(D)  Go together

Answer: (B)

183. A closefisted man does not know the significance of human life.

(A)  A miser

(B)  An ill-mannered man

(C)  A poor man

(D)  A rich man

Answer: (A)

   Directions-(Q. 184-193) A sentence/a part of the sentence is bold. Below are given alternatives to the bold part which may improve the sentence. Choose the correct alternative. In case no improvement is needed choose ‘No Improvement’. Mark your answer in the answer sheet.

184. The notorious criminal went to the police to go to prison.

(A)  No improvement

(B)  surrendered himself before the police

(C)  gave himself up for the police

(D)  submitted to the police

Answer: (B)

185. The glass figurine that was being shown in the store window appealed to me.

(A)  that was advertised

(B)  that was on display

(C)  that was exhibited

(D)  No improvement

Answer: (B)

186. He is shomehow tall for his age.

(A)  many

(B)  much

(C)  No improvement

(D)  rather

Answer: (D)

187. The practice of starving the children in order to cure diarrhea also aggravates the situation.

(A)  No improvement

(B)  Starving children

(C)  Satrve children

(D)  Starving child

Answer: (A)

188. We do not believe in a dual policy of the company.

(A)  in these dual

(B)  No improvement

(C)  on these dual

(D)  in this dual

Answer: (D)

189. She said that she was glad to be here that evening.

(A)  No improvement

(B)  to be there

(C)  to come here

(D)  to have been here

Answer: (B)

190. This news is too good to be true.

(A)  so good that it should be true

(B)  cannot be true

(C)  so good that it cannot be true

(D)  No improvement

Answer: (D)

191. I did not see you for a long time.

(A)  have not been seeing

(B)  saw

(C)  have not seen

(D)  No improvement

Answer: (C)

192. If he has time he will

(A)  would

(B)  might

(C)  No improvement

(D)  could

Answer: (C)

193. He met European lady at the conference who works for an NGO-

(A)  European lady who works for an NGO at the conference

(B)  An European lady at the conference who works for an NGO

(C)  No improvement

(D)  A European lady who works for an NGO, at the conference

Answer: (D)

Directions-(Q. 194-200) Out of four alternatives, choose the one which can be substituted for the given words/sentences and indicate it b y blackening the appropriate circle in the answer sheet.

194. A person who deliberately sets fire to a building-

(A)  Assassin

(B)  Hijacker

(C)  Arsonist

(D)  Extortionist

Answer: (C)

195. That which cannot be effaced-

(A)  Invincible

(B)  Indelible

(C)  Illegible

(D)  Affable

Answer: (B)

196. The burial of a corpse-

(A)  Interment

(B)  Internment

(C)  Interrogate

(D)  Interpose

Answer: (A)

197. Conferred as an honor-

(A)  Honorary

(B)  Honorific

(C)  Honorable

(D)  Honorarium

Answer: (A)

198. One who makes an official examination of accounts-

(A)  Registrar

(B)  Creditor

(C)  Auditor

(D)  Chartered Accountant

Answer: (C)

199. One who does not believe in the existence of God-

(A)  Atheist

(B)  Theist

(C)  Mystic

(D)  Cynic

Answer: (A)

200. To free someone from all blames-

(A)  Consolidate

(B)  Exonerate

(C)  Forbid

(D)  Fling

Answer: (B)

© Copyright Entrance India - Engineering and Medical Entrance Exams in India | Website Maintained by Firewall Firm - IT Monteur